Bariatric Surgery + Medical Therapy: Effective Tx for T2DM?

Article Type
Changed
Tue, 05/03/2022 - 15:14
Display Headline
Bariatric Surgery + Medical Therapy: Effective Tx for T2DM?

Practice Changer

A 46-year-old woman presents with a BMI of 28, a 4-year history of type 2 diabetes mellitus (T2DM), and an A1C of 9.8%. The patient is currently being treated with intensive medical therapy (IMT), including metformin 2000 mg/d, sitagliptin 100 mg/d, and insulin glargine 12 U/d, with minimal change in A1C. Should you recommend bariatric surgery?

One in 11 Americans has diabetes, and at least 95% of those have T2DM.2,3 The treatment of T2DM is generally multimodal to target the various mechanisms that cause hyperglycemia. Strategies may include making lifestyle modifications, decreasing insulin resistance, increasing insulin secretion, replacing insulin, and targeting incretin-hormonal pathways.

The American Diabetes Association (ADA) recommends diet, exercise, and behavioral modifications as firstline therapy for diabetes management, but these methods are often inadequate.2 In addition to various pharmacotherapeutic strategies for some populations with T2DM, the ADA recommends bariatric surgery for those with a BMI ≥ 35 and uncontrolled hyper­glycemia.2,4

However, this recommendation is based only on short-term studies. For example, in a single-center, nonblinded RCT of 60 patients with a BMI ≥ 35, the average baseline A1C levels of 8.65 ± 1.45% were reduced to 7.7 ± 0.6% in the IMT group and to 6.4 ± 1.4% in the gastric-bypass group at 2 years.5 In another study, a randomized double-blind trial involving 60 moderately obese patients (BMI, 25-35), gastric bypass yielded better outcomes than sleeve gastrectomy: 93% of patients in the former group and 47% of those in the latter group achieved remission of T2DM over a 12-month period.6

The current study by Schauer et al examined the long-term outcomes of IMT alone vs bariatric surgery with IMT for the treatment of T2DM in patients who are overweight or obese.1

STUDY SUMMARY

5-year follow-up: surgery + IMT works

This study was a 5-year follow-up of a nonblinded, single-center RCT comparing IMT alone to IMT with Roux-en-Y gastric bypass or sleeve gastrectomy in 150 patients with T2DM.1 Patients were included if they were ages 20 to 60, had a BMI of 27 to 43, and had an A1C > 7%. Patients with a history of bariatric surgery, complex abdominal surgery, or uncontrolled medical or psychiatric disorders were excluded.

Patients were randomly placed in a 1:1:1 fashion into 3 groups: IMT (as defined by the ADA) only, IMT and gastric bypass, or IMT and sleeve gastrectomy. The primary outcome was the number of patients with an A1C ≤ 6%. Secondary outcomes included weight loss, glucose control, lipid levels, blood pressure, medication use, renal function, adverse effects, ophthalmologic outcomes, and quality of life.

Continue to: Of the 150 patients...

 

 

Of the 150 patients, 1 died during the follow-up period, leaving 149. Of these, 134 completed the 5-year follow-up. Eight patients in the IMT group and 1 patient in the sleeve gastrectomy group never initiated assigned treatment, and 6 patients were lost to follow-up. One patient from the IMT group and 1 patient from the sleeve gastrectomy group crossed over to the gastric bypass group.

Results. More patients in the bariatric surgery and sleeve gastrectomy groups achieved an A1C of ≤ 6% than in the IMT group (14 of 49 gastric bypass patients, 11 of 47 sleeve gastrectomy patients, and 2 of 38 IMT patients). Compared with those in the IMT group, the patients in the 2 surgery groups showed greater reductions from baseline in body weight and triglyceride levels and greater increases from baseline in HDL cholesterol levels; they also required less antidiabetes medication for glycemic control (see Table).1

Secondary Outcomes After 5 Years

WHAT’S NEW?

Big benefits, minimal adverse effects

Prior studies evaluating the effect of gastric bypass surgery on diabetes were observational or had a shorter follow-up duration. This study demonstrates that bariatric surgery plus IMT has long-term benefits with minimal adverse events, compared with IMT alone.1,5 Additionally, this study supports recommendations for bariatric surgery as treatment for T2DM in patients with a BMI ≥ 27, which is below the starting BMI (35) recommended by the ADA.1,4

 

CAVEATS

Surgery is not without risks

The risk for surgical complications—eg, gastrointestinal bleeding, severe hypoglycemia requiring intervention, and ketoacidosis—in this patient population is significant.1 Other potential complications include gastrointestinal leak, stroke, and infection.1 Additionally, long-term complications from bariatric surgery are emerging and include choledocholithiasis, intestinal obstruction, and esophageal pathology.7 Extensive patient counseling is necessary to ensure that patients make an informed decision regarding surgery.

This study utilized surrogate markers (A1C, lipid levels, and body weight) as disease-oriented outcome measures. Patient-oriented outcomes, such as morbidity and mortality, were not explored in this study.

Continue to: Due to the small sample size...

 

 

Due to the small sample size of the study, it is unclear if the outcomes of the 2 surgery groups were significantly different. Patients who underwent gastric bypass surgery had more weight loss and used less diabetes medication at the end of follow-up, compared with patients who underwent sleeve gastrectomy. More information is needed to determine which gastric surgery is preferable for the treatment of T2DM while minimizing adverse effects. However, both of the procedures had outcomes superior to those of IMT, and selection of a particular type of surgery should be a joint decision between the patient and provider.

CHALLENGES TO IMPLEMENTATION

Access and cost may be barriers

The major barriers to implementation are access to, and cost of, bariatric surgery.

ACKNOWLEDGEMENT

The PURLs Surveillance System was supported in part by Grant Number UL1RR024999 from the National Center For Research Resources, a Clinical Translational Science Award to the University of Chicago. The content is solely the responsibility of the authors and does not necessarily represent the official views of the National Center For Research Resources or the National Institutes of Health.

Copyright © 2019. The Family Physicians Inquiries Network. All rights reserved.

Reprinted with permission from the Family Physicians Inquiries Network and The Journal of Family Practice (2019;68[2]:102-104).

References

1. Schauer PR, Bhatt DL, Kirwan JP, et al; STAMPEDE Investigators. Bariatric surgery versus intensive medical therapy for diabetes—5-year outcomes. N Engl J Med. 2017;376:641-651.
2. American Diabetes Association. Obesity management for the treatment of type 2 diabetes: standards of medical care in diabetes—2019. Diabetes Care. 2019;42(suppl 1):S81-S89.
3. CDC. National Diabetes Statistics Report, 2017. Atlanta, GA: CDC, US Department of Health and Human Services; 2017. www.cdc.gov/diabetes/pdfs/data/statistics/national-diabetes-statistics-report.pdf. Accessed June 27, 2019.
4. Rubino F, Nathan DM, Eckel RH, et al. Metabolic surgery in the treatment algorithm for type 2 diabetes: a joint statement by international diabetes organizations. Diabetes Care. 2016;39:861-877.
5. Mingrone G, Panunzi S, De Gaetano A, et al. Bariatric surgery versus conventional medical therapy for type 2 diabetes. N Engl J Med. 2012;366:1577-1585.
6. Lee WJ, Chong K, Ser KH, et al. Gastric bypass vs sleeve gastrectomy for type 2 diabetes mellitus: a randomized controlled trial. Arch Surg. 2011; 146:143-148.
7. Schulman AR, Thompson CC. Complications of bariatric surgery: what you can expect to see in your GI practice. Am J Gastroenterol. 2017;112:1640-1655.

Article PDF
Author and Disclosure Information

Kortnee Y. Roberson and Emily White Van Gompel are in the Department of Family Medicine at the University of Chicago. Jennie B. Jarrett is with the University of Illinois at Chicago.

Issue
Clinician Reviews - 29(7)
Publications
Topics
Page Number
1e-3e
Sections
Author and Disclosure Information

Kortnee Y. Roberson and Emily White Van Gompel are in the Department of Family Medicine at the University of Chicago. Jennie B. Jarrett is with the University of Illinois at Chicago.

Author and Disclosure Information

Kortnee Y. Roberson and Emily White Van Gompel are in the Department of Family Medicine at the University of Chicago. Jennie B. Jarrett is with the University of Illinois at Chicago.

Article PDF
Article PDF

Practice Changer

A 46-year-old woman presents with a BMI of 28, a 4-year history of type 2 diabetes mellitus (T2DM), and an A1C of 9.8%. The patient is currently being treated with intensive medical therapy (IMT), including metformin 2000 mg/d, sitagliptin 100 mg/d, and insulin glargine 12 U/d, with minimal change in A1C. Should you recommend bariatric surgery?

One in 11 Americans has diabetes, and at least 95% of those have T2DM.2,3 The treatment of T2DM is generally multimodal to target the various mechanisms that cause hyperglycemia. Strategies may include making lifestyle modifications, decreasing insulin resistance, increasing insulin secretion, replacing insulin, and targeting incretin-hormonal pathways.

The American Diabetes Association (ADA) recommends diet, exercise, and behavioral modifications as firstline therapy for diabetes management, but these methods are often inadequate.2 In addition to various pharmacotherapeutic strategies for some populations with T2DM, the ADA recommends bariatric surgery for those with a BMI ≥ 35 and uncontrolled hyper­glycemia.2,4

However, this recommendation is based only on short-term studies. For example, in a single-center, nonblinded RCT of 60 patients with a BMI ≥ 35, the average baseline A1C levels of 8.65 ± 1.45% were reduced to 7.7 ± 0.6% in the IMT group and to 6.4 ± 1.4% in the gastric-bypass group at 2 years.5 In another study, a randomized double-blind trial involving 60 moderately obese patients (BMI, 25-35), gastric bypass yielded better outcomes than sleeve gastrectomy: 93% of patients in the former group and 47% of those in the latter group achieved remission of T2DM over a 12-month period.6

The current study by Schauer et al examined the long-term outcomes of IMT alone vs bariatric surgery with IMT for the treatment of T2DM in patients who are overweight or obese.1

STUDY SUMMARY

5-year follow-up: surgery + IMT works

This study was a 5-year follow-up of a nonblinded, single-center RCT comparing IMT alone to IMT with Roux-en-Y gastric bypass or sleeve gastrectomy in 150 patients with T2DM.1 Patients were included if they were ages 20 to 60, had a BMI of 27 to 43, and had an A1C > 7%. Patients with a history of bariatric surgery, complex abdominal surgery, or uncontrolled medical or psychiatric disorders were excluded.

Patients were randomly placed in a 1:1:1 fashion into 3 groups: IMT (as defined by the ADA) only, IMT and gastric bypass, or IMT and sleeve gastrectomy. The primary outcome was the number of patients with an A1C ≤ 6%. Secondary outcomes included weight loss, glucose control, lipid levels, blood pressure, medication use, renal function, adverse effects, ophthalmologic outcomes, and quality of life.

Continue to: Of the 150 patients...

 

 

Of the 150 patients, 1 died during the follow-up period, leaving 149. Of these, 134 completed the 5-year follow-up. Eight patients in the IMT group and 1 patient in the sleeve gastrectomy group never initiated assigned treatment, and 6 patients were lost to follow-up. One patient from the IMT group and 1 patient from the sleeve gastrectomy group crossed over to the gastric bypass group.

Results. More patients in the bariatric surgery and sleeve gastrectomy groups achieved an A1C of ≤ 6% than in the IMT group (14 of 49 gastric bypass patients, 11 of 47 sleeve gastrectomy patients, and 2 of 38 IMT patients). Compared with those in the IMT group, the patients in the 2 surgery groups showed greater reductions from baseline in body weight and triglyceride levels and greater increases from baseline in HDL cholesterol levels; they also required less antidiabetes medication for glycemic control (see Table).1

Secondary Outcomes After 5 Years

WHAT’S NEW?

Big benefits, minimal adverse effects

Prior studies evaluating the effect of gastric bypass surgery on diabetes were observational or had a shorter follow-up duration. This study demonstrates that bariatric surgery plus IMT has long-term benefits with minimal adverse events, compared with IMT alone.1,5 Additionally, this study supports recommendations for bariatric surgery as treatment for T2DM in patients with a BMI ≥ 27, which is below the starting BMI (35) recommended by the ADA.1,4

 

CAVEATS

Surgery is not without risks

The risk for surgical complications—eg, gastrointestinal bleeding, severe hypoglycemia requiring intervention, and ketoacidosis—in this patient population is significant.1 Other potential complications include gastrointestinal leak, stroke, and infection.1 Additionally, long-term complications from bariatric surgery are emerging and include choledocholithiasis, intestinal obstruction, and esophageal pathology.7 Extensive patient counseling is necessary to ensure that patients make an informed decision regarding surgery.

This study utilized surrogate markers (A1C, lipid levels, and body weight) as disease-oriented outcome measures. Patient-oriented outcomes, such as morbidity and mortality, were not explored in this study.

Continue to: Due to the small sample size...

 

 

Due to the small sample size of the study, it is unclear if the outcomes of the 2 surgery groups were significantly different. Patients who underwent gastric bypass surgery had more weight loss and used less diabetes medication at the end of follow-up, compared with patients who underwent sleeve gastrectomy. More information is needed to determine which gastric surgery is preferable for the treatment of T2DM while minimizing adverse effects. However, both of the procedures had outcomes superior to those of IMT, and selection of a particular type of surgery should be a joint decision between the patient and provider.

CHALLENGES TO IMPLEMENTATION

Access and cost may be barriers

The major barriers to implementation are access to, and cost of, bariatric surgery.

ACKNOWLEDGEMENT

The PURLs Surveillance System was supported in part by Grant Number UL1RR024999 from the National Center For Research Resources, a Clinical Translational Science Award to the University of Chicago. The content is solely the responsibility of the authors and does not necessarily represent the official views of the National Center For Research Resources or the National Institutes of Health.

Copyright © 2019. The Family Physicians Inquiries Network. All rights reserved.

Reprinted with permission from the Family Physicians Inquiries Network and The Journal of Family Practice (2019;68[2]:102-104).

Practice Changer

A 46-year-old woman presents with a BMI of 28, a 4-year history of type 2 diabetes mellitus (T2DM), and an A1C of 9.8%. The patient is currently being treated with intensive medical therapy (IMT), including metformin 2000 mg/d, sitagliptin 100 mg/d, and insulin glargine 12 U/d, with minimal change in A1C. Should you recommend bariatric surgery?

One in 11 Americans has diabetes, and at least 95% of those have T2DM.2,3 The treatment of T2DM is generally multimodal to target the various mechanisms that cause hyperglycemia. Strategies may include making lifestyle modifications, decreasing insulin resistance, increasing insulin secretion, replacing insulin, and targeting incretin-hormonal pathways.

The American Diabetes Association (ADA) recommends diet, exercise, and behavioral modifications as firstline therapy for diabetes management, but these methods are often inadequate.2 In addition to various pharmacotherapeutic strategies for some populations with T2DM, the ADA recommends bariatric surgery for those with a BMI ≥ 35 and uncontrolled hyper­glycemia.2,4

However, this recommendation is based only on short-term studies. For example, in a single-center, nonblinded RCT of 60 patients with a BMI ≥ 35, the average baseline A1C levels of 8.65 ± 1.45% were reduced to 7.7 ± 0.6% in the IMT group and to 6.4 ± 1.4% in the gastric-bypass group at 2 years.5 In another study, a randomized double-blind trial involving 60 moderately obese patients (BMI, 25-35), gastric bypass yielded better outcomes than sleeve gastrectomy: 93% of patients in the former group and 47% of those in the latter group achieved remission of T2DM over a 12-month period.6

The current study by Schauer et al examined the long-term outcomes of IMT alone vs bariatric surgery with IMT for the treatment of T2DM in patients who are overweight or obese.1

STUDY SUMMARY

5-year follow-up: surgery + IMT works

This study was a 5-year follow-up of a nonblinded, single-center RCT comparing IMT alone to IMT with Roux-en-Y gastric bypass or sleeve gastrectomy in 150 patients with T2DM.1 Patients were included if they were ages 20 to 60, had a BMI of 27 to 43, and had an A1C > 7%. Patients with a history of bariatric surgery, complex abdominal surgery, or uncontrolled medical or psychiatric disorders were excluded.

Patients were randomly placed in a 1:1:1 fashion into 3 groups: IMT (as defined by the ADA) only, IMT and gastric bypass, or IMT and sleeve gastrectomy. The primary outcome was the number of patients with an A1C ≤ 6%. Secondary outcomes included weight loss, glucose control, lipid levels, blood pressure, medication use, renal function, adverse effects, ophthalmologic outcomes, and quality of life.

Continue to: Of the 150 patients...

 

 

Of the 150 patients, 1 died during the follow-up period, leaving 149. Of these, 134 completed the 5-year follow-up. Eight patients in the IMT group and 1 patient in the sleeve gastrectomy group never initiated assigned treatment, and 6 patients were lost to follow-up. One patient from the IMT group and 1 patient from the sleeve gastrectomy group crossed over to the gastric bypass group.

Results. More patients in the bariatric surgery and sleeve gastrectomy groups achieved an A1C of ≤ 6% than in the IMT group (14 of 49 gastric bypass patients, 11 of 47 sleeve gastrectomy patients, and 2 of 38 IMT patients). Compared with those in the IMT group, the patients in the 2 surgery groups showed greater reductions from baseline in body weight and triglyceride levels and greater increases from baseline in HDL cholesterol levels; they also required less antidiabetes medication for glycemic control (see Table).1

Secondary Outcomes After 5 Years

WHAT’S NEW?

Big benefits, minimal adverse effects

Prior studies evaluating the effect of gastric bypass surgery on diabetes were observational or had a shorter follow-up duration. This study demonstrates that bariatric surgery plus IMT has long-term benefits with minimal adverse events, compared with IMT alone.1,5 Additionally, this study supports recommendations for bariatric surgery as treatment for T2DM in patients with a BMI ≥ 27, which is below the starting BMI (35) recommended by the ADA.1,4

 

CAVEATS

Surgery is not without risks

The risk for surgical complications—eg, gastrointestinal bleeding, severe hypoglycemia requiring intervention, and ketoacidosis—in this patient population is significant.1 Other potential complications include gastrointestinal leak, stroke, and infection.1 Additionally, long-term complications from bariatric surgery are emerging and include choledocholithiasis, intestinal obstruction, and esophageal pathology.7 Extensive patient counseling is necessary to ensure that patients make an informed decision regarding surgery.

This study utilized surrogate markers (A1C, lipid levels, and body weight) as disease-oriented outcome measures. Patient-oriented outcomes, such as morbidity and mortality, were not explored in this study.

Continue to: Due to the small sample size...

 

 

Due to the small sample size of the study, it is unclear if the outcomes of the 2 surgery groups were significantly different. Patients who underwent gastric bypass surgery had more weight loss and used less diabetes medication at the end of follow-up, compared with patients who underwent sleeve gastrectomy. More information is needed to determine which gastric surgery is preferable for the treatment of T2DM while minimizing adverse effects. However, both of the procedures had outcomes superior to those of IMT, and selection of a particular type of surgery should be a joint decision between the patient and provider.

CHALLENGES TO IMPLEMENTATION

Access and cost may be barriers

The major barriers to implementation are access to, and cost of, bariatric surgery.

ACKNOWLEDGEMENT

The PURLs Surveillance System was supported in part by Grant Number UL1RR024999 from the National Center For Research Resources, a Clinical Translational Science Award to the University of Chicago. The content is solely the responsibility of the authors and does not necessarily represent the official views of the National Center For Research Resources or the National Institutes of Health.

Copyright © 2019. The Family Physicians Inquiries Network. All rights reserved.

Reprinted with permission from the Family Physicians Inquiries Network and The Journal of Family Practice (2019;68[2]:102-104).

References

1. Schauer PR, Bhatt DL, Kirwan JP, et al; STAMPEDE Investigators. Bariatric surgery versus intensive medical therapy for diabetes—5-year outcomes. N Engl J Med. 2017;376:641-651.
2. American Diabetes Association. Obesity management for the treatment of type 2 diabetes: standards of medical care in diabetes—2019. Diabetes Care. 2019;42(suppl 1):S81-S89.
3. CDC. National Diabetes Statistics Report, 2017. Atlanta, GA: CDC, US Department of Health and Human Services; 2017. www.cdc.gov/diabetes/pdfs/data/statistics/national-diabetes-statistics-report.pdf. Accessed June 27, 2019.
4. Rubino F, Nathan DM, Eckel RH, et al. Metabolic surgery in the treatment algorithm for type 2 diabetes: a joint statement by international diabetes organizations. Diabetes Care. 2016;39:861-877.
5. Mingrone G, Panunzi S, De Gaetano A, et al. Bariatric surgery versus conventional medical therapy for type 2 diabetes. N Engl J Med. 2012;366:1577-1585.
6. Lee WJ, Chong K, Ser KH, et al. Gastric bypass vs sleeve gastrectomy for type 2 diabetes mellitus: a randomized controlled trial. Arch Surg. 2011; 146:143-148.
7. Schulman AR, Thompson CC. Complications of bariatric surgery: what you can expect to see in your GI practice. Am J Gastroenterol. 2017;112:1640-1655.

References

1. Schauer PR, Bhatt DL, Kirwan JP, et al; STAMPEDE Investigators. Bariatric surgery versus intensive medical therapy for diabetes—5-year outcomes. N Engl J Med. 2017;376:641-651.
2. American Diabetes Association. Obesity management for the treatment of type 2 diabetes: standards of medical care in diabetes—2019. Diabetes Care. 2019;42(suppl 1):S81-S89.
3. CDC. National Diabetes Statistics Report, 2017. Atlanta, GA: CDC, US Department of Health and Human Services; 2017. www.cdc.gov/diabetes/pdfs/data/statistics/national-diabetes-statistics-report.pdf. Accessed June 27, 2019.
4. Rubino F, Nathan DM, Eckel RH, et al. Metabolic surgery in the treatment algorithm for type 2 diabetes: a joint statement by international diabetes organizations. Diabetes Care. 2016;39:861-877.
5. Mingrone G, Panunzi S, De Gaetano A, et al. Bariatric surgery versus conventional medical therapy for type 2 diabetes. N Engl J Med. 2012;366:1577-1585.
6. Lee WJ, Chong K, Ser KH, et al. Gastric bypass vs sleeve gastrectomy for type 2 diabetes mellitus: a randomized controlled trial. Arch Surg. 2011; 146:143-148.
7. Schulman AR, Thompson CC. Complications of bariatric surgery: what you can expect to see in your GI practice. Am J Gastroenterol. 2017;112:1640-1655.

Issue
Clinician Reviews - 29(7)
Issue
Clinician Reviews - 29(7)
Page Number
1e-3e
Page Number
1e-3e
Publications
Publications
Topics
Article Type
Display Headline
Bariatric Surgery + Medical Therapy: Effective Tx for T2DM?
Display Headline
Bariatric Surgery + Medical Therapy: Effective Tx for T2DM?
Sections
Disallow All Ads
Content Gating
No Gating (article Unlocked/Free)
Alternative CME
Disqus Comments
Default
Use ProPublica
Hide sidebar & use full width
render the right sidebar.
Article PDF Media

Bariatric surgery + medical therapy: Effective Tx for T2DM?

Article Type
Changed
Tue, 05/03/2022 - 15:15
Display Headline
Bariatric surgery + medical therapy: Effective Tx for T2DM?

ILLUSTRATIVE CASE

A 46-year-old woman presents with a body mass index (BMI) of 28 kg/m2, a 4-year history of type 2 diabetes mellitus (T2DM), and a glycated hemoglobin (HgbA1c) of 9.8%. The patient is currently being treated with intensive medical therapy (IMT), including metformin 2000 mg/d, sitagliptin 100 mg/d, and insulin glargine 12 units/d, with minimal change in HgbA1c. Should you recommend bariatric surgery as an option for the treatment of diabetes?

One in 11 Americans has diabetes and at least 95% of those have type 2.2,3 The treatment of T2DM is generally multimodal in order to target the various mechanisms that cause hyperglycemia. Treatment strategies may include lifestyle modifications, decreasing insulin resistance, increasing secretion of insulin, insulin replacement, and targeting incretin-hormonal pathways.

The American Diabetes Association (ADA) currently recommends diet, exercise, and behavioral modifications as first-line therapy for the management of diabetes,2 but these by themselves are often inadequate. In addition to various pharmacotherapeutic strategies for other populations with T2DM (see the PURL, “How do these 3 diabetes agents compare in reducing mortality?”), the ADA recommends bariatric surgery for the treatment of patients with T2DM, a BMI ≥35 kg/m2, and uncontrolled hyperglycemia.2,4 However, this recommendation from the ADA supporting bariatric surgery is based only on short-term studies.

For example, one single-center nonblinded randomized controlled trial (RCT) involving 60 patients with a BMI ≥35 kg/m2 found reductions in HgbA1C levels from the average baseline of 8.65±1.45% to 7.7±0.6% in the IMT group and to 6.4±1.4% in the gastric-bypass group at 2 years.5 In another study, a randomized double-blind trial involving 60 moderately obese patients (BMI, 25-35 kg/m2), gastric bypass had better outcomes than sleeve gastrectomy, with 93% of patients in the gastric bypass group achieving remission of T2DM vs 47% of patients in the sleeve gastrectomy group (P=.02) over a 12-month period.6

The current study sought to examine the long-term outcomes of IMT alone vs bariatric surgery with IMT for the treatment of T2DM in patients who are overweight or obese.1

STUDY SUMMARY

5-year follow-up shows surgery + intensive medical therapy works

This study by Schauer et al was a 5-year follow-up of a nonblinded, single-center RCT comparing IMT alone to IMT with Roux-en-Y gastric bypass or sleeve gastrectomy in 150 patients with T2DM.1 Patients were included if they were 20 to 60 years of age, had a BMI of 27 to 43 kg/m2, and had an HgbA1C >7%. Patients with previous bariatric surgery, complex abdominal surgery, or uncontrolled medical or psychiatric disorders were excluded.

Each patient was randomly placed in a 1:1:1 fashion into 3 groups: IMT only, IMT and gastric bypass, or IMT and sleeve gastrectomy. All patients underwent IMT as defined by the ADA. The primary outcome was the number of patients with an HgbA1c ≤6%. Secondary outcomes included weight loss, glucose control, lipid levels, blood pressure, medication use, renal function, adverse effects, ophthalmologic outcomes, and quality of life.

Continue to: Of the 150 patients...

 

 

Of the 150 patients, 1 died during the follow-up period leaving 149; 134 completed the 5-year follow-up; 8 patients in the IMT group and 1 patient in the sleeve gastrectomy group never initiated assigned treatment; an additional 6 patients were lost to follow-up. One patient from the IMT group and 1 patient from the sleeve gastrectomy group crossed over to the gastric bypass group.

Results. More patients in the bariatric surgery and sleeve gastrectomy groups achieved an HgbA1c of ≤6% compared with the IMT group (14 of 49 gastric bypass patients vs 2 of 38 IMT patients; P=.01; 11 of 47 sleeve gastrectomy patients vs 2 of 38 IMT patients; P=.03). Compared with those in the IMT group, the patients in the bariatric surgery and sleeve gastrectomy groups showed greater reductions from baseline in body weight and triglyceride levels, and greater increases from baseline in high-density lipoprotein (HDL) cholesterol levels; they also required less diabetic medication for glycemic control (see TABLE 11). However, when data were imputed for the intention-to-treat analysis, P-values were P=0.08 for gastric bypass and P=0.17 for sleeve gastrectomy compared with the IMT group for lowering HgbA1c.

Secondary outcomes at the end of 5 years

WHAT’S NEW?

Adding surgery has big benefits with minimal adverse effects

Prior studies that evaluated the effect of gastric bypass surgery on diabetes were observational or had a shorter follow-up duration. This study demonstrates bariatric surgery plus IMT has long-term benefits with minimal adverse events compared with IMT alone.1,5 Additionally, this study supports recommendations for bariatric surgery as treatment for T2DM for patients with a BMI ≥27 kg/m2, which is below the starting BMI (35 kg/m2) recommended by the ADA.1,4

 

CAVEATS

Surgery is not without risks

The risk for surgical complications, such as gastrointestinal bleeding, severe hypoglycemia requiring intervention, and ketoacidosis, in this patient population is significant.1 Complications can include gastrointestinal leak, stroke, and infection.1 Additionally, long-term complications from bariatric surgery are emerging and include choledocholithiasis, intestinal obstruction, and esophageal pathology.7 Extensive patient counseling regarding the possible complications is necessary to ensure that patients make an informed decision regarding surgery.

This study supports recommedations for bariatric surgery as treatment for T2DM for patients with a body mass index ≥27 kg/m2, which is below the starting BMI recommended by the ADA.

This study utilized surrogate markers (A1c, lipid levels, and body weight) as disease-oriented outcome measures. Patient-oriented outcomes, such as morbidity and mortality, were not explored in this study.

Continue to: Due to the small sample size of the study...

 

 

Due to the small sample size of the study, it is unclear if the outcomes of the 2 surgery groups were significantly different. Patients who received gastric bypass surgery had more weight loss and used less diabetes medication at the end of follow-up compared with the patients who received sleeve gastrectomy. More information is needed to determine which gastric surgery is preferable for the treatment of T2DM while minimizing adverse effects. However, both of the procedures had outcomes superior to that with IMT, and selection of a particular type of surgery should be a joint decision between the patient and provider.

CHALLENGES TO IMPLEMENTATION

Access and cost may be barriers

The major barriers to implementation are access to, and the cost of, bariatric surgery.

ACKNOWLEDGEMENT

The PURLs Surveillance System was supported in part by Grant Number UL1RR024999 from the National Center For Research Resources, a Clinical Translational Science Award to the University of Chicago. The content is solely the responsibility of the authors and does not necessarily represent the official views of the National Center For Research Resources or the National Institutes of Health.

Files
References

1. Schauer PR, Bhatt DL, Kirwan JP, et al; STAMPEDE Investigators. Bariatric surgery versus intensive medical therapy for diabetes—5-year outcomes. N Engl J Med. 2017;376:641-651.

2. American Diabetes Asssociation. Obesity management for the treatment of type 2 diabetes: standards of medical care in diabetes—2019. Diabetes Care. 2019;42 (suppl 1):S81-S89.


3. Centers for Disease Control and Prevention. National Diabetes Statistics Report, 2017. Atlanta, GA: Centers for Disease Control and Prevention, US Department of Health and Human Services; 2017. www.cdc.gov/diabetes/pdfs/data/statistics/national-diabetes-statistics-report.pdf. Accessed March 1, 2019.

4. Rubino F, Nathan DM, Eckel RH, et al. Metabolic surgery in the treatment algorithm for type 2 diabetes: a joint statement by international diabetes organizations. Diabetes Care. 2016;39:861-877.

5. Mingrone G, Panunzi S, De Gaetano A, et al. Bariatric surgery versus conventional medical therapy for type 2 diabetes. N Engl J Med. 2012;366:1577-1585.

6. Lee WJ, Chong K, Ser KH, et al. Gastric bypass vs sleeve gastrectomy for type 2 diabetes mellitus: a randomized controlled trial. Arch Surg. 2011;146:143-148.

7. Schulman AR, Thompson CC. Complications of bariatric surgery: what you can expect to see in your GI practice. Am J Gastroenterol. 2017;112:1640-1655.

Article PDF
Author and Disclosure Information

University of Chicago, Department of Family Medicine (Drs. Roberson and Van Gompel); University of Illinois at Chicago (Dr. Jarrett)

DEPUTY EDITOR
Shailendra Prasad, MBBS, MPH

University of Minnesota, Department of Family Medicine and Community Health, Minneapolis

Issue
The Journal of Family Practice - 68(2)
Publications
Topics
Page Number
102-104
Sections
Files
Files
Author and Disclosure Information

University of Chicago, Department of Family Medicine (Drs. Roberson and Van Gompel); University of Illinois at Chicago (Dr. Jarrett)

DEPUTY EDITOR
Shailendra Prasad, MBBS, MPH

University of Minnesota, Department of Family Medicine and Community Health, Minneapolis

Author and Disclosure Information

University of Chicago, Department of Family Medicine (Drs. Roberson and Van Gompel); University of Illinois at Chicago (Dr. Jarrett)

DEPUTY EDITOR
Shailendra Prasad, MBBS, MPH

University of Minnesota, Department of Family Medicine and Community Health, Minneapolis

Article PDF
Article PDF

ILLUSTRATIVE CASE

A 46-year-old woman presents with a body mass index (BMI) of 28 kg/m2, a 4-year history of type 2 diabetes mellitus (T2DM), and a glycated hemoglobin (HgbA1c) of 9.8%. The patient is currently being treated with intensive medical therapy (IMT), including metformin 2000 mg/d, sitagliptin 100 mg/d, and insulin glargine 12 units/d, with minimal change in HgbA1c. Should you recommend bariatric surgery as an option for the treatment of diabetes?

One in 11 Americans has diabetes and at least 95% of those have type 2.2,3 The treatment of T2DM is generally multimodal in order to target the various mechanisms that cause hyperglycemia. Treatment strategies may include lifestyle modifications, decreasing insulin resistance, increasing secretion of insulin, insulin replacement, and targeting incretin-hormonal pathways.

The American Diabetes Association (ADA) currently recommends diet, exercise, and behavioral modifications as first-line therapy for the management of diabetes,2 but these by themselves are often inadequate. In addition to various pharmacotherapeutic strategies for other populations with T2DM (see the PURL, “How do these 3 diabetes agents compare in reducing mortality?”), the ADA recommends bariatric surgery for the treatment of patients with T2DM, a BMI ≥35 kg/m2, and uncontrolled hyperglycemia.2,4 However, this recommendation from the ADA supporting bariatric surgery is based only on short-term studies.

For example, one single-center nonblinded randomized controlled trial (RCT) involving 60 patients with a BMI ≥35 kg/m2 found reductions in HgbA1C levels from the average baseline of 8.65±1.45% to 7.7±0.6% in the IMT group and to 6.4±1.4% in the gastric-bypass group at 2 years.5 In another study, a randomized double-blind trial involving 60 moderately obese patients (BMI, 25-35 kg/m2), gastric bypass had better outcomes than sleeve gastrectomy, with 93% of patients in the gastric bypass group achieving remission of T2DM vs 47% of patients in the sleeve gastrectomy group (P=.02) over a 12-month period.6

The current study sought to examine the long-term outcomes of IMT alone vs bariatric surgery with IMT for the treatment of T2DM in patients who are overweight or obese.1

STUDY SUMMARY

5-year follow-up shows surgery + intensive medical therapy works

This study by Schauer et al was a 5-year follow-up of a nonblinded, single-center RCT comparing IMT alone to IMT with Roux-en-Y gastric bypass or sleeve gastrectomy in 150 patients with T2DM.1 Patients were included if they were 20 to 60 years of age, had a BMI of 27 to 43 kg/m2, and had an HgbA1C >7%. Patients with previous bariatric surgery, complex abdominal surgery, or uncontrolled medical or psychiatric disorders were excluded.

Each patient was randomly placed in a 1:1:1 fashion into 3 groups: IMT only, IMT and gastric bypass, or IMT and sleeve gastrectomy. All patients underwent IMT as defined by the ADA. The primary outcome was the number of patients with an HgbA1c ≤6%. Secondary outcomes included weight loss, glucose control, lipid levels, blood pressure, medication use, renal function, adverse effects, ophthalmologic outcomes, and quality of life.

Continue to: Of the 150 patients...

 

 

Of the 150 patients, 1 died during the follow-up period leaving 149; 134 completed the 5-year follow-up; 8 patients in the IMT group and 1 patient in the sleeve gastrectomy group never initiated assigned treatment; an additional 6 patients were lost to follow-up. One patient from the IMT group and 1 patient from the sleeve gastrectomy group crossed over to the gastric bypass group.

Results. More patients in the bariatric surgery and sleeve gastrectomy groups achieved an HgbA1c of ≤6% compared with the IMT group (14 of 49 gastric bypass patients vs 2 of 38 IMT patients; P=.01; 11 of 47 sleeve gastrectomy patients vs 2 of 38 IMT patients; P=.03). Compared with those in the IMT group, the patients in the bariatric surgery and sleeve gastrectomy groups showed greater reductions from baseline in body weight and triglyceride levels, and greater increases from baseline in high-density lipoprotein (HDL) cholesterol levels; they also required less diabetic medication for glycemic control (see TABLE 11). However, when data were imputed for the intention-to-treat analysis, P-values were P=0.08 for gastric bypass and P=0.17 for sleeve gastrectomy compared with the IMT group for lowering HgbA1c.

Secondary outcomes at the end of 5 years

WHAT’S NEW?

Adding surgery has big benefits with minimal adverse effects

Prior studies that evaluated the effect of gastric bypass surgery on diabetes were observational or had a shorter follow-up duration. This study demonstrates bariatric surgery plus IMT has long-term benefits with minimal adverse events compared with IMT alone.1,5 Additionally, this study supports recommendations for bariatric surgery as treatment for T2DM for patients with a BMI ≥27 kg/m2, which is below the starting BMI (35 kg/m2) recommended by the ADA.1,4

 

CAVEATS

Surgery is not without risks

The risk for surgical complications, such as gastrointestinal bleeding, severe hypoglycemia requiring intervention, and ketoacidosis, in this patient population is significant.1 Complications can include gastrointestinal leak, stroke, and infection.1 Additionally, long-term complications from bariatric surgery are emerging and include choledocholithiasis, intestinal obstruction, and esophageal pathology.7 Extensive patient counseling regarding the possible complications is necessary to ensure that patients make an informed decision regarding surgery.

This study supports recommedations for bariatric surgery as treatment for T2DM for patients with a body mass index ≥27 kg/m2, which is below the starting BMI recommended by the ADA.

This study utilized surrogate markers (A1c, lipid levels, and body weight) as disease-oriented outcome measures. Patient-oriented outcomes, such as morbidity and mortality, were not explored in this study.

Continue to: Due to the small sample size of the study...

 

 

Due to the small sample size of the study, it is unclear if the outcomes of the 2 surgery groups were significantly different. Patients who received gastric bypass surgery had more weight loss and used less diabetes medication at the end of follow-up compared with the patients who received sleeve gastrectomy. More information is needed to determine which gastric surgery is preferable for the treatment of T2DM while minimizing adverse effects. However, both of the procedures had outcomes superior to that with IMT, and selection of a particular type of surgery should be a joint decision between the patient and provider.

CHALLENGES TO IMPLEMENTATION

Access and cost may be barriers

The major barriers to implementation are access to, and the cost of, bariatric surgery.

ACKNOWLEDGEMENT

The PURLs Surveillance System was supported in part by Grant Number UL1RR024999 from the National Center For Research Resources, a Clinical Translational Science Award to the University of Chicago. The content is solely the responsibility of the authors and does not necessarily represent the official views of the National Center For Research Resources or the National Institutes of Health.

ILLUSTRATIVE CASE

A 46-year-old woman presents with a body mass index (BMI) of 28 kg/m2, a 4-year history of type 2 diabetes mellitus (T2DM), and a glycated hemoglobin (HgbA1c) of 9.8%. The patient is currently being treated with intensive medical therapy (IMT), including metformin 2000 mg/d, sitagliptin 100 mg/d, and insulin glargine 12 units/d, with minimal change in HgbA1c. Should you recommend bariatric surgery as an option for the treatment of diabetes?

One in 11 Americans has diabetes and at least 95% of those have type 2.2,3 The treatment of T2DM is generally multimodal in order to target the various mechanisms that cause hyperglycemia. Treatment strategies may include lifestyle modifications, decreasing insulin resistance, increasing secretion of insulin, insulin replacement, and targeting incretin-hormonal pathways.

The American Diabetes Association (ADA) currently recommends diet, exercise, and behavioral modifications as first-line therapy for the management of diabetes,2 but these by themselves are often inadequate. In addition to various pharmacotherapeutic strategies for other populations with T2DM (see the PURL, “How do these 3 diabetes agents compare in reducing mortality?”), the ADA recommends bariatric surgery for the treatment of patients with T2DM, a BMI ≥35 kg/m2, and uncontrolled hyperglycemia.2,4 However, this recommendation from the ADA supporting bariatric surgery is based only on short-term studies.

For example, one single-center nonblinded randomized controlled trial (RCT) involving 60 patients with a BMI ≥35 kg/m2 found reductions in HgbA1C levels from the average baseline of 8.65±1.45% to 7.7±0.6% in the IMT group and to 6.4±1.4% in the gastric-bypass group at 2 years.5 In another study, a randomized double-blind trial involving 60 moderately obese patients (BMI, 25-35 kg/m2), gastric bypass had better outcomes than sleeve gastrectomy, with 93% of patients in the gastric bypass group achieving remission of T2DM vs 47% of patients in the sleeve gastrectomy group (P=.02) over a 12-month period.6

The current study sought to examine the long-term outcomes of IMT alone vs bariatric surgery with IMT for the treatment of T2DM in patients who are overweight or obese.1

STUDY SUMMARY

5-year follow-up shows surgery + intensive medical therapy works

This study by Schauer et al was a 5-year follow-up of a nonblinded, single-center RCT comparing IMT alone to IMT with Roux-en-Y gastric bypass or sleeve gastrectomy in 150 patients with T2DM.1 Patients were included if they were 20 to 60 years of age, had a BMI of 27 to 43 kg/m2, and had an HgbA1C >7%. Patients with previous bariatric surgery, complex abdominal surgery, or uncontrolled medical or psychiatric disorders were excluded.

Each patient was randomly placed in a 1:1:1 fashion into 3 groups: IMT only, IMT and gastric bypass, or IMT and sleeve gastrectomy. All patients underwent IMT as defined by the ADA. The primary outcome was the number of patients with an HgbA1c ≤6%. Secondary outcomes included weight loss, glucose control, lipid levels, blood pressure, medication use, renal function, adverse effects, ophthalmologic outcomes, and quality of life.

Continue to: Of the 150 patients...

 

 

Of the 150 patients, 1 died during the follow-up period leaving 149; 134 completed the 5-year follow-up; 8 patients in the IMT group and 1 patient in the sleeve gastrectomy group never initiated assigned treatment; an additional 6 patients were lost to follow-up. One patient from the IMT group and 1 patient from the sleeve gastrectomy group crossed over to the gastric bypass group.

Results. More patients in the bariatric surgery and sleeve gastrectomy groups achieved an HgbA1c of ≤6% compared with the IMT group (14 of 49 gastric bypass patients vs 2 of 38 IMT patients; P=.01; 11 of 47 sleeve gastrectomy patients vs 2 of 38 IMT patients; P=.03). Compared with those in the IMT group, the patients in the bariatric surgery and sleeve gastrectomy groups showed greater reductions from baseline in body weight and triglyceride levels, and greater increases from baseline in high-density lipoprotein (HDL) cholesterol levels; they also required less diabetic medication for glycemic control (see TABLE 11). However, when data were imputed for the intention-to-treat analysis, P-values were P=0.08 for gastric bypass and P=0.17 for sleeve gastrectomy compared with the IMT group for lowering HgbA1c.

Secondary outcomes at the end of 5 years

WHAT’S NEW?

Adding surgery has big benefits with minimal adverse effects

Prior studies that evaluated the effect of gastric bypass surgery on diabetes were observational or had a shorter follow-up duration. This study demonstrates bariatric surgery plus IMT has long-term benefits with minimal adverse events compared with IMT alone.1,5 Additionally, this study supports recommendations for bariatric surgery as treatment for T2DM for patients with a BMI ≥27 kg/m2, which is below the starting BMI (35 kg/m2) recommended by the ADA.1,4

 

CAVEATS

Surgery is not without risks

The risk for surgical complications, such as gastrointestinal bleeding, severe hypoglycemia requiring intervention, and ketoacidosis, in this patient population is significant.1 Complications can include gastrointestinal leak, stroke, and infection.1 Additionally, long-term complications from bariatric surgery are emerging and include choledocholithiasis, intestinal obstruction, and esophageal pathology.7 Extensive patient counseling regarding the possible complications is necessary to ensure that patients make an informed decision regarding surgery.

This study supports recommedations for bariatric surgery as treatment for T2DM for patients with a body mass index ≥27 kg/m2, which is below the starting BMI recommended by the ADA.

This study utilized surrogate markers (A1c, lipid levels, and body weight) as disease-oriented outcome measures. Patient-oriented outcomes, such as morbidity and mortality, were not explored in this study.

Continue to: Due to the small sample size of the study...

 

 

Due to the small sample size of the study, it is unclear if the outcomes of the 2 surgery groups were significantly different. Patients who received gastric bypass surgery had more weight loss and used less diabetes medication at the end of follow-up compared with the patients who received sleeve gastrectomy. More information is needed to determine which gastric surgery is preferable for the treatment of T2DM while minimizing adverse effects. However, both of the procedures had outcomes superior to that with IMT, and selection of a particular type of surgery should be a joint decision between the patient and provider.

CHALLENGES TO IMPLEMENTATION

Access and cost may be barriers

The major barriers to implementation are access to, and the cost of, bariatric surgery.

ACKNOWLEDGEMENT

The PURLs Surveillance System was supported in part by Grant Number UL1RR024999 from the National Center For Research Resources, a Clinical Translational Science Award to the University of Chicago. The content is solely the responsibility of the authors and does not necessarily represent the official views of the National Center For Research Resources or the National Institutes of Health.

References

1. Schauer PR, Bhatt DL, Kirwan JP, et al; STAMPEDE Investigators. Bariatric surgery versus intensive medical therapy for diabetes—5-year outcomes. N Engl J Med. 2017;376:641-651.

2. American Diabetes Asssociation. Obesity management for the treatment of type 2 diabetes: standards of medical care in diabetes—2019. Diabetes Care. 2019;42 (suppl 1):S81-S89.


3. Centers for Disease Control and Prevention. National Diabetes Statistics Report, 2017. Atlanta, GA: Centers for Disease Control and Prevention, US Department of Health and Human Services; 2017. www.cdc.gov/diabetes/pdfs/data/statistics/national-diabetes-statistics-report.pdf. Accessed March 1, 2019.

4. Rubino F, Nathan DM, Eckel RH, et al. Metabolic surgery in the treatment algorithm for type 2 diabetes: a joint statement by international diabetes organizations. Diabetes Care. 2016;39:861-877.

5. Mingrone G, Panunzi S, De Gaetano A, et al. Bariatric surgery versus conventional medical therapy for type 2 diabetes. N Engl J Med. 2012;366:1577-1585.

6. Lee WJ, Chong K, Ser KH, et al. Gastric bypass vs sleeve gastrectomy for type 2 diabetes mellitus: a randomized controlled trial. Arch Surg. 2011;146:143-148.

7. Schulman AR, Thompson CC. Complications of bariatric surgery: what you can expect to see in your GI practice. Am J Gastroenterol. 2017;112:1640-1655.

References

1. Schauer PR, Bhatt DL, Kirwan JP, et al; STAMPEDE Investigators. Bariatric surgery versus intensive medical therapy for diabetes—5-year outcomes. N Engl J Med. 2017;376:641-651.

2. American Diabetes Asssociation. Obesity management for the treatment of type 2 diabetes: standards of medical care in diabetes—2019. Diabetes Care. 2019;42 (suppl 1):S81-S89.


3. Centers for Disease Control and Prevention. National Diabetes Statistics Report, 2017. Atlanta, GA: Centers for Disease Control and Prevention, US Department of Health and Human Services; 2017. www.cdc.gov/diabetes/pdfs/data/statistics/national-diabetes-statistics-report.pdf. Accessed March 1, 2019.

4. Rubino F, Nathan DM, Eckel RH, et al. Metabolic surgery in the treatment algorithm for type 2 diabetes: a joint statement by international diabetes organizations. Diabetes Care. 2016;39:861-877.

5. Mingrone G, Panunzi S, De Gaetano A, et al. Bariatric surgery versus conventional medical therapy for type 2 diabetes. N Engl J Med. 2012;366:1577-1585.

6. Lee WJ, Chong K, Ser KH, et al. Gastric bypass vs sleeve gastrectomy for type 2 diabetes mellitus: a randomized controlled trial. Arch Surg. 2011;146:143-148.

7. Schulman AR, Thompson CC. Complications of bariatric surgery: what you can expect to see in your GI practice. Am J Gastroenterol. 2017;112:1640-1655.

Issue
The Journal of Family Practice - 68(2)
Issue
The Journal of Family Practice - 68(2)
Page Number
102-104
Page Number
102-104
Publications
Publications
Topics
Article Type
Display Headline
Bariatric surgery + medical therapy: Effective Tx for T2DM?
Display Headline
Bariatric surgery + medical therapy: Effective Tx for T2DM?
Sections
PURLs Copyright
Copyright © 2019. The Family Physicians Inquiries Network. All rights reserved.
Inside the Article

PRACTICE CHANGER

Consider bariatric surgery with medical therapy as a treatment option for adults with uncontrolled type 2 diabetes and a body mass index ≥27 kg/m2.1

STRENGTH OF RECOMMENDATION

B: Based on a nonblinded, single-center, randomized controlled trial.

Schauer PR, Bhatt DL, Kirwan JP, et al; STAMPEDE Investigators. Bariatric surgery versus intensive medical therapy for diabetes—5-year outcomes. N Engl J Med. 2017;376:641-651.

Disallow All Ads
Content Gating
No Gating (article Unlocked/Free)
Alternative CME
Disqus Comments
Default
Use ProPublica
Hide sidebar & use full width
render the right sidebar.
Article PDF Media
Media Files

Does Azithromycin Have a Role in Cesarean Sections?

Article Type
Changed
Wed, 08/01/2018 - 12:15
Display Headline
Does Azithromycin Have a Role in Cesarean Sections?

A 26-year-old G1P0 at 40w1d presents in spontaneous labor and is dilated to 4 cm. The patient reached complete cervical dilation after artificial rupture of membranes and oxytocin augmentation. After four hours of pushing, there has been minimal descent of the fetal vertex beyond +1 station with significant caput succedaneum. Her physician decides to proceed with cesarean delivery.2,3 What antibiotics should be administered prior to incision to reduce postoperative infection?

The CDC reports that nearly 1.3 million cesarean deliveries were performed in the United States in 2015, which represents about a third of all births.4 C-section is the most common major surgical procedure performed in this country and is associated with an infection rate five to 10 times that of vaginal delivery.5,6 Pregnancy-associated infection, particularly during delivery, is a significant risk and the fourth most common cause of maternal death in the United States.5

The current standard of care in cesarean delivery is antibiotic prophylaxis (often a first-generation cephalosporin) prior to skin incision.7 The majority of c-sections performed are nonelective, and of these, postoperative infections occur in 12% of women who receive standard prophylaxis.8,9 A small, single-center design trial suggested that azithromycin adjunctive therapy expands antibiotic coverage to Ureaplasma species, resulting in a lower risk for postoperative infection.10

This study evaluated the use of azithromycin adjunctive therapy, in addition to standard antibiotic prophylaxis, to reduce the risk for postoperative infections in women receiving nonelective c-sections.

STUDY SUMMARY

Reduced infections up to 6 weeks post–c-section

A multicenter, randomized, double-blind trial conducted in 14 hospitals in the United States evaluated the effect of a one-time dose of azithromycin (500 mg IV) on post­cesarean infections. Women with a singleton pregnancy of at least 24 weeks’ gestation were eligible for inclusion if they required nonelective cesarean delivery during labor or at least four hours after membrane rupture. Patients were excluded if they had a known azithromycin allergy, subsequent vaginal delivery, azithromycin use within the week prior to randomization, extensive hepatic or renal dysfunction, a known history of prolonged QT interval, or substantial electrolyte abnormalities. Patients were eligible even if they were receiving other antibiotics for a positive group B Streptococcus screening.1

All patients (N = 2,013) were treated with standard antibiotic prophylaxis (most often cefazolin), according to individual institution protocols. The women were randomized to receive either an azithromycin 500 mg/250 mL IV infusion (n = 1,019) or an identical placebo IV infusion (n = 994) within one hour of the procedure. The primary outcome was a composite endpoint of endometritis, wound infection, or other infections occurring up to six weeks after the c-section. Secondary outcomes included neonatal death, sepsis, and other neonatal and maternal complications.1

Patients in the placebo group had a higher rate of smoking during pregnancy; the researchers found no other significant differences.1

Results. The primary composite outcome occurred less frequently in the azithromycin group than in the placebo group (6.1% vs 12.0%; relative risk [RR], 0.51; number needed to treat [NNT], 17). When the researchers looked at the individual elements of the primary composite outcome, two had significant reductions versus placebo. Endometritis (3.8% vs 6.1%; RR, 0.62; NNT, 43) and wound infections (2.4% vs 6.6%; RR, 0.35; NNT, 24) occurred significantly less frequently, but there was no difference for other infections (0.3% vs 0.6%; RR, 0.49).

Serious maternal adverse events were also lower in the treatment group than in the control group (1.5% vs 2.9%; RR, 0.5; NNT, 71). There was no difference in composite secondary neonatal outcomes, including death and serious complications (14.3% vs 13.6%; RR, 1.05).1

 

 

WHAT’S NEW

Fewer infections without more adverse events

This study showed that adding azithromycin to standard antibiotic prophylaxis within one hour of a c-section reduces post–cesarean delivery infection rates without increasing the risk for maternal or neonatal adverse events.

CAVEATS

Caution with prolonged QT

While azithromycin was efficacious and well tolerated in the study, not every patient can take it. Patients with a previous drug reaction or allergy should avoid it, and experts advise prescribing it with caution for patients who have (or are at increased risk for) a prolonged QT interval, including those on other QT-prolonging medications.

Of note, women with scheduled c-sections and those with chorioamnionitis or another infection requiring postpartum antibiotics were excluded from this study. Thus, it is unknown if azithromycin use decreases complications in these patients.

CHALLENGES TO IMPLEMENTATION

Drug availability is key

Nonelective c-sections are performed based on many factors, including a nonreassuring fetal heart rate. In many of these cases, speed of cesarean delivery may mean the difference between positive and negative outcomes. Availability of azithromycin on labor and delivery floors for timely administration within one hour of the procedure is important.

Additionally, azithromycin has known QT prolongation risks.11 While the baseline QT interval is not known for many healthy young women, this should be considered when azithromycin is utilized in combination with other medications that may prolong the QT interval.

ACKNOWLEDGEMENT

The PURLs Surveillance System was supported in part by Grant Number UL1RR024999 from the National Center For Research Resources, a Clinical Translational Science Award to the University of Chicago. The content is solely the responsibility of the authors and does not necessarily represent the official views of the National Center For Research Resources or the National Institutes of Health.

Copyright © 2017. The Family Physicians Inquiries Network. All rights reserved.

Reprinted with permission from the Family Physicians Inquiries Network and The Journal of Family Practice (2017;66[12]:762-764).

References

1. Tita AT, Szychowski JM, Boggess K, et al. Adjunctive azithromycin prophylaxis for cesarean delivery. N Engl J Med. 2016; 375:1231-1241.
2. American College of Obstetricians and Gynecologists. Safe prevention of the primary cesarean delivery. Obstetric Care Consensus No. 1. Obstet Gynecol. 2014;123:693-711.
3. Rouse DJ, Weiner SJ, Bloom SL, et al. Second-stage labor duration in nulliparous women: relationship to maternal and perinatal outcomes. Am J Obstet Gynecol. 2009;201(4):357.e1-e7.
4. CDC National Center for Health Statistics. Births—Method of Delivery. www.cdc.gov/nchs/fastats/delivery.htm. Accessed December 1, 2017.
5. Perencevich EN, Sands KE, Cosgrove SE, et al. Health and economic impact of surgical site infections diagnosed after hospital discharge. Emerg Infect Dis. 2003;9:196-203.
6. DeFrances CJ, Cullen KA, Kozak LJ. National Hospital Discharge Survey: 2005 annual summary with detailed diagnosis and procedure data. Vital Health Stat 13. 2007;(165):1-209.
7. American College of Obstetricians and Gynecologists. ACOG practice bulletin no. 120: use of prophylactic antibiotics in labor and delivery. Obstet Gynecol. 2011;117:1472-1483.
8. Thigpen BD, Hood WA, Chauhan S, et al. Timing of prophylactic antibiotic administration in the uninfected laboring gravida: a randomized clinical trial. Am J Obstet Gynecol. 2005; 192:1864-1868.
9. Costantine MM, Rahman M, Ghulmiyah L, et al. Timing of perioperative antibiotics for cesarean delivery: a meta-analysis. Am J Obstet Gynecol. 2008;199:301.e1-e6.
10. Andrews WW, Hauth JC, Cliver SP, et al. Randomized clinical trial of extended spectrum antibiotic prophylaxis with coverage for Ureaplasma urealyticum to reduce post-cesarean delivery endometritis. Obstet Gynecol. 2003;101:1183-1189.
11. Howard PA. Azithromycin-induced proarrhythmia and cardiovascular death. Ann Pharmacother. 2013;47:1547-1551.

Article PDF
Author and Disclosure Information

Gregory Castelli and Allison Flaherty practice at the University of Pittsburgh Medical Center St. Margaret. Jennie B. Jarrett is with the University of Illinois at Chicago.

Issue
Clinician Reviews - 28(1)
Publications
Topics
Page Number
12-14
Sections
Author and Disclosure Information

Gregory Castelli and Allison Flaherty practice at the University of Pittsburgh Medical Center St. Margaret. Jennie B. Jarrett is with the University of Illinois at Chicago.

Author and Disclosure Information

Gregory Castelli and Allison Flaherty practice at the University of Pittsburgh Medical Center St. Margaret. Jennie B. Jarrett is with the University of Illinois at Chicago.

Article PDF
Article PDF

A 26-year-old G1P0 at 40w1d presents in spontaneous labor and is dilated to 4 cm. The patient reached complete cervical dilation after artificial rupture of membranes and oxytocin augmentation. After four hours of pushing, there has been minimal descent of the fetal vertex beyond +1 station with significant caput succedaneum. Her physician decides to proceed with cesarean delivery.2,3 What antibiotics should be administered prior to incision to reduce postoperative infection?

The CDC reports that nearly 1.3 million cesarean deliveries were performed in the United States in 2015, which represents about a third of all births.4 C-section is the most common major surgical procedure performed in this country and is associated with an infection rate five to 10 times that of vaginal delivery.5,6 Pregnancy-associated infection, particularly during delivery, is a significant risk and the fourth most common cause of maternal death in the United States.5

The current standard of care in cesarean delivery is antibiotic prophylaxis (often a first-generation cephalosporin) prior to skin incision.7 The majority of c-sections performed are nonelective, and of these, postoperative infections occur in 12% of women who receive standard prophylaxis.8,9 A small, single-center design trial suggested that azithromycin adjunctive therapy expands antibiotic coverage to Ureaplasma species, resulting in a lower risk for postoperative infection.10

This study evaluated the use of azithromycin adjunctive therapy, in addition to standard antibiotic prophylaxis, to reduce the risk for postoperative infections in women receiving nonelective c-sections.

STUDY SUMMARY

Reduced infections up to 6 weeks post–c-section

A multicenter, randomized, double-blind trial conducted in 14 hospitals in the United States evaluated the effect of a one-time dose of azithromycin (500 mg IV) on post­cesarean infections. Women with a singleton pregnancy of at least 24 weeks’ gestation were eligible for inclusion if they required nonelective cesarean delivery during labor or at least four hours after membrane rupture. Patients were excluded if they had a known azithromycin allergy, subsequent vaginal delivery, azithromycin use within the week prior to randomization, extensive hepatic or renal dysfunction, a known history of prolonged QT interval, or substantial electrolyte abnormalities. Patients were eligible even if they were receiving other antibiotics for a positive group B Streptococcus screening.1

All patients (N = 2,013) were treated with standard antibiotic prophylaxis (most often cefazolin), according to individual institution protocols. The women were randomized to receive either an azithromycin 500 mg/250 mL IV infusion (n = 1,019) or an identical placebo IV infusion (n = 994) within one hour of the procedure. The primary outcome was a composite endpoint of endometritis, wound infection, or other infections occurring up to six weeks after the c-section. Secondary outcomes included neonatal death, sepsis, and other neonatal and maternal complications.1

Patients in the placebo group had a higher rate of smoking during pregnancy; the researchers found no other significant differences.1

Results. The primary composite outcome occurred less frequently in the azithromycin group than in the placebo group (6.1% vs 12.0%; relative risk [RR], 0.51; number needed to treat [NNT], 17). When the researchers looked at the individual elements of the primary composite outcome, two had significant reductions versus placebo. Endometritis (3.8% vs 6.1%; RR, 0.62; NNT, 43) and wound infections (2.4% vs 6.6%; RR, 0.35; NNT, 24) occurred significantly less frequently, but there was no difference for other infections (0.3% vs 0.6%; RR, 0.49).

Serious maternal adverse events were also lower in the treatment group than in the control group (1.5% vs 2.9%; RR, 0.5; NNT, 71). There was no difference in composite secondary neonatal outcomes, including death and serious complications (14.3% vs 13.6%; RR, 1.05).1

 

 

WHAT’S NEW

Fewer infections without more adverse events

This study showed that adding azithromycin to standard antibiotic prophylaxis within one hour of a c-section reduces post–cesarean delivery infection rates without increasing the risk for maternal or neonatal adverse events.

CAVEATS

Caution with prolonged QT

While azithromycin was efficacious and well tolerated in the study, not every patient can take it. Patients with a previous drug reaction or allergy should avoid it, and experts advise prescribing it with caution for patients who have (or are at increased risk for) a prolonged QT interval, including those on other QT-prolonging medications.

Of note, women with scheduled c-sections and those with chorioamnionitis or another infection requiring postpartum antibiotics were excluded from this study. Thus, it is unknown if azithromycin use decreases complications in these patients.

CHALLENGES TO IMPLEMENTATION

Drug availability is key

Nonelective c-sections are performed based on many factors, including a nonreassuring fetal heart rate. In many of these cases, speed of cesarean delivery may mean the difference between positive and negative outcomes. Availability of azithromycin on labor and delivery floors for timely administration within one hour of the procedure is important.

Additionally, azithromycin has known QT prolongation risks.11 While the baseline QT interval is not known for many healthy young women, this should be considered when azithromycin is utilized in combination with other medications that may prolong the QT interval.

ACKNOWLEDGEMENT

The PURLs Surveillance System was supported in part by Grant Number UL1RR024999 from the National Center For Research Resources, a Clinical Translational Science Award to the University of Chicago. The content is solely the responsibility of the authors and does not necessarily represent the official views of the National Center For Research Resources or the National Institutes of Health.

Copyright © 2017. The Family Physicians Inquiries Network. All rights reserved.

Reprinted with permission from the Family Physicians Inquiries Network and The Journal of Family Practice (2017;66[12]:762-764).

A 26-year-old G1P0 at 40w1d presents in spontaneous labor and is dilated to 4 cm. The patient reached complete cervical dilation after artificial rupture of membranes and oxytocin augmentation. After four hours of pushing, there has been minimal descent of the fetal vertex beyond +1 station with significant caput succedaneum. Her physician decides to proceed with cesarean delivery.2,3 What antibiotics should be administered prior to incision to reduce postoperative infection?

The CDC reports that nearly 1.3 million cesarean deliveries were performed in the United States in 2015, which represents about a third of all births.4 C-section is the most common major surgical procedure performed in this country and is associated with an infection rate five to 10 times that of vaginal delivery.5,6 Pregnancy-associated infection, particularly during delivery, is a significant risk and the fourth most common cause of maternal death in the United States.5

The current standard of care in cesarean delivery is antibiotic prophylaxis (often a first-generation cephalosporin) prior to skin incision.7 The majority of c-sections performed are nonelective, and of these, postoperative infections occur in 12% of women who receive standard prophylaxis.8,9 A small, single-center design trial suggested that azithromycin adjunctive therapy expands antibiotic coverage to Ureaplasma species, resulting in a lower risk for postoperative infection.10

This study evaluated the use of azithromycin adjunctive therapy, in addition to standard antibiotic prophylaxis, to reduce the risk for postoperative infections in women receiving nonelective c-sections.

STUDY SUMMARY

Reduced infections up to 6 weeks post–c-section

A multicenter, randomized, double-blind trial conducted in 14 hospitals in the United States evaluated the effect of a one-time dose of azithromycin (500 mg IV) on post­cesarean infections. Women with a singleton pregnancy of at least 24 weeks’ gestation were eligible for inclusion if they required nonelective cesarean delivery during labor or at least four hours after membrane rupture. Patients were excluded if they had a known azithromycin allergy, subsequent vaginal delivery, azithromycin use within the week prior to randomization, extensive hepatic or renal dysfunction, a known history of prolonged QT interval, or substantial electrolyte abnormalities. Patients were eligible even if they were receiving other antibiotics for a positive group B Streptococcus screening.1

All patients (N = 2,013) were treated with standard antibiotic prophylaxis (most often cefazolin), according to individual institution protocols. The women were randomized to receive either an azithromycin 500 mg/250 mL IV infusion (n = 1,019) or an identical placebo IV infusion (n = 994) within one hour of the procedure. The primary outcome was a composite endpoint of endometritis, wound infection, or other infections occurring up to six weeks after the c-section. Secondary outcomes included neonatal death, sepsis, and other neonatal and maternal complications.1

Patients in the placebo group had a higher rate of smoking during pregnancy; the researchers found no other significant differences.1

Results. The primary composite outcome occurred less frequently in the azithromycin group than in the placebo group (6.1% vs 12.0%; relative risk [RR], 0.51; number needed to treat [NNT], 17). When the researchers looked at the individual elements of the primary composite outcome, two had significant reductions versus placebo. Endometritis (3.8% vs 6.1%; RR, 0.62; NNT, 43) and wound infections (2.4% vs 6.6%; RR, 0.35; NNT, 24) occurred significantly less frequently, but there was no difference for other infections (0.3% vs 0.6%; RR, 0.49).

Serious maternal adverse events were also lower in the treatment group than in the control group (1.5% vs 2.9%; RR, 0.5; NNT, 71). There was no difference in composite secondary neonatal outcomes, including death and serious complications (14.3% vs 13.6%; RR, 1.05).1

 

 

WHAT’S NEW

Fewer infections without more adverse events

This study showed that adding azithromycin to standard antibiotic prophylaxis within one hour of a c-section reduces post–cesarean delivery infection rates without increasing the risk for maternal or neonatal adverse events.

CAVEATS

Caution with prolonged QT

While azithromycin was efficacious and well tolerated in the study, not every patient can take it. Patients with a previous drug reaction or allergy should avoid it, and experts advise prescribing it with caution for patients who have (or are at increased risk for) a prolonged QT interval, including those on other QT-prolonging medications.

Of note, women with scheduled c-sections and those with chorioamnionitis or another infection requiring postpartum antibiotics were excluded from this study. Thus, it is unknown if azithromycin use decreases complications in these patients.

CHALLENGES TO IMPLEMENTATION

Drug availability is key

Nonelective c-sections are performed based on many factors, including a nonreassuring fetal heart rate. In many of these cases, speed of cesarean delivery may mean the difference between positive and negative outcomes. Availability of azithromycin on labor and delivery floors for timely administration within one hour of the procedure is important.

Additionally, azithromycin has known QT prolongation risks.11 While the baseline QT interval is not known for many healthy young women, this should be considered when azithromycin is utilized in combination with other medications that may prolong the QT interval.

ACKNOWLEDGEMENT

The PURLs Surveillance System was supported in part by Grant Number UL1RR024999 from the National Center For Research Resources, a Clinical Translational Science Award to the University of Chicago. The content is solely the responsibility of the authors and does not necessarily represent the official views of the National Center For Research Resources or the National Institutes of Health.

Copyright © 2017. The Family Physicians Inquiries Network. All rights reserved.

Reprinted with permission from the Family Physicians Inquiries Network and The Journal of Family Practice (2017;66[12]:762-764).

References

1. Tita AT, Szychowski JM, Boggess K, et al. Adjunctive azithromycin prophylaxis for cesarean delivery. N Engl J Med. 2016; 375:1231-1241.
2. American College of Obstetricians and Gynecologists. Safe prevention of the primary cesarean delivery. Obstetric Care Consensus No. 1. Obstet Gynecol. 2014;123:693-711.
3. Rouse DJ, Weiner SJ, Bloom SL, et al. Second-stage labor duration in nulliparous women: relationship to maternal and perinatal outcomes. Am J Obstet Gynecol. 2009;201(4):357.e1-e7.
4. CDC National Center for Health Statistics. Births—Method of Delivery. www.cdc.gov/nchs/fastats/delivery.htm. Accessed December 1, 2017.
5. Perencevich EN, Sands KE, Cosgrove SE, et al. Health and economic impact of surgical site infections diagnosed after hospital discharge. Emerg Infect Dis. 2003;9:196-203.
6. DeFrances CJ, Cullen KA, Kozak LJ. National Hospital Discharge Survey: 2005 annual summary with detailed diagnosis and procedure data. Vital Health Stat 13. 2007;(165):1-209.
7. American College of Obstetricians and Gynecologists. ACOG practice bulletin no. 120: use of prophylactic antibiotics in labor and delivery. Obstet Gynecol. 2011;117:1472-1483.
8. Thigpen BD, Hood WA, Chauhan S, et al. Timing of prophylactic antibiotic administration in the uninfected laboring gravida: a randomized clinical trial. Am J Obstet Gynecol. 2005; 192:1864-1868.
9. Costantine MM, Rahman M, Ghulmiyah L, et al. Timing of perioperative antibiotics for cesarean delivery: a meta-analysis. Am J Obstet Gynecol. 2008;199:301.e1-e6.
10. Andrews WW, Hauth JC, Cliver SP, et al. Randomized clinical trial of extended spectrum antibiotic prophylaxis with coverage for Ureaplasma urealyticum to reduce post-cesarean delivery endometritis. Obstet Gynecol. 2003;101:1183-1189.
11. Howard PA. Azithromycin-induced proarrhythmia and cardiovascular death. Ann Pharmacother. 2013;47:1547-1551.

References

1. Tita AT, Szychowski JM, Boggess K, et al. Adjunctive azithromycin prophylaxis for cesarean delivery. N Engl J Med. 2016; 375:1231-1241.
2. American College of Obstetricians and Gynecologists. Safe prevention of the primary cesarean delivery. Obstetric Care Consensus No. 1. Obstet Gynecol. 2014;123:693-711.
3. Rouse DJ, Weiner SJ, Bloom SL, et al. Second-stage labor duration in nulliparous women: relationship to maternal and perinatal outcomes. Am J Obstet Gynecol. 2009;201(4):357.e1-e7.
4. CDC National Center for Health Statistics. Births—Method of Delivery. www.cdc.gov/nchs/fastats/delivery.htm. Accessed December 1, 2017.
5. Perencevich EN, Sands KE, Cosgrove SE, et al. Health and economic impact of surgical site infections diagnosed after hospital discharge. Emerg Infect Dis. 2003;9:196-203.
6. DeFrances CJ, Cullen KA, Kozak LJ. National Hospital Discharge Survey: 2005 annual summary with detailed diagnosis and procedure data. Vital Health Stat 13. 2007;(165):1-209.
7. American College of Obstetricians and Gynecologists. ACOG practice bulletin no. 120: use of prophylactic antibiotics in labor and delivery. Obstet Gynecol. 2011;117:1472-1483.
8. Thigpen BD, Hood WA, Chauhan S, et al. Timing of prophylactic antibiotic administration in the uninfected laboring gravida: a randomized clinical trial. Am J Obstet Gynecol. 2005; 192:1864-1868.
9. Costantine MM, Rahman M, Ghulmiyah L, et al. Timing of perioperative antibiotics for cesarean delivery: a meta-analysis. Am J Obstet Gynecol. 2008;199:301.e1-e6.
10. Andrews WW, Hauth JC, Cliver SP, et al. Randomized clinical trial of extended spectrum antibiotic prophylaxis with coverage for Ureaplasma urealyticum to reduce post-cesarean delivery endometritis. Obstet Gynecol. 2003;101:1183-1189.
11. Howard PA. Azithromycin-induced proarrhythmia and cardiovascular death. Ann Pharmacother. 2013;47:1547-1551.

Issue
Clinician Reviews - 28(1)
Issue
Clinician Reviews - 28(1)
Page Number
12-14
Page Number
12-14
Publications
Publications
Topics
Article Type
Display Headline
Does Azithromycin Have a Role in Cesarean Sections?
Display Headline
Does Azithromycin Have a Role in Cesarean Sections?
Sections
Disallow All Ads
Content Gating
No Gating (article Unlocked/Free)
Alternative CME
Disqus Comments
Default
Use ProPublica
Article PDF Media

Does azithromycin have a role in cesarean sections?

Article Type
Changed
Tue, 06/23/2020 - 12:00
Display Headline
Does azithromycin have a role in cesarean sections?
 

ILLUSTRATIVE CASE

A 26-year-old G1P0 at 40w1d presents in spontaneous labor and is dilated to 4 cm. The patient reached complete cervical dilation after artificial rupture of membranes and oxytocin augmentation. After 4 hours of pushing, there has been minimal descent of the fetal vertex beyond +1 station with significant caput succedaneum. Her physician decides to proceed with cesarean delivery.2,3 What antibiotics should be administered prior to incision to reduce postoperative infection?

The Centers for Disease Control and Prevention (CDC) reports that nearly 1.3 million cesarean deliveries were performed in the United States in 2015, which represents about a third of all births.4 C-section is the most common major surgical procedure performed in this country and is associated with an infection rate 5 to 10 times that of vaginal delivery.5,6 Pregnancy-associated infection, particularly during delivery, is a significant risk and the fourth most common cause of maternal death in the United States.5

The current standard of care in cesarean delivery is antibiotic prophylaxis (often a first-generation cephalosporin) prior to skin incision.7 The majority of c-sections performed are nonelective, and of these, postoperative infections occur in 12% of women who receive standard prophylaxis.8,9 A small, single-center design trial suggested azithromycin adjunctive therapy expands antibiotic coverage to Ureaplasma species, resulting in a lower risk of postoperative infection.10

This study evaluated the use of azithromycin adjunctive therapy, in addition to standard antibiotic prophylaxis, to reduce the risk of postoperative infections in women receiving nonelective c-sections.

STUDY SUMMARY

Azithromycin reduced maternal infections up to 6 weeks post–c-section

A multicenter, randomized double-blind trial conducted in 14 hospitals in the United States evaluated the effect of a one-time dose of 500 mg intravenous (IV) azithromycin on post-cesarean infections. Women with a singleton pregnancy of at least 24 weeks’ gestation were eligible for inclusion if they required nonelective cesarean delivery during labor or at least 4 hours after membrane rupture. Patients were excluded if they had a known azithromycin allergy, subsequent vaginal delivery, azithromycin use within the week prior to randomization, extensive hepatic or renal dysfunction, a known history of prolonged QT interval, or substantial electrolyte abnormalities. Patients were eligible even if they were receiving other antibiotics for a positive group B Streptococcus screening.1

All patients (N=2013) were treated with standard antibiotic prophylaxis, most often cefazolin, according to individual institution protocols. The women were randomized to receive either an azithromycin 500 mg/250 mL IV infusion (n=1019) or an identical placebo IV infusion (n=994) within one hour of the procedure. The primary outcome was a composite endpoint of endometritis, wound infection, or other infections occurring up to 6 weeks after the c-section. Secondary outcomes included neonatal death, sepsis, and other neonatal and maternal complications.1

Patients in the placebo group had a higher rate of smoking during pregnancy; the researchers found no other significant differences.1

Results. The primary composite outcome occurred less frequently in the azithromycin group than in the placebo group (6.1% vs 12.1%; relative risk [RR]=0.51; 95% confidence interval [CI], 0.38-0.68; number needed to treat [NNT]=17). When the researchers looked at the individual elements of the primary composite outcome, 2 had significant reductions vs placebo.

Endometritis (3.8% vs 6.1%; RR=0.62; 95% CI, 0.42-0.92; NNT=44) and wound infections (2.4% vs 6.6%; RR=0.35; 95% CI, 0.22-0.56; NNT=24) occurred significantly less frequently, but there was no difference for other infections (0.3% vs 0.6%; RR=0.49; 95% CI, 0.12-1.94). Serious maternal adverse events were also lower with treatment than in the control group (1.5% vs 2.9%; RR=0.5; 95% CI, 0.27-0.94; NNT=71). There was no difference in composite secondary neonatal outcomes including death and serious complications (14.3% vs 13.6%; RR=1.05; 95% CI, 0.85-1.31).1

 

 

 

WHAT’S NEW

Azithromycin reduces infections without increasing adverse events

This study showed that adding azithromycin to standard antibiotic prophylaxis within one hour of a c-section reduces post-cesarean delivery infection rates without increasing the risk of maternal or neonatal adverse events.

CAVEATS

Proceed with caution in those with prolonged QT intervals

While azithromycin was efficacious and well tolerated in the study, not every patient can take it. Patients with a previous drug reaction or allergy should avoid it, and experts advise prescribing it with caution for patients who have (or are at increased risk for) a prolonged QT interval, including those on other QT-prolonging medications.

Adding azithromycin to standard antibiotic prophylaxis within one hour of a c-section reduces post-cesarean delivery infection rates without increasing the risk of adverse events.

Of note, women with scheduled c-sections and those with chorioamnionitis or another infection requiring postpartum antibiotics were excluded from this study. Thus, it is unknown if azithromycin use decreases complications in these patients.

CHALLENGES TO IMPLEMENTATION

Speed of procedure is often paramount, so drug availability is key

Nonelective c-sections occur based on many factors that include a non-reassuring fetal heart rate. In many of these cases, speed of cesarean delivery may mean the difference between positive and negative outcomes. Availability of azithromycin on labor and delivery floors for timely administration within one hour of the procedure is important.

Additionally, azithromycin has known QT prolongation risks.11 While the baseline QT interval is not known for many healthy, young women, this should be considered when azithromycin is utilized in combination with other medications that may prolong the QT interval.

ACKNOWLEDGEMENT

The PURLs Surveillance System was supported in part by Grant Number UL1RR024999 from the National Center For Research Resources, a Clinical Translational Science Award to the University of Chicago. The content is solely the responsibility of the authors and does not necessarily represent the official views of the National Center For Research Resources or the National Institutes of Health.

Files
References

1. Tita AT, Szychowski JM, Boggess K, et al. Adjunctive azithromycin prophylaxis for cesarean delivery. N Engl J Med. 2016;375:1231-1241.

2. Safe prevention of the primary cesarean delivery. Obstetric Care Consensus No. 1. American College of Obstetricians and Gynecologists. Obstet Gynecol. 2014;123:693-711.

3. Rouse DJ, Weiner SJ, Bloom SL, et al. Second-stage labor duration in nulliparous women: relationship to maternal and perinatal outcomes. Am J Obstet Gynecol. 2009;201:357. e1-e7.

4. National Vital Statistics Reports. Centers for Disease Control and Prevention: Births, Mode of Delivery. Available at: https://www.cdc.gov/nchs/fastats/delivery.htm. Updated January 5, 2017. Accessed August 4, 2017.

5. Perencevich EN, Sands KE, Cosgrove SE, et al. Health and economic impact of surgical site infections diagnosed after hospital discharge. Emerg Infect Dis. 2003;9:196-203.

6. DeFrances CJ, Cullen KA, Kozak LJ. National Hospital Discharge Survey: 2005 annual summary with detailed diagnosis and procedure data. Vital Health Stat 13. 2007:1-209.

7. American College of Obstetricians and Gynecologists. ACOG practice bulletin no. 120: use of prophylactic antibiotics in labor and delivery. Obstet Gynecol. 2011;117:1472-1483.

8. Thigpen BD, Hood WA, Chauhan S, et al. Timing of prophylactic antibiotic administration in the uninfected laboring gravida: a randomized clinical trial. Am J Obstet Gynecol. 2005;192:1864-1868.

9. Costantine MM, Rahman M, Ghulmiyah L, et al. Timing of perioperative antibiotics for cesarean delivery: a metaanalysis. Am J Obstet Gynecol. 2008;199:301. e1-e6.

10. Andrews WW, Hauth JC, Cliver SP, et al. Randomized clinical trial of extended spectrum antibiotic prophylaxis with coverage for Ureaplasma urealyticum to reduce post-cesarean delivery endometritis. Obstet Gynecol. 2003;101:1183-1189.

11. Howard PA. Azithromycin-induced proarrhythmia and cardiovascular death. Ann Pharmacother. 2013;47:1547-1551.

Article PDF
Author and Disclosure Information

UPMC St. Margaret, Pittsburgh, Pa (Drs. Castelli and Flaherty); University of Illinois at Chicago (Dr. Jarrett)

DEPUTY EDITOR
Corey Lyon, DO

University of Colorado Family Medicine Residency, Denver

Issue
The Journal of Family Practice - 66(12)
Publications
Topics
Page Number
762-764
Sections
Files
Files
Author and Disclosure Information

UPMC St. Margaret, Pittsburgh, Pa (Drs. Castelli and Flaherty); University of Illinois at Chicago (Dr. Jarrett)

DEPUTY EDITOR
Corey Lyon, DO

University of Colorado Family Medicine Residency, Denver

Author and Disclosure Information

UPMC St. Margaret, Pittsburgh, Pa (Drs. Castelli and Flaherty); University of Illinois at Chicago (Dr. Jarrett)

DEPUTY EDITOR
Corey Lyon, DO

University of Colorado Family Medicine Residency, Denver

Article PDF
Article PDF
 

ILLUSTRATIVE CASE

A 26-year-old G1P0 at 40w1d presents in spontaneous labor and is dilated to 4 cm. The patient reached complete cervical dilation after artificial rupture of membranes and oxytocin augmentation. After 4 hours of pushing, there has been minimal descent of the fetal vertex beyond +1 station with significant caput succedaneum. Her physician decides to proceed with cesarean delivery.2,3 What antibiotics should be administered prior to incision to reduce postoperative infection?

The Centers for Disease Control and Prevention (CDC) reports that nearly 1.3 million cesarean deliveries were performed in the United States in 2015, which represents about a third of all births.4 C-section is the most common major surgical procedure performed in this country and is associated with an infection rate 5 to 10 times that of vaginal delivery.5,6 Pregnancy-associated infection, particularly during delivery, is a significant risk and the fourth most common cause of maternal death in the United States.5

The current standard of care in cesarean delivery is antibiotic prophylaxis (often a first-generation cephalosporin) prior to skin incision.7 The majority of c-sections performed are nonelective, and of these, postoperative infections occur in 12% of women who receive standard prophylaxis.8,9 A small, single-center design trial suggested azithromycin adjunctive therapy expands antibiotic coverage to Ureaplasma species, resulting in a lower risk of postoperative infection.10

This study evaluated the use of azithromycin adjunctive therapy, in addition to standard antibiotic prophylaxis, to reduce the risk of postoperative infections in women receiving nonelective c-sections.

STUDY SUMMARY

Azithromycin reduced maternal infections up to 6 weeks post–c-section

A multicenter, randomized double-blind trial conducted in 14 hospitals in the United States evaluated the effect of a one-time dose of 500 mg intravenous (IV) azithromycin on post-cesarean infections. Women with a singleton pregnancy of at least 24 weeks’ gestation were eligible for inclusion if they required nonelective cesarean delivery during labor or at least 4 hours after membrane rupture. Patients were excluded if they had a known azithromycin allergy, subsequent vaginal delivery, azithromycin use within the week prior to randomization, extensive hepatic or renal dysfunction, a known history of prolonged QT interval, or substantial electrolyte abnormalities. Patients were eligible even if they were receiving other antibiotics for a positive group B Streptococcus screening.1

All patients (N=2013) were treated with standard antibiotic prophylaxis, most often cefazolin, according to individual institution protocols. The women were randomized to receive either an azithromycin 500 mg/250 mL IV infusion (n=1019) or an identical placebo IV infusion (n=994) within one hour of the procedure. The primary outcome was a composite endpoint of endometritis, wound infection, or other infections occurring up to 6 weeks after the c-section. Secondary outcomes included neonatal death, sepsis, and other neonatal and maternal complications.1

Patients in the placebo group had a higher rate of smoking during pregnancy; the researchers found no other significant differences.1

Results. The primary composite outcome occurred less frequently in the azithromycin group than in the placebo group (6.1% vs 12.1%; relative risk [RR]=0.51; 95% confidence interval [CI], 0.38-0.68; number needed to treat [NNT]=17). When the researchers looked at the individual elements of the primary composite outcome, 2 had significant reductions vs placebo.

Endometritis (3.8% vs 6.1%; RR=0.62; 95% CI, 0.42-0.92; NNT=44) and wound infections (2.4% vs 6.6%; RR=0.35; 95% CI, 0.22-0.56; NNT=24) occurred significantly less frequently, but there was no difference for other infections (0.3% vs 0.6%; RR=0.49; 95% CI, 0.12-1.94). Serious maternal adverse events were also lower with treatment than in the control group (1.5% vs 2.9%; RR=0.5; 95% CI, 0.27-0.94; NNT=71). There was no difference in composite secondary neonatal outcomes including death and serious complications (14.3% vs 13.6%; RR=1.05; 95% CI, 0.85-1.31).1

 

 

 

WHAT’S NEW

Azithromycin reduces infections without increasing adverse events

This study showed that adding azithromycin to standard antibiotic prophylaxis within one hour of a c-section reduces post-cesarean delivery infection rates without increasing the risk of maternal or neonatal adverse events.

CAVEATS

Proceed with caution in those with prolonged QT intervals

While azithromycin was efficacious and well tolerated in the study, not every patient can take it. Patients with a previous drug reaction or allergy should avoid it, and experts advise prescribing it with caution for patients who have (or are at increased risk for) a prolonged QT interval, including those on other QT-prolonging medications.

Adding azithromycin to standard antibiotic prophylaxis within one hour of a c-section reduces post-cesarean delivery infection rates without increasing the risk of adverse events.

Of note, women with scheduled c-sections and those with chorioamnionitis or another infection requiring postpartum antibiotics were excluded from this study. Thus, it is unknown if azithromycin use decreases complications in these patients.

CHALLENGES TO IMPLEMENTATION

Speed of procedure is often paramount, so drug availability is key

Nonelective c-sections occur based on many factors that include a non-reassuring fetal heart rate. In many of these cases, speed of cesarean delivery may mean the difference between positive and negative outcomes. Availability of azithromycin on labor and delivery floors for timely administration within one hour of the procedure is important.

Additionally, azithromycin has known QT prolongation risks.11 While the baseline QT interval is not known for many healthy, young women, this should be considered when azithromycin is utilized in combination with other medications that may prolong the QT interval.

ACKNOWLEDGEMENT

The PURLs Surveillance System was supported in part by Grant Number UL1RR024999 from the National Center For Research Resources, a Clinical Translational Science Award to the University of Chicago. The content is solely the responsibility of the authors and does not necessarily represent the official views of the National Center For Research Resources or the National Institutes of Health.

 

ILLUSTRATIVE CASE

A 26-year-old G1P0 at 40w1d presents in spontaneous labor and is dilated to 4 cm. The patient reached complete cervical dilation after artificial rupture of membranes and oxytocin augmentation. After 4 hours of pushing, there has been minimal descent of the fetal vertex beyond +1 station with significant caput succedaneum. Her physician decides to proceed with cesarean delivery.2,3 What antibiotics should be administered prior to incision to reduce postoperative infection?

The Centers for Disease Control and Prevention (CDC) reports that nearly 1.3 million cesarean deliveries were performed in the United States in 2015, which represents about a third of all births.4 C-section is the most common major surgical procedure performed in this country and is associated with an infection rate 5 to 10 times that of vaginal delivery.5,6 Pregnancy-associated infection, particularly during delivery, is a significant risk and the fourth most common cause of maternal death in the United States.5

The current standard of care in cesarean delivery is antibiotic prophylaxis (often a first-generation cephalosporin) prior to skin incision.7 The majority of c-sections performed are nonelective, and of these, postoperative infections occur in 12% of women who receive standard prophylaxis.8,9 A small, single-center design trial suggested azithromycin adjunctive therapy expands antibiotic coverage to Ureaplasma species, resulting in a lower risk of postoperative infection.10

This study evaluated the use of azithromycin adjunctive therapy, in addition to standard antibiotic prophylaxis, to reduce the risk of postoperative infections in women receiving nonelective c-sections.

STUDY SUMMARY

Azithromycin reduced maternal infections up to 6 weeks post–c-section

A multicenter, randomized double-blind trial conducted in 14 hospitals in the United States evaluated the effect of a one-time dose of 500 mg intravenous (IV) azithromycin on post-cesarean infections. Women with a singleton pregnancy of at least 24 weeks’ gestation were eligible for inclusion if they required nonelective cesarean delivery during labor or at least 4 hours after membrane rupture. Patients were excluded if they had a known azithromycin allergy, subsequent vaginal delivery, azithromycin use within the week prior to randomization, extensive hepatic or renal dysfunction, a known history of prolonged QT interval, or substantial electrolyte abnormalities. Patients were eligible even if they were receiving other antibiotics for a positive group B Streptococcus screening.1

All patients (N=2013) were treated with standard antibiotic prophylaxis, most often cefazolin, according to individual institution protocols. The women were randomized to receive either an azithromycin 500 mg/250 mL IV infusion (n=1019) or an identical placebo IV infusion (n=994) within one hour of the procedure. The primary outcome was a composite endpoint of endometritis, wound infection, or other infections occurring up to 6 weeks after the c-section. Secondary outcomes included neonatal death, sepsis, and other neonatal and maternal complications.1

Patients in the placebo group had a higher rate of smoking during pregnancy; the researchers found no other significant differences.1

Results. The primary composite outcome occurred less frequently in the azithromycin group than in the placebo group (6.1% vs 12.1%; relative risk [RR]=0.51; 95% confidence interval [CI], 0.38-0.68; number needed to treat [NNT]=17). When the researchers looked at the individual elements of the primary composite outcome, 2 had significant reductions vs placebo.

Endometritis (3.8% vs 6.1%; RR=0.62; 95% CI, 0.42-0.92; NNT=44) and wound infections (2.4% vs 6.6%; RR=0.35; 95% CI, 0.22-0.56; NNT=24) occurred significantly less frequently, but there was no difference for other infections (0.3% vs 0.6%; RR=0.49; 95% CI, 0.12-1.94). Serious maternal adverse events were also lower with treatment than in the control group (1.5% vs 2.9%; RR=0.5; 95% CI, 0.27-0.94; NNT=71). There was no difference in composite secondary neonatal outcomes including death and serious complications (14.3% vs 13.6%; RR=1.05; 95% CI, 0.85-1.31).1

 

 

 

WHAT’S NEW

Azithromycin reduces infections without increasing adverse events

This study showed that adding azithromycin to standard antibiotic prophylaxis within one hour of a c-section reduces post-cesarean delivery infection rates without increasing the risk of maternal or neonatal adverse events.

CAVEATS

Proceed with caution in those with prolonged QT intervals

While azithromycin was efficacious and well tolerated in the study, not every patient can take it. Patients with a previous drug reaction or allergy should avoid it, and experts advise prescribing it with caution for patients who have (or are at increased risk for) a prolonged QT interval, including those on other QT-prolonging medications.

Adding azithromycin to standard antibiotic prophylaxis within one hour of a c-section reduces post-cesarean delivery infection rates without increasing the risk of adverse events.

Of note, women with scheduled c-sections and those with chorioamnionitis or another infection requiring postpartum antibiotics were excluded from this study. Thus, it is unknown if azithromycin use decreases complications in these patients.

CHALLENGES TO IMPLEMENTATION

Speed of procedure is often paramount, so drug availability is key

Nonelective c-sections occur based on many factors that include a non-reassuring fetal heart rate. In many of these cases, speed of cesarean delivery may mean the difference between positive and negative outcomes. Availability of azithromycin on labor and delivery floors for timely administration within one hour of the procedure is important.

Additionally, azithromycin has known QT prolongation risks.11 While the baseline QT interval is not known for many healthy, young women, this should be considered when azithromycin is utilized in combination with other medications that may prolong the QT interval.

ACKNOWLEDGEMENT

The PURLs Surveillance System was supported in part by Grant Number UL1RR024999 from the National Center For Research Resources, a Clinical Translational Science Award to the University of Chicago. The content is solely the responsibility of the authors and does not necessarily represent the official views of the National Center For Research Resources or the National Institutes of Health.

References

1. Tita AT, Szychowski JM, Boggess K, et al. Adjunctive azithromycin prophylaxis for cesarean delivery. N Engl J Med. 2016;375:1231-1241.

2. Safe prevention of the primary cesarean delivery. Obstetric Care Consensus No. 1. American College of Obstetricians and Gynecologists. Obstet Gynecol. 2014;123:693-711.

3. Rouse DJ, Weiner SJ, Bloom SL, et al. Second-stage labor duration in nulliparous women: relationship to maternal and perinatal outcomes. Am J Obstet Gynecol. 2009;201:357. e1-e7.

4. National Vital Statistics Reports. Centers for Disease Control and Prevention: Births, Mode of Delivery. Available at: https://www.cdc.gov/nchs/fastats/delivery.htm. Updated January 5, 2017. Accessed August 4, 2017.

5. Perencevich EN, Sands KE, Cosgrove SE, et al. Health and economic impact of surgical site infections diagnosed after hospital discharge. Emerg Infect Dis. 2003;9:196-203.

6. DeFrances CJ, Cullen KA, Kozak LJ. National Hospital Discharge Survey: 2005 annual summary with detailed diagnosis and procedure data. Vital Health Stat 13. 2007:1-209.

7. American College of Obstetricians and Gynecologists. ACOG practice bulletin no. 120: use of prophylactic antibiotics in labor and delivery. Obstet Gynecol. 2011;117:1472-1483.

8. Thigpen BD, Hood WA, Chauhan S, et al. Timing of prophylactic antibiotic administration in the uninfected laboring gravida: a randomized clinical trial. Am J Obstet Gynecol. 2005;192:1864-1868.

9. Costantine MM, Rahman M, Ghulmiyah L, et al. Timing of perioperative antibiotics for cesarean delivery: a metaanalysis. Am J Obstet Gynecol. 2008;199:301. e1-e6.

10. Andrews WW, Hauth JC, Cliver SP, et al. Randomized clinical trial of extended spectrum antibiotic prophylaxis with coverage for Ureaplasma urealyticum to reduce post-cesarean delivery endometritis. Obstet Gynecol. 2003;101:1183-1189.

11. Howard PA. Azithromycin-induced proarrhythmia and cardiovascular death. Ann Pharmacother. 2013;47:1547-1551.

References

1. Tita AT, Szychowski JM, Boggess K, et al. Adjunctive azithromycin prophylaxis for cesarean delivery. N Engl J Med. 2016;375:1231-1241.

2. Safe prevention of the primary cesarean delivery. Obstetric Care Consensus No. 1. American College of Obstetricians and Gynecologists. Obstet Gynecol. 2014;123:693-711.

3. Rouse DJ, Weiner SJ, Bloom SL, et al. Second-stage labor duration in nulliparous women: relationship to maternal and perinatal outcomes. Am J Obstet Gynecol. 2009;201:357. e1-e7.

4. National Vital Statistics Reports. Centers for Disease Control and Prevention: Births, Mode of Delivery. Available at: https://www.cdc.gov/nchs/fastats/delivery.htm. Updated January 5, 2017. Accessed August 4, 2017.

5. Perencevich EN, Sands KE, Cosgrove SE, et al. Health and economic impact of surgical site infections diagnosed after hospital discharge. Emerg Infect Dis. 2003;9:196-203.

6. DeFrances CJ, Cullen KA, Kozak LJ. National Hospital Discharge Survey: 2005 annual summary with detailed diagnosis and procedure data. Vital Health Stat 13. 2007:1-209.

7. American College of Obstetricians and Gynecologists. ACOG practice bulletin no. 120: use of prophylactic antibiotics in labor and delivery. Obstet Gynecol. 2011;117:1472-1483.

8. Thigpen BD, Hood WA, Chauhan S, et al. Timing of prophylactic antibiotic administration in the uninfected laboring gravida: a randomized clinical trial. Am J Obstet Gynecol. 2005;192:1864-1868.

9. Costantine MM, Rahman M, Ghulmiyah L, et al. Timing of perioperative antibiotics for cesarean delivery: a metaanalysis. Am J Obstet Gynecol. 2008;199:301. e1-e6.

10. Andrews WW, Hauth JC, Cliver SP, et al. Randomized clinical trial of extended spectrum antibiotic prophylaxis with coverage for Ureaplasma urealyticum to reduce post-cesarean delivery endometritis. Obstet Gynecol. 2003;101:1183-1189.

11. Howard PA. Azithromycin-induced proarrhythmia and cardiovascular death. Ann Pharmacother. 2013;47:1547-1551.

Issue
The Journal of Family Practice - 66(12)
Issue
The Journal of Family Practice - 66(12)
Page Number
762-764
Page Number
762-764
Publications
Publications
Topics
Article Type
Display Headline
Does azithromycin have a role in cesarean sections?
Display Headline
Does azithromycin have a role in cesarean sections?
Sections
PURLs Copyright

Copyright © 2017. The Family Physicians Inquiries Network. All rights reserved.

Inside the Article

PRACTICE CHANGER

Prescribe a one-time dose of azithromycin 500 mg intravenously, along with standard antibiotic prophylaxis, at the time of cesarean delivery to prevent postoperative infections.1

STRENGTH OF RECOMMENDATION

B: Based on a single good-quality, randomized controlled trial.

Tita AT, Szychowski JM, Boggess K, et al. Adjunctive azithromycin prophylaxis for cesarean delivery. N Engl J Med. 2016;375:1231-1241.

Disallow All Ads
Content Gating
No Gating (article Unlocked/Free)
Alternative CME
PubMed ID
29202146
Disqus Comments
Default
Use ProPublica
Hide sidebar & use full width
render the right sidebar.
Conference Recap Checkbox
Not Conference Recap
Clinical Edge
Article PDF Media
Media Files

When to “CAP” Off Pneumonia Treatment

Article Type
Changed
Wed, 03/27/2019 - 11:46
Display Headline
When to “CAP” Off Pneumonia Treatment

 

A 65-year-old woman is admitted to your inpatient service from the family health center. She is diagnosed with community-acquired pneumonia (CAP) based on a five-day history of cough and fever and a positive chest x-ray. She now requires oxygen at rest. She has a history of hypertension and diabetes, both of which have been controlled by oral medications. Antibiotic therapy is initiated—but what treatment duration is ideal?

The World Health Organization estimates that pneumonia is the third most common cause of mortality worldwide, causing 3.2 million deaths per year.2 Appropriate prescribing of antibiotics is critical for successful treatment of CAP.

In 2007, the Infectious Diseases Society of America (IDSA) and the American Thoracic Society (ATS) created consensus guidelines for the treatment of CAP.3 These guidelines recommend a minimum five-day course of antibiotics if the patient is clinically stable (defined as afebrile for 48 h; heart rate ≤ 100 beats/min; respiratory rate ≤ 24 breaths/min-1; systolic blood pressure ≥ 90 mm Hg; oxygen saturation ≥ 90%; normal mental status; and able to tolerate oral intake). Longer antibiotic treatment durations are recommended on an individualized basis (eg, if the isolated pathogen is not susceptible to the initial antibiotic or if the infection was caused by an extrapulmonary source).

However, these recommendations are not routinely followed. Practitioners often make it their custom to prescribe longer courses of antibiotics.4 Yet, we know that there are several reasons to consider shorter courses of antibiotics, including lower health care costs, fewer adverse effects, and lower rates of bacterial resistance.5-7

Two meta-analyses were performed to compare the safety and efficacy of short- and long-course antibiotic therapy in CAP (≤ 7 d vs > 7 d, respectively).8,9 Both meta-analyses found no difference in efficacy or safety between shorter and longer courses of antibiotics for CAP. Secondary outcomes noted a trend toward decreased antibiotic-associated adverse events with shorter courses of therapy.8,9

However, there are limitations to broad implementation. Studies included in these analyses utilized a variety of antibiotic treatment regimens and longer courses (7 d vs 5 d) that are not recommended by the IDSA/ATS guidelines. Additionally, studies included both inpatient and outpatient treatment groups, so findings may not apply to an exclusively inpatient CAP population.8,9

This study sought to validate the IDSA/ATS guidelines recommending a five-day course of antibiotics for hospitalized patients with CAP.1

STUDY SUMMARY

No differences in clinical outcomes

This multicenter, double-blind, noninferiority randomized trial compared short-term antibiotic treatment duration (5 d) to physician-discretion antibiotic treatment duration among 312 patients (ages 18 and older) admitted for CAP to one of four teaching hospitals in Spain.1 Pneumonia was diagnosed on chest radiograph with at least one symptom: cough, fever, dyspnea, or chest pain. Patients were excluded if, among other things, they had an immunocompromising condition, lived in a nursing home, had a recent hospital stay, used antibiotics within the previous 30 days, or had an uncommon pathogen, such as Pseudomonas aeruginosa or Staphylococcus aureus.1

After receiving a minimum of five days of antibiotics, patients were randomly assigned to an intervention group (where, if clinically stable, no further antibiotics were given) or a control group (where physicians determined antibiotic duration).1

Primary outcomes were clinical success rate at days 10 and 30 from admission (defined as resolution of signs and symptoms of CAP without further antibiotics) and improvement of CAP-related symptoms (as determined by an 18-item questionnaire scored 0-90, with higher scores indicative of greater severity). Secondary outcomes included duration of antibiotic use, time to clinical improvement, mortality, hospital readmission, hospital length of stay, and CAP recurrence.1

Of the 312 participants, 162 were randomized to the intervention group and 150 to the control group. Mean age of patients in the intervention and control groups was 66.2 and 64.7, respectively. Other baseline demographics were similar between the groups. Nearly 80% of patients received quinolone treatment; < 10% received a ß-lactam plus a macrolide.1

Clinical success rates were similar for the control and intervention groups at day 10 (49% vs 56%, respectively) and day 30 (89% vs 92%). Median antibiotic treatment duration was shorter in the intervention group than in the control group (5 d vs 10 d); the intervention group also had a lower rate of 30-day hospital readmissions (1.4% vs 6.6%). There were no differences for other secondary outcomes.1

 

 

 

WHAT’S NEW

Clinical support for 2007 guidelines

This is the first study to clinically support the IDSA/ATS guidelines, which state that a five-day course of antibiotic therapy for hospitalized adults with CAP is effective and without increased risk for adverse events.

CAVEATS

Generalizability is unclear

This study focused on antibiotic duration for the treatment of CAP in hospitalized patients and mainly used quinolone antibiotics. It remains unclear if duration of therapy is as effective in the outpatient setting or when using alternative antibiotic regimens.

If patients continued to have symptoms (eg, fever or low oxygen saturation on room air) after five days of antibiotics, treatment was continued in the study. Thus, patients in real life who continue to have symptoms may need individualized therapy and may require more than five days of antibiotics.

CHALLENGES TO IMPLEMENTATION

Antibiotics end before clinical improvement

In this study, it took an average of 12 days in both groups for patients to achieve clinical improvement, and upwards of 15 to 18 days for patients to return to normal activity. Patients and providers may be dissatisfied if the treatment course ends days before clinical improvement of symptoms. This may cause prescribers to lengthen the duration of antibiotic therapy inappropriately.

ACKNOWLEDGEMENT

The PURLs Surveillance System was supported in part by Grant Number UL1RR024999 from the National Center For Research Resources, a Clinical Translational Science Award to the University of Chicago. The content is solely the responsibility of the authors and does not necessarily represent the official views of the National Center For Research Resources or the National Institutes of Health.

Copyright © 2017. The Family Physicians Inquiries Network. All rights reserved.

Reprinted with permission from the Family Physicians Inquiries Network and The Journal of Family Practice (2017; 66[10]:629-631).

References

1. Uranga A, España PP, Bilbao A, et al. Duration of antibiotic treatment in community-acquired pneumonia: a multicenter randomized clinical trial. JAMA Intern Med. 2016;176:1257-1265.
2. World Health Organization. The top 10 causes of death. www.who.int/mediacentre/factsheets/fs310/en/index.html. Accessed October 18, 2017.
3. Mandell LA, Wunderink RG, Anzueto A, et al. Infectious Diseases Society of America/American Thoracic Society consensus guidelines on the management of community-acquired pneumonia in adults. Clin Infect Dis. 2007;44(suppl 2):S27-S72.
4. Aliberti S, Blasi F, Zanaboni AM, et al. Duration of antibiotic therapy in hospitalised patients with community-acquired pneumonia. Eur Respir J. 2010;36:128-134.
5. Guillemot D, Carbon C, Balkau B, et al. Low dosage and long treatment duration of ß-lactam: risk factors for carriage of penicillin-resistant Streptococcus pneumoniae. JAMA. 1998; 279:365-370.
6. Opmeer BC, el Moussaoui R, Bossuyt PM, et al. Costs associated with shorter duration of antibiotic therapy in hospitalized patients with mild-to-moderate severe community-acquired pneumonia. J Antimicrob Chemother. 2007;60: 1131-1136.
7. File TM Jr. Clinical efficacy of newer agents in short-duration therapy for community-acquired pneumonia. Clin Infect Dis. 2004;39(suppl 3):S159-S164.
8. Li JZ, Winston LG, Moore DH, et al. Efficacy of short-course antibiotic regimens for community-acquired pneumonia: a meta-analysis. Am J Med. 2007;120:783-790.
9. Dimopoulos G, Matthaiou DK, Karageorgopoulos DE, et al. Short- versus long-course antibacterial therapy for community-acquired pneumonia: a meta-analysis. Drugs. 2008;68: 1841-1854.

Article PDF
Author and Disclosure Information

Gergory Castelli practices at the University of Pittsburgh Medical Center St. Margaret. Jennie B. Jarrett is with the University of Illinois at Chicago.

Issue
Clinician Reviews - 27(11)
Publications
Topics
Page Number
47-49
Sections
Author and Disclosure Information

Gergory Castelli practices at the University of Pittsburgh Medical Center St. Margaret. Jennie B. Jarrett is with the University of Illinois at Chicago.

Author and Disclosure Information

Gergory Castelli practices at the University of Pittsburgh Medical Center St. Margaret. Jennie B. Jarrett is with the University of Illinois at Chicago.

Article PDF
Article PDF
Related Articles

 

A 65-year-old woman is admitted to your inpatient service from the family health center. She is diagnosed with community-acquired pneumonia (CAP) based on a five-day history of cough and fever and a positive chest x-ray. She now requires oxygen at rest. She has a history of hypertension and diabetes, both of which have been controlled by oral medications. Antibiotic therapy is initiated—but what treatment duration is ideal?

The World Health Organization estimates that pneumonia is the third most common cause of mortality worldwide, causing 3.2 million deaths per year.2 Appropriate prescribing of antibiotics is critical for successful treatment of CAP.

In 2007, the Infectious Diseases Society of America (IDSA) and the American Thoracic Society (ATS) created consensus guidelines for the treatment of CAP.3 These guidelines recommend a minimum five-day course of antibiotics if the patient is clinically stable (defined as afebrile for 48 h; heart rate ≤ 100 beats/min; respiratory rate ≤ 24 breaths/min-1; systolic blood pressure ≥ 90 mm Hg; oxygen saturation ≥ 90%; normal mental status; and able to tolerate oral intake). Longer antibiotic treatment durations are recommended on an individualized basis (eg, if the isolated pathogen is not susceptible to the initial antibiotic or if the infection was caused by an extrapulmonary source).

However, these recommendations are not routinely followed. Practitioners often make it their custom to prescribe longer courses of antibiotics.4 Yet, we know that there are several reasons to consider shorter courses of antibiotics, including lower health care costs, fewer adverse effects, and lower rates of bacterial resistance.5-7

Two meta-analyses were performed to compare the safety and efficacy of short- and long-course antibiotic therapy in CAP (≤ 7 d vs > 7 d, respectively).8,9 Both meta-analyses found no difference in efficacy or safety between shorter and longer courses of antibiotics for CAP. Secondary outcomes noted a trend toward decreased antibiotic-associated adverse events with shorter courses of therapy.8,9

However, there are limitations to broad implementation. Studies included in these analyses utilized a variety of antibiotic treatment regimens and longer courses (7 d vs 5 d) that are not recommended by the IDSA/ATS guidelines. Additionally, studies included both inpatient and outpatient treatment groups, so findings may not apply to an exclusively inpatient CAP population.8,9

This study sought to validate the IDSA/ATS guidelines recommending a five-day course of antibiotics for hospitalized patients with CAP.1

STUDY SUMMARY

No differences in clinical outcomes

This multicenter, double-blind, noninferiority randomized trial compared short-term antibiotic treatment duration (5 d) to physician-discretion antibiotic treatment duration among 312 patients (ages 18 and older) admitted for CAP to one of four teaching hospitals in Spain.1 Pneumonia was diagnosed on chest radiograph with at least one symptom: cough, fever, dyspnea, or chest pain. Patients were excluded if, among other things, they had an immunocompromising condition, lived in a nursing home, had a recent hospital stay, used antibiotics within the previous 30 days, or had an uncommon pathogen, such as Pseudomonas aeruginosa or Staphylococcus aureus.1

After receiving a minimum of five days of antibiotics, patients were randomly assigned to an intervention group (where, if clinically stable, no further antibiotics were given) or a control group (where physicians determined antibiotic duration).1

Primary outcomes were clinical success rate at days 10 and 30 from admission (defined as resolution of signs and symptoms of CAP without further antibiotics) and improvement of CAP-related symptoms (as determined by an 18-item questionnaire scored 0-90, with higher scores indicative of greater severity). Secondary outcomes included duration of antibiotic use, time to clinical improvement, mortality, hospital readmission, hospital length of stay, and CAP recurrence.1

Of the 312 participants, 162 were randomized to the intervention group and 150 to the control group. Mean age of patients in the intervention and control groups was 66.2 and 64.7, respectively. Other baseline demographics were similar between the groups. Nearly 80% of patients received quinolone treatment; < 10% received a ß-lactam plus a macrolide.1

Clinical success rates were similar for the control and intervention groups at day 10 (49% vs 56%, respectively) and day 30 (89% vs 92%). Median antibiotic treatment duration was shorter in the intervention group than in the control group (5 d vs 10 d); the intervention group also had a lower rate of 30-day hospital readmissions (1.4% vs 6.6%). There were no differences for other secondary outcomes.1

 

 

 

WHAT’S NEW

Clinical support for 2007 guidelines

This is the first study to clinically support the IDSA/ATS guidelines, which state that a five-day course of antibiotic therapy for hospitalized adults with CAP is effective and without increased risk for adverse events.

CAVEATS

Generalizability is unclear

This study focused on antibiotic duration for the treatment of CAP in hospitalized patients and mainly used quinolone antibiotics. It remains unclear if duration of therapy is as effective in the outpatient setting or when using alternative antibiotic regimens.

If patients continued to have symptoms (eg, fever or low oxygen saturation on room air) after five days of antibiotics, treatment was continued in the study. Thus, patients in real life who continue to have symptoms may need individualized therapy and may require more than five days of antibiotics.

CHALLENGES TO IMPLEMENTATION

Antibiotics end before clinical improvement

In this study, it took an average of 12 days in both groups for patients to achieve clinical improvement, and upwards of 15 to 18 days for patients to return to normal activity. Patients and providers may be dissatisfied if the treatment course ends days before clinical improvement of symptoms. This may cause prescribers to lengthen the duration of antibiotic therapy inappropriately.

ACKNOWLEDGEMENT

The PURLs Surveillance System was supported in part by Grant Number UL1RR024999 from the National Center For Research Resources, a Clinical Translational Science Award to the University of Chicago. The content is solely the responsibility of the authors and does not necessarily represent the official views of the National Center For Research Resources or the National Institutes of Health.

Copyright © 2017. The Family Physicians Inquiries Network. All rights reserved.

Reprinted with permission from the Family Physicians Inquiries Network and The Journal of Family Practice (2017; 66[10]:629-631).

 

A 65-year-old woman is admitted to your inpatient service from the family health center. She is diagnosed with community-acquired pneumonia (CAP) based on a five-day history of cough and fever and a positive chest x-ray. She now requires oxygen at rest. She has a history of hypertension and diabetes, both of which have been controlled by oral medications. Antibiotic therapy is initiated—but what treatment duration is ideal?

The World Health Organization estimates that pneumonia is the third most common cause of mortality worldwide, causing 3.2 million deaths per year.2 Appropriate prescribing of antibiotics is critical for successful treatment of CAP.

In 2007, the Infectious Diseases Society of America (IDSA) and the American Thoracic Society (ATS) created consensus guidelines for the treatment of CAP.3 These guidelines recommend a minimum five-day course of antibiotics if the patient is clinically stable (defined as afebrile for 48 h; heart rate ≤ 100 beats/min; respiratory rate ≤ 24 breaths/min-1; systolic blood pressure ≥ 90 mm Hg; oxygen saturation ≥ 90%; normal mental status; and able to tolerate oral intake). Longer antibiotic treatment durations are recommended on an individualized basis (eg, if the isolated pathogen is not susceptible to the initial antibiotic or if the infection was caused by an extrapulmonary source).

However, these recommendations are not routinely followed. Practitioners often make it their custom to prescribe longer courses of antibiotics.4 Yet, we know that there are several reasons to consider shorter courses of antibiotics, including lower health care costs, fewer adverse effects, and lower rates of bacterial resistance.5-7

Two meta-analyses were performed to compare the safety and efficacy of short- and long-course antibiotic therapy in CAP (≤ 7 d vs > 7 d, respectively).8,9 Both meta-analyses found no difference in efficacy or safety between shorter and longer courses of antibiotics for CAP. Secondary outcomes noted a trend toward decreased antibiotic-associated adverse events with shorter courses of therapy.8,9

However, there are limitations to broad implementation. Studies included in these analyses utilized a variety of antibiotic treatment regimens and longer courses (7 d vs 5 d) that are not recommended by the IDSA/ATS guidelines. Additionally, studies included both inpatient and outpatient treatment groups, so findings may not apply to an exclusively inpatient CAP population.8,9

This study sought to validate the IDSA/ATS guidelines recommending a five-day course of antibiotics for hospitalized patients with CAP.1

STUDY SUMMARY

No differences in clinical outcomes

This multicenter, double-blind, noninferiority randomized trial compared short-term antibiotic treatment duration (5 d) to physician-discretion antibiotic treatment duration among 312 patients (ages 18 and older) admitted for CAP to one of four teaching hospitals in Spain.1 Pneumonia was diagnosed on chest radiograph with at least one symptom: cough, fever, dyspnea, or chest pain. Patients were excluded if, among other things, they had an immunocompromising condition, lived in a nursing home, had a recent hospital stay, used antibiotics within the previous 30 days, or had an uncommon pathogen, such as Pseudomonas aeruginosa or Staphylococcus aureus.1

After receiving a minimum of five days of antibiotics, patients were randomly assigned to an intervention group (where, if clinically stable, no further antibiotics were given) or a control group (where physicians determined antibiotic duration).1

Primary outcomes were clinical success rate at days 10 and 30 from admission (defined as resolution of signs and symptoms of CAP without further antibiotics) and improvement of CAP-related symptoms (as determined by an 18-item questionnaire scored 0-90, with higher scores indicative of greater severity). Secondary outcomes included duration of antibiotic use, time to clinical improvement, mortality, hospital readmission, hospital length of stay, and CAP recurrence.1

Of the 312 participants, 162 were randomized to the intervention group and 150 to the control group. Mean age of patients in the intervention and control groups was 66.2 and 64.7, respectively. Other baseline demographics were similar between the groups. Nearly 80% of patients received quinolone treatment; < 10% received a ß-lactam plus a macrolide.1

Clinical success rates were similar for the control and intervention groups at day 10 (49% vs 56%, respectively) and day 30 (89% vs 92%). Median antibiotic treatment duration was shorter in the intervention group than in the control group (5 d vs 10 d); the intervention group also had a lower rate of 30-day hospital readmissions (1.4% vs 6.6%). There were no differences for other secondary outcomes.1

 

 

 

WHAT’S NEW

Clinical support for 2007 guidelines

This is the first study to clinically support the IDSA/ATS guidelines, which state that a five-day course of antibiotic therapy for hospitalized adults with CAP is effective and without increased risk for adverse events.

CAVEATS

Generalizability is unclear

This study focused on antibiotic duration for the treatment of CAP in hospitalized patients and mainly used quinolone antibiotics. It remains unclear if duration of therapy is as effective in the outpatient setting or when using alternative antibiotic regimens.

If patients continued to have symptoms (eg, fever or low oxygen saturation on room air) after five days of antibiotics, treatment was continued in the study. Thus, patients in real life who continue to have symptoms may need individualized therapy and may require more than five days of antibiotics.

CHALLENGES TO IMPLEMENTATION

Antibiotics end before clinical improvement

In this study, it took an average of 12 days in both groups for patients to achieve clinical improvement, and upwards of 15 to 18 days for patients to return to normal activity. Patients and providers may be dissatisfied if the treatment course ends days before clinical improvement of symptoms. This may cause prescribers to lengthen the duration of antibiotic therapy inappropriately.

ACKNOWLEDGEMENT

The PURLs Surveillance System was supported in part by Grant Number UL1RR024999 from the National Center For Research Resources, a Clinical Translational Science Award to the University of Chicago. The content is solely the responsibility of the authors and does not necessarily represent the official views of the National Center For Research Resources or the National Institutes of Health.

Copyright © 2017. The Family Physicians Inquiries Network. All rights reserved.

Reprinted with permission from the Family Physicians Inquiries Network and The Journal of Family Practice (2017; 66[10]:629-631).

References

1. Uranga A, España PP, Bilbao A, et al. Duration of antibiotic treatment in community-acquired pneumonia: a multicenter randomized clinical trial. JAMA Intern Med. 2016;176:1257-1265.
2. World Health Organization. The top 10 causes of death. www.who.int/mediacentre/factsheets/fs310/en/index.html. Accessed October 18, 2017.
3. Mandell LA, Wunderink RG, Anzueto A, et al. Infectious Diseases Society of America/American Thoracic Society consensus guidelines on the management of community-acquired pneumonia in adults. Clin Infect Dis. 2007;44(suppl 2):S27-S72.
4. Aliberti S, Blasi F, Zanaboni AM, et al. Duration of antibiotic therapy in hospitalised patients with community-acquired pneumonia. Eur Respir J. 2010;36:128-134.
5. Guillemot D, Carbon C, Balkau B, et al. Low dosage and long treatment duration of ß-lactam: risk factors for carriage of penicillin-resistant Streptococcus pneumoniae. JAMA. 1998; 279:365-370.
6. Opmeer BC, el Moussaoui R, Bossuyt PM, et al. Costs associated with shorter duration of antibiotic therapy in hospitalized patients with mild-to-moderate severe community-acquired pneumonia. J Antimicrob Chemother. 2007;60: 1131-1136.
7. File TM Jr. Clinical efficacy of newer agents in short-duration therapy for community-acquired pneumonia. Clin Infect Dis. 2004;39(suppl 3):S159-S164.
8. Li JZ, Winston LG, Moore DH, et al. Efficacy of short-course antibiotic regimens for community-acquired pneumonia: a meta-analysis. Am J Med. 2007;120:783-790.
9. Dimopoulos G, Matthaiou DK, Karageorgopoulos DE, et al. Short- versus long-course antibacterial therapy for community-acquired pneumonia: a meta-analysis. Drugs. 2008;68: 1841-1854.

References

1. Uranga A, España PP, Bilbao A, et al. Duration of antibiotic treatment in community-acquired pneumonia: a multicenter randomized clinical trial. JAMA Intern Med. 2016;176:1257-1265.
2. World Health Organization. The top 10 causes of death. www.who.int/mediacentre/factsheets/fs310/en/index.html. Accessed October 18, 2017.
3. Mandell LA, Wunderink RG, Anzueto A, et al. Infectious Diseases Society of America/American Thoracic Society consensus guidelines on the management of community-acquired pneumonia in adults. Clin Infect Dis. 2007;44(suppl 2):S27-S72.
4. Aliberti S, Blasi F, Zanaboni AM, et al. Duration of antibiotic therapy in hospitalised patients with community-acquired pneumonia. Eur Respir J. 2010;36:128-134.
5. Guillemot D, Carbon C, Balkau B, et al. Low dosage and long treatment duration of ß-lactam: risk factors for carriage of penicillin-resistant Streptococcus pneumoniae. JAMA. 1998; 279:365-370.
6. Opmeer BC, el Moussaoui R, Bossuyt PM, et al. Costs associated with shorter duration of antibiotic therapy in hospitalized patients with mild-to-moderate severe community-acquired pneumonia. J Antimicrob Chemother. 2007;60: 1131-1136.
7. File TM Jr. Clinical efficacy of newer agents in short-duration therapy for community-acquired pneumonia. Clin Infect Dis. 2004;39(suppl 3):S159-S164.
8. Li JZ, Winston LG, Moore DH, et al. Efficacy of short-course antibiotic regimens for community-acquired pneumonia: a meta-analysis. Am J Med. 2007;120:783-790.
9. Dimopoulos G, Matthaiou DK, Karageorgopoulos DE, et al. Short- versus long-course antibacterial therapy for community-acquired pneumonia: a meta-analysis. Drugs. 2008;68: 1841-1854.

Issue
Clinician Reviews - 27(11)
Issue
Clinician Reviews - 27(11)
Page Number
47-49
Page Number
47-49
Publications
Publications
Topics
Article Type
Display Headline
When to “CAP” Off Pneumonia Treatment
Display Headline
When to “CAP” Off Pneumonia Treatment
Sections
Disallow All Ads
Content Gating
No Gating (article Unlocked/Free)
Alternative CME
Disqus Comments
Default
Article PDF Media

When to “CAP” off treatment for pneumonia

Article Type
Changed
Tue, 06/23/2020 - 12:11
Display Headline
When to “CAP” off treatment for pneumonia
 

ILLUSTRATIVE CASE

A 65-year-old woman is admitted to your inpatient service from your family health center. She is diagnosed with community-acquired pneumonia (CAP) based on a 5-day history of cough and fever and a positive chest x-ray. She now requires oxygen at rest. She has a past medical history of hypertension and diabetes, both of which have been controlled on oral medications. Antibiotic therapy is initiated for the treatment of the pneumonia, but what treatment duration is ideal?

The World Health Organization estimates that pneumonia is the third most common cause of mortality worldwide, causing 3.2 million deaths per year.2 Appropriate prescribing of antibiotics is critical for the successful treatment of CAP.

The Infectious Diseases Society of America (IDSA) and the American Thoracic Society (ATS) created consensus guidelines, published in 2007, for the treatment of CAP.3 These guidelines recommend a minimum 5-day course of antibiotics if the patient is clinically stable, which is defined as: afebrile for 48 hours, heart rate ≤100 beats/minute, respiratory rate ≤24 respirations/minute, systolic blood pressure ≥90 mm Hg, oxygen saturation ≥90%, normal mental status, and able to tolerate oral intake. Longer antibiotic treatment durations are recommended on an individualized basis, if, for example, the isolated pathogen is not susceptible to the initial antibiotic or if the infection was caused by an extrapulmonary source.

However, these recommendations are not routinely followed. Practitioners often make it their custom to prescribe longer courses of antibiotics.4 And yet we know that there are several reasons to consider shorter courses of antibiotics, including lower health care costs, fewer adverse effects, and lower rates of bacterial resistance.5-7

Two meta-analyses were performed to compare the safety and efficacy of short- (≤7 days) vs long-course (>7 days) antibiotic therapy in CAP.8,9 Both meta-analyses found no difference in efficacy or safety between shorter and longer courses of antibiotic treatment regimens for CAP. Secondary outcomes noted a trend toward decreased antibiotic-associated adverse events with shorter courses of therapy.8,9

While these meta-analyses supported shorter courses of antibiotics for CAP, there are limitations to the broad implementation of their findings. Studies included in these analyses utilized a variety of antibiotic treatment regimens and longer courses (7 days vs 5 days) that are not recommended by the IDSA/ATS guidelines. Additionally, studies included both inpatient and outpatient treatment groups, so findings may not apply to an exclusively inpatient CAP population.8,9

This study sought to validate the IDSA/ATS guidelines recommending a 5-day course of antibiotics for hospitalized patients with CAP.1

STUDY SUMMARY

No differences in clinical outcomes between 5 days of Tx—and longer

This multicenter, double-blind, noninferiority randomized trial compared short-term antibiotic treatment duration (5 days) to physician-discretion antibiotic treatment duration among 312 patients ≥18 years of age admitted for CAP to one of 4 teaching hospitals in Spain.1 Pneumonia was diagnosed on chest radiograph with at least one symptom: cough, fever, dyspnea, or chest pain. Patients were excluded if, among other things, they had an immunocompromising condition, lived in a nursing home, had a recent hospital stay, used antibiotics within the previous 30 days, or had an uncommon pathogen, such as Pseudomonas aeruginosa or Staphylococcus aureus.1

This is the first study to support the efficacy of the 5-day course of antibiotics for hospitalized CAP patients recommended by IDSA/ATS guidelines.

Patients were randomized after receiving a minimum of 5 days of antibiotics to an intervention group (where, if clinically stable, no further antibiotics were given) or a control group (where physicians determined antibiotic duration).1 Primary outcomes were clinical success rate at Days 10 and 30 from admission, defined as resolution of signs and symptoms of CAP without further antibiotics, and improvement of CAP-related symptoms as determined by an 18-item CAP symptom questionnaire. This questionnaire was scored 0 to 90, where higher scores indicated greater severity. Secondary outcomes included: duration of antibiotic use, time to clinical improvement, mortality, hospital readmission, hospital length of stay, and CAP recurrence.1A total of 312 patients were randomized with 162 patients in the intervention group and 150 patients in the control group. The mean age of patients in the intervention and control groups was 66.2 and 64.7 years, respectively. Other baseline demographics were similar between the groups. Nearly 80% of patients received quinolone treatment; <10% received a beta-lactam plus a macrolide.1

Clinical success rates were similar for the control and intervention groups, respectively, at Day 10 (49% vs 56%; P=.18) and Day 30 (89% vs 92%; P=.33). There was shorter median treatment duration with antibiotics in the intervention group compared with the control group (5 days vs 10 days; P<.001) and fewer 30-day hospital readmissions (1.4% vs 6.6%; P=.02). There were no differences for other secondary outcomes.1

 

 

 

WHAT’S NEW

Clinical support for 2007 guidelines

This is the first study to clinically support the IDSA/ATS guidelines, which state that a 5-day course of antibiotic therapy for hospitalized adults with CAP is effective and without increased risk of adverse events.

CAVEATS

Generalizability to other meds and settings is unclear

This study focused on antibiotic duration for the treatment of CAP in hospitalized patients and mainly used quinolone antibiotics. It remains unclear if duration of therapy is as effective in the outpatient setting or when using alternative antibiotic regimens.

If patients continued to have symptoms (such as fever or low oxygen saturation on room air) after 5 days of antibiotic treatment, antibiotic treatment was continued in the study. Thus, patients in real life who continue to have symptoms may need individualized therapy and may require more than 5 days of antibiotics.

CHALLENGES TO IMPLEMENTATION

Antibiotics end before clinical improvement occurs

This study noted an average of 12 days in both groups for patients to achieve clinical improvement, with upwards of 15 to 18 days for patients to return to normal activity. Patients and providers may be dissatisfied if the treatment course ends days before clinical improvement of symptoms. This may cause prescribers to lengthen the duration of antibiotic therapy inappropriately.

ACKNOWLEDGEMENT

The PURLs Surveillance System was supported in part by Grant Number UL1RR024999 from the National Center For Research Resources, a Clinical Translational Science Award to the University of Chicago. The content is solely the responsibility of the authors and does not necessarily represent the official views of the National Center For Research Resources or the National Institutes of Health.

Files
References

1. Uranga A, España PP, Bilbao A, et al. Duration of antibiotic treatment in community-acquired pneumonia: a multicenter randomized clinical trial. JAMA Intern Med. 2016;176:1257-1265.

2. World Health Organization. The top 10 causes of death. Available at: http://www.who.int/mediacentre/factsheets/fs310/en/index.html. Accessed September 5, 2017.

3. Mandell LA, Wunderink RG, Anzueto A, et al. Infectious Diseases Society of America/American Thoracic Society consensus guidelines on the management of community-acquired pneumonia in adults. Clin Infect Dis. 2007;44(suppl 2):S27-S72.

4. Aliberti S, Blasi F, Zanaboni AM, et al. Duration of antibiotic therapy in hospitalised patients with community-acquired pneumonia. Eur Respir J. 2010;36:128-134.

5. Guillemot D, Carbon C, Balkau B, et al. Low dosage and long treatment duration of ß-lactam: risk factors for carriage of penicillin-resistant Streptococcus pneumoniae. JAMA. 1998;279:365-370.

6. Opmeer BC, el Moussaoui R, Bossuyt PM, et al. Costs associated with shorter duration of antibiotic therapy in hospitalized patients with mild-to-moderate severe community-acquired pneumonia. J Antimicrob Chemother. 2007;60:1131-1136.

7. File TM Jr. Clinical efficacy of newer agents in short-duration therapy for community-acquired pneumonia. Clin Infect Dis. 2004;39(suppl 3):S159-S164.

8. Li JZ, Winston LG, Moore DH, et al. Efficacy of short-course antibiotic regimens for community-acquired pneumonia: a meta-analysis. Am J Med. 2007;120:783-790.

9. Dimopoulos G, Matthaiou DK, Karageorgopoulos DE, et al. Short- versus long-course antibacterial therapy for community-acquired pneumonia: a meta-analysis. Drugs. 2008;68:1841-1854.

Article PDF
Author and Disclosure Information

University of Pittsburgh Medical Center St. Margaret, Pa (Dr. Castelli); University of Illinois at Chicago (Dr. Jarrett)

DEPUTY EDITOR
Corey Lyon, DO

University of Colorado Family Medicine Residency, Denver

Issue
The Journal of Family Practice - 66(10)
Publications
Topics
Page Number
629-631
Sections
Files
Files
Author and Disclosure Information

University of Pittsburgh Medical Center St. Margaret, Pa (Dr. Castelli); University of Illinois at Chicago (Dr. Jarrett)

DEPUTY EDITOR
Corey Lyon, DO

University of Colorado Family Medicine Residency, Denver

Author and Disclosure Information

University of Pittsburgh Medical Center St. Margaret, Pa (Dr. Castelli); University of Illinois at Chicago (Dr. Jarrett)

DEPUTY EDITOR
Corey Lyon, DO

University of Colorado Family Medicine Residency, Denver

Article PDF
Article PDF
 

ILLUSTRATIVE CASE

A 65-year-old woman is admitted to your inpatient service from your family health center. She is diagnosed with community-acquired pneumonia (CAP) based on a 5-day history of cough and fever and a positive chest x-ray. She now requires oxygen at rest. She has a past medical history of hypertension and diabetes, both of which have been controlled on oral medications. Antibiotic therapy is initiated for the treatment of the pneumonia, but what treatment duration is ideal?

The World Health Organization estimates that pneumonia is the third most common cause of mortality worldwide, causing 3.2 million deaths per year.2 Appropriate prescribing of antibiotics is critical for the successful treatment of CAP.

The Infectious Diseases Society of America (IDSA) and the American Thoracic Society (ATS) created consensus guidelines, published in 2007, for the treatment of CAP.3 These guidelines recommend a minimum 5-day course of antibiotics if the patient is clinically stable, which is defined as: afebrile for 48 hours, heart rate ≤100 beats/minute, respiratory rate ≤24 respirations/minute, systolic blood pressure ≥90 mm Hg, oxygen saturation ≥90%, normal mental status, and able to tolerate oral intake. Longer antibiotic treatment durations are recommended on an individualized basis, if, for example, the isolated pathogen is not susceptible to the initial antibiotic or if the infection was caused by an extrapulmonary source.

However, these recommendations are not routinely followed. Practitioners often make it their custom to prescribe longer courses of antibiotics.4 And yet we know that there are several reasons to consider shorter courses of antibiotics, including lower health care costs, fewer adverse effects, and lower rates of bacterial resistance.5-7

Two meta-analyses were performed to compare the safety and efficacy of short- (≤7 days) vs long-course (>7 days) antibiotic therapy in CAP.8,9 Both meta-analyses found no difference in efficacy or safety between shorter and longer courses of antibiotic treatment regimens for CAP. Secondary outcomes noted a trend toward decreased antibiotic-associated adverse events with shorter courses of therapy.8,9

While these meta-analyses supported shorter courses of antibiotics for CAP, there are limitations to the broad implementation of their findings. Studies included in these analyses utilized a variety of antibiotic treatment regimens and longer courses (7 days vs 5 days) that are not recommended by the IDSA/ATS guidelines. Additionally, studies included both inpatient and outpatient treatment groups, so findings may not apply to an exclusively inpatient CAP population.8,9

This study sought to validate the IDSA/ATS guidelines recommending a 5-day course of antibiotics for hospitalized patients with CAP.1

STUDY SUMMARY

No differences in clinical outcomes between 5 days of Tx—and longer

This multicenter, double-blind, noninferiority randomized trial compared short-term antibiotic treatment duration (5 days) to physician-discretion antibiotic treatment duration among 312 patients ≥18 years of age admitted for CAP to one of 4 teaching hospitals in Spain.1 Pneumonia was diagnosed on chest radiograph with at least one symptom: cough, fever, dyspnea, or chest pain. Patients were excluded if, among other things, they had an immunocompromising condition, lived in a nursing home, had a recent hospital stay, used antibiotics within the previous 30 days, or had an uncommon pathogen, such as Pseudomonas aeruginosa or Staphylococcus aureus.1

This is the first study to support the efficacy of the 5-day course of antibiotics for hospitalized CAP patients recommended by IDSA/ATS guidelines.

Patients were randomized after receiving a minimum of 5 days of antibiotics to an intervention group (where, if clinically stable, no further antibiotics were given) or a control group (where physicians determined antibiotic duration).1 Primary outcomes were clinical success rate at Days 10 and 30 from admission, defined as resolution of signs and symptoms of CAP without further antibiotics, and improvement of CAP-related symptoms as determined by an 18-item CAP symptom questionnaire. This questionnaire was scored 0 to 90, where higher scores indicated greater severity. Secondary outcomes included: duration of antibiotic use, time to clinical improvement, mortality, hospital readmission, hospital length of stay, and CAP recurrence.1A total of 312 patients were randomized with 162 patients in the intervention group and 150 patients in the control group. The mean age of patients in the intervention and control groups was 66.2 and 64.7 years, respectively. Other baseline demographics were similar between the groups. Nearly 80% of patients received quinolone treatment; <10% received a beta-lactam plus a macrolide.1

Clinical success rates were similar for the control and intervention groups, respectively, at Day 10 (49% vs 56%; P=.18) and Day 30 (89% vs 92%; P=.33). There was shorter median treatment duration with antibiotics in the intervention group compared with the control group (5 days vs 10 days; P<.001) and fewer 30-day hospital readmissions (1.4% vs 6.6%; P=.02). There were no differences for other secondary outcomes.1

 

 

 

WHAT’S NEW

Clinical support for 2007 guidelines

This is the first study to clinically support the IDSA/ATS guidelines, which state that a 5-day course of antibiotic therapy for hospitalized adults with CAP is effective and without increased risk of adverse events.

CAVEATS

Generalizability to other meds and settings is unclear

This study focused on antibiotic duration for the treatment of CAP in hospitalized patients and mainly used quinolone antibiotics. It remains unclear if duration of therapy is as effective in the outpatient setting or when using alternative antibiotic regimens.

If patients continued to have symptoms (such as fever or low oxygen saturation on room air) after 5 days of antibiotic treatment, antibiotic treatment was continued in the study. Thus, patients in real life who continue to have symptoms may need individualized therapy and may require more than 5 days of antibiotics.

CHALLENGES TO IMPLEMENTATION

Antibiotics end before clinical improvement occurs

This study noted an average of 12 days in both groups for patients to achieve clinical improvement, with upwards of 15 to 18 days for patients to return to normal activity. Patients and providers may be dissatisfied if the treatment course ends days before clinical improvement of symptoms. This may cause prescribers to lengthen the duration of antibiotic therapy inappropriately.

ACKNOWLEDGEMENT

The PURLs Surveillance System was supported in part by Grant Number UL1RR024999 from the National Center For Research Resources, a Clinical Translational Science Award to the University of Chicago. The content is solely the responsibility of the authors and does not necessarily represent the official views of the National Center For Research Resources or the National Institutes of Health.

 

ILLUSTRATIVE CASE

A 65-year-old woman is admitted to your inpatient service from your family health center. She is diagnosed with community-acquired pneumonia (CAP) based on a 5-day history of cough and fever and a positive chest x-ray. She now requires oxygen at rest. She has a past medical history of hypertension and diabetes, both of which have been controlled on oral medications. Antibiotic therapy is initiated for the treatment of the pneumonia, but what treatment duration is ideal?

The World Health Organization estimates that pneumonia is the third most common cause of mortality worldwide, causing 3.2 million deaths per year.2 Appropriate prescribing of antibiotics is critical for the successful treatment of CAP.

The Infectious Diseases Society of America (IDSA) and the American Thoracic Society (ATS) created consensus guidelines, published in 2007, for the treatment of CAP.3 These guidelines recommend a minimum 5-day course of antibiotics if the patient is clinically stable, which is defined as: afebrile for 48 hours, heart rate ≤100 beats/minute, respiratory rate ≤24 respirations/minute, systolic blood pressure ≥90 mm Hg, oxygen saturation ≥90%, normal mental status, and able to tolerate oral intake. Longer antibiotic treatment durations are recommended on an individualized basis, if, for example, the isolated pathogen is not susceptible to the initial antibiotic or if the infection was caused by an extrapulmonary source.

However, these recommendations are not routinely followed. Practitioners often make it their custom to prescribe longer courses of antibiotics.4 And yet we know that there are several reasons to consider shorter courses of antibiotics, including lower health care costs, fewer adverse effects, and lower rates of bacterial resistance.5-7

Two meta-analyses were performed to compare the safety and efficacy of short- (≤7 days) vs long-course (>7 days) antibiotic therapy in CAP.8,9 Both meta-analyses found no difference in efficacy or safety between shorter and longer courses of antibiotic treatment regimens for CAP. Secondary outcomes noted a trend toward decreased antibiotic-associated adverse events with shorter courses of therapy.8,9

While these meta-analyses supported shorter courses of antibiotics for CAP, there are limitations to the broad implementation of their findings. Studies included in these analyses utilized a variety of antibiotic treatment regimens and longer courses (7 days vs 5 days) that are not recommended by the IDSA/ATS guidelines. Additionally, studies included both inpatient and outpatient treatment groups, so findings may not apply to an exclusively inpatient CAP population.8,9

This study sought to validate the IDSA/ATS guidelines recommending a 5-day course of antibiotics for hospitalized patients with CAP.1

STUDY SUMMARY

No differences in clinical outcomes between 5 days of Tx—and longer

This multicenter, double-blind, noninferiority randomized trial compared short-term antibiotic treatment duration (5 days) to physician-discretion antibiotic treatment duration among 312 patients ≥18 years of age admitted for CAP to one of 4 teaching hospitals in Spain.1 Pneumonia was diagnosed on chest radiograph with at least one symptom: cough, fever, dyspnea, or chest pain. Patients were excluded if, among other things, they had an immunocompromising condition, lived in a nursing home, had a recent hospital stay, used antibiotics within the previous 30 days, or had an uncommon pathogen, such as Pseudomonas aeruginosa or Staphylococcus aureus.1

This is the first study to support the efficacy of the 5-day course of antibiotics for hospitalized CAP patients recommended by IDSA/ATS guidelines.

Patients were randomized after receiving a minimum of 5 days of antibiotics to an intervention group (where, if clinically stable, no further antibiotics were given) or a control group (where physicians determined antibiotic duration).1 Primary outcomes were clinical success rate at Days 10 and 30 from admission, defined as resolution of signs and symptoms of CAP without further antibiotics, and improvement of CAP-related symptoms as determined by an 18-item CAP symptom questionnaire. This questionnaire was scored 0 to 90, where higher scores indicated greater severity. Secondary outcomes included: duration of antibiotic use, time to clinical improvement, mortality, hospital readmission, hospital length of stay, and CAP recurrence.1A total of 312 patients were randomized with 162 patients in the intervention group and 150 patients in the control group. The mean age of patients in the intervention and control groups was 66.2 and 64.7 years, respectively. Other baseline demographics were similar between the groups. Nearly 80% of patients received quinolone treatment; <10% received a beta-lactam plus a macrolide.1

Clinical success rates were similar for the control and intervention groups, respectively, at Day 10 (49% vs 56%; P=.18) and Day 30 (89% vs 92%; P=.33). There was shorter median treatment duration with antibiotics in the intervention group compared with the control group (5 days vs 10 days; P<.001) and fewer 30-day hospital readmissions (1.4% vs 6.6%; P=.02). There were no differences for other secondary outcomes.1

 

 

 

WHAT’S NEW

Clinical support for 2007 guidelines

This is the first study to clinically support the IDSA/ATS guidelines, which state that a 5-day course of antibiotic therapy for hospitalized adults with CAP is effective and without increased risk of adverse events.

CAVEATS

Generalizability to other meds and settings is unclear

This study focused on antibiotic duration for the treatment of CAP in hospitalized patients and mainly used quinolone antibiotics. It remains unclear if duration of therapy is as effective in the outpatient setting or when using alternative antibiotic regimens.

If patients continued to have symptoms (such as fever or low oxygen saturation on room air) after 5 days of antibiotic treatment, antibiotic treatment was continued in the study. Thus, patients in real life who continue to have symptoms may need individualized therapy and may require more than 5 days of antibiotics.

CHALLENGES TO IMPLEMENTATION

Antibiotics end before clinical improvement occurs

This study noted an average of 12 days in both groups for patients to achieve clinical improvement, with upwards of 15 to 18 days for patients to return to normal activity. Patients and providers may be dissatisfied if the treatment course ends days before clinical improvement of symptoms. This may cause prescribers to lengthen the duration of antibiotic therapy inappropriately.

ACKNOWLEDGEMENT

The PURLs Surveillance System was supported in part by Grant Number UL1RR024999 from the National Center For Research Resources, a Clinical Translational Science Award to the University of Chicago. The content is solely the responsibility of the authors and does not necessarily represent the official views of the National Center For Research Resources or the National Institutes of Health.

References

1. Uranga A, España PP, Bilbao A, et al. Duration of antibiotic treatment in community-acquired pneumonia: a multicenter randomized clinical trial. JAMA Intern Med. 2016;176:1257-1265.

2. World Health Organization. The top 10 causes of death. Available at: http://www.who.int/mediacentre/factsheets/fs310/en/index.html. Accessed September 5, 2017.

3. Mandell LA, Wunderink RG, Anzueto A, et al. Infectious Diseases Society of America/American Thoracic Society consensus guidelines on the management of community-acquired pneumonia in adults. Clin Infect Dis. 2007;44(suppl 2):S27-S72.

4. Aliberti S, Blasi F, Zanaboni AM, et al. Duration of antibiotic therapy in hospitalised patients with community-acquired pneumonia. Eur Respir J. 2010;36:128-134.

5. Guillemot D, Carbon C, Balkau B, et al. Low dosage and long treatment duration of ß-lactam: risk factors for carriage of penicillin-resistant Streptococcus pneumoniae. JAMA. 1998;279:365-370.

6. Opmeer BC, el Moussaoui R, Bossuyt PM, et al. Costs associated with shorter duration of antibiotic therapy in hospitalized patients with mild-to-moderate severe community-acquired pneumonia. J Antimicrob Chemother. 2007;60:1131-1136.

7. File TM Jr. Clinical efficacy of newer agents in short-duration therapy for community-acquired pneumonia. Clin Infect Dis. 2004;39(suppl 3):S159-S164.

8. Li JZ, Winston LG, Moore DH, et al. Efficacy of short-course antibiotic regimens for community-acquired pneumonia: a meta-analysis. Am J Med. 2007;120:783-790.

9. Dimopoulos G, Matthaiou DK, Karageorgopoulos DE, et al. Short- versus long-course antibacterial therapy for community-acquired pneumonia: a meta-analysis. Drugs. 2008;68:1841-1854.

References

1. Uranga A, España PP, Bilbao A, et al. Duration of antibiotic treatment in community-acquired pneumonia: a multicenter randomized clinical trial. JAMA Intern Med. 2016;176:1257-1265.

2. World Health Organization. The top 10 causes of death. Available at: http://www.who.int/mediacentre/factsheets/fs310/en/index.html. Accessed September 5, 2017.

3. Mandell LA, Wunderink RG, Anzueto A, et al. Infectious Diseases Society of America/American Thoracic Society consensus guidelines on the management of community-acquired pneumonia in adults. Clin Infect Dis. 2007;44(suppl 2):S27-S72.

4. Aliberti S, Blasi F, Zanaboni AM, et al. Duration of antibiotic therapy in hospitalised patients with community-acquired pneumonia. Eur Respir J. 2010;36:128-134.

5. Guillemot D, Carbon C, Balkau B, et al. Low dosage and long treatment duration of ß-lactam: risk factors for carriage of penicillin-resistant Streptococcus pneumoniae. JAMA. 1998;279:365-370.

6. Opmeer BC, el Moussaoui R, Bossuyt PM, et al. Costs associated with shorter duration of antibiotic therapy in hospitalized patients with mild-to-moderate severe community-acquired pneumonia. J Antimicrob Chemother. 2007;60:1131-1136.

7. File TM Jr. Clinical efficacy of newer agents in short-duration therapy for community-acquired pneumonia. Clin Infect Dis. 2004;39(suppl 3):S159-S164.

8. Li JZ, Winston LG, Moore DH, et al. Efficacy of short-course antibiotic regimens for community-acquired pneumonia: a meta-analysis. Am J Med. 2007;120:783-790.

9. Dimopoulos G, Matthaiou DK, Karageorgopoulos DE, et al. Short- versus long-course antibacterial therapy for community-acquired pneumonia: a meta-analysis. Drugs. 2008;68:1841-1854.

Issue
The Journal of Family Practice - 66(10)
Issue
The Journal of Family Practice - 66(10)
Page Number
629-631
Page Number
629-631
Publications
Publications
Topics
Article Type
Display Headline
When to “CAP” off treatment for pneumonia
Display Headline
When to “CAP” off treatment for pneumonia
Sections
PURLs Copyright

Copyright © 2017. The Family Physicians Inquiries Network. All rights reserved.

Inside the Article

PRACTICE CHANGER

Prescribe 5 days of antibiotic treatment for inpatients with community-acquired pneumonia because it produces the same clinical success rates as longer treatment regimens, but is associated with fewer negative patient outcomes.1

STRENGTH OF RECOMMENDATION

B: Based on a single, good-quality randomized control trial.

Uranga A, España PP, Bilbao A, et al. Duration of antibiotic treatment in community-acquired pneumonia: a multicenter randomized clinical trial. JAMA Intern Med. 2016;176:1257-1265.1

Disallow All Ads
Content Gating
No Gating (article Unlocked/Free)
Alternative CME
PubMed ID
28991939
Disqus Comments
Default
Use ProPublica
Hide sidebar & use full width
render the right sidebar.
Conference Recap Checkbox
Not Conference Recap
Clinical Edge
Article PDF Media
Media Files

Oral Agent Offers Relief From Generalized Hyperhidrosis

Article Type
Changed
Sat, 07/01/2017 - 02:40
Display Headline
Oral Agent Offers Relief From Generalized Hyperhidrosis

 

A 34-year-old woman presents to your office for unbearable sweating on her hands, face, and axillary regions. It occurs nearly daily, causing social embarrassment. She has tried multiple antiperspirants to no avail. What can she can take to reduce the sweating?

Hyperhidrosis is a common, self-limiting problem that affects 2% to 3% of the United States population.2 Patients may complain of localized sweating of the hands, feet, face, or underarms, or more systemic, generalized sweating in multiple locations. Either way, patients note a significant impact on their quality of life.

Treatment of hyperhidrosis has traditionally focused on topical therapies to the affected areas. Research by both subjective report and objective measurements has shown that antiperspirants containing aluminum salt are effective at reducing sweating, particularly in the axilla, hands, and feet.3,4 Additionally, a systematic review of observational and experimental studies found topical glycopyrrolate to be efficacious for craniofacial hyperhidrosis, with minimal adverse effects.5 The availability of low-cost prescription and OTC aluminum-based antiperspirant agents makes topicals the firstline choice.

More invasive treatments are available for hyperhidrosis refractory to topicals. In a double-blind RCT, researchers injected either botulinum toxin type A (BTX-A) 50 U or placebo in patients with bilateral primary axillary hyperhidrosis.6 Of the 207 patients who received treatment injections, 96.1% had at least a 50% reduction of axillary sweating four weeks after initial injection, as measured by gravimetric assessment. The BTX-A injections also produced a prolonged effect; mean duration between injections was 30.6 weeks.

Other invasive treatments include iontophoresis, surgery, and laser therapy; however, these methods are not suitable for body-wide application and are thus not appropriate for patients with generalized hyperhidrosis.

Oxybutynin is the first oral agent to emerge as a treatment option for hyperhidrosis. This cholinergic antagonist has historically been used to treat overactive bladder. But oxybutynin not only reduces urinary frequency, it also decreases secretions in various locations and can therefore reduce perspiration and cause dry mouth.

In one prospective placebo-controlled trial, 50 patients with generalized hyperhidrosis were randomly assigned to either oxybutynin (titrated from 2.5 mg orally once daily to 5 mg orally twice daily) or placebo for six weeks.7 Seventeen patients (73.9%) receiving oxybutynin for palmar or axillary hyperhidrosis reported moderate to “great” resolution of their symptoms, compared with six patients (27.3%) in the placebo group. Dry mouth was reported in 34.8% of patients receiving oxybutynin versus 9.1% of those who received placebo; however, no patients dropped out of the study due to this adverse effect.7

STUDY SUMMARY

This multicenter RCT compared oxybutynin to placebo in 62 adults with localized or generalized hyperhidrosis from 12 outpatient dermatology practices in France. It is the first study to include patients with both localized and generalized forms of the condition.

Patients were included if they were older than 18, enrolled in the National Health Insurance system in France, and reported a Hyperhidrosis Disease Severity Scale (HDSS) score ≥ 2. The HDSS is a validated, one-question tool (“How would you rate the severity of your sweating?”). Patients provide a score of 1 (no perceptible sweating and no interference with everyday life) to 4 (intolerable sweating with constant interference with everyday life).8 Patients were excluded if they had any contraindications to the use of an anticholinergic medication.

Patients randomly assigned to oxybutynin took 2.5 mg/d by mouth initially and increased gradually over eight days until reaching an effective dose that was no more than 7.5 mg/d. They then continued at that dose for six weeks.

The primary outcome was improvement on the HDSS by one or more points, measured at the beginning of the trial and again at six weeks. Secondary outcomes included change in quality of life, as measured by the Dermatology Life Quality Index (DLQI) and reported adverse effects. The DLQI is a dermatology-specific quality-of-life measure consisting of 10 questions. Scores range from 0 (where disease has no impact on quality of life) to 30 (maximum impact of disease on quality of life).9

Improved HDSS and DLQI scores. Most patients (83%) in the study had generalized hyperhidrosis. Patients were in their mid-30s. Sixty percent of patients in the oxybutynin group had an improvement of one point or more on the 4-point HDSS, compared to 27% in the placebo group. DLQI scores improved by 6.9 points in the oxybutynin group and 2.3 points in the placebo group.

The most common adverse effect was dry mouth, which occurred in 13 patients (43%) in the oxybutynin group and in three patients (11%) in the placebo group; it did not cause any patients to drop out of the study. The second most common adverse effect was blurred vision, which only occurred in the oxybutynin group (four patients; 13%).

 

 

 

WHAT’S NEW

This is the first RCT to demonstrate the efficacy of an oral agent for generalized primary hyperhidrosis. This trial used a relatively low dose of oxybutynin, which produced significant benefit while minimizing anticholinergic adverse effects.

CAVEATS

There are many situations for which anticholinergic medications are inappropriate, including use by geriatric patients and those with gastrointestinal disorders, urinary retention, or glaucoma.

CHALLENGES TO IMPLEMENTATION

Few, if any, challenges exist to the utilization of oxybutynin; inexpensive generic versions are widely available.

ACKNOWLEDGEMENT

The PURLs Surveillance System was supported in part by Grant Number UL1RR024999 from the National Center for Research Resources, a Clinical Translational Science Award to the University of Chicago. The content is solely the responsibility of the authors and does not necessarily represent the official views of the National Center for Research Resources or the National Institutes of Health.

Copyright © 2017. The Family Physicians Inquiries Network. All rights reserved.

Reprinted with permission from the Family Physicians Inquires Network and The Journal of Family Practice (2017;66[6]:392-394).

References

1. Schollhammer M, Brenaut E, Menard-Andivot N, et al. Oxybutynin as a treatment for generalized hyperhidrosis: a randomized, placebo-controlled trial. Br J Dermatol. 2015; 173:1163-1168.
2. Grabell DA, Hebert AA. Current and emerging medical therapies for primary hyperhidrosis. Dermatol Ther (Heidelb). 2017;7:25-36.
3. Innocenzi D, Lupi F, Bruni F, et al. Efficacy of a new aluminium salt thermophobic foam in the treatment of axillary and palmar primary hyperhidrosis: a pilot exploratory trial. Curr Med Res Opin. 2005;21:1949-1953.
4. Goh CL. Aluminum chloride hexahydrate versus palmar hyperhidrosis. Evaporimeter assessment. Int J Dermatol. 1990;29:368-370.
5. Nicholas R, Quddus A, Baker DM. Treatment of primary craniofacial hyperhidrosis: a systematic review. Am J Clin Dermatol. 2015;16:361-370.
6. Naumann M, Lowe NJ, Kumar CR, et al. Botulinum toxin type A is a safe and effective treatment for axillary hyperhidrosis over 16 months: a prospective study. Arch Dermatol. 2003; 139:731-736.
7. Wolosker N, de Campos JR, Kauffman P, et al. A randomized placebo-controlled trial of oxybutynin for the initial treatment of palmar and axillary hyperhidrosis. J Vasc Surg. 2012; 55:1696-1700.
8. Varella AY, Fukuda JM, Teivelis MP, et al. Translation and validation of Hyperhidrosis Disease Severity Scale. Rev Assoc Med Bras. 2016;62:843-847.
9. Finlay AY, Khan GK. Dermatology Life Quality Index (DLQI)—a simple practical measure for routine clinical use. Clin Exp Dermatol. 1994;19:210-216.

Article PDF
Author and Disclosure Information

Jennie B. Jarrett is with the University of Chicago. David Moss is with Nellis Air Force Base Family Medicine Residency in Las Vegas.

Issue
Clinician Reviews - 27(7)
Publications
Topics
Page Number
24,30-31
Sections
Author and Disclosure Information

Jennie B. Jarrett is with the University of Chicago. David Moss is with Nellis Air Force Base Family Medicine Residency in Las Vegas.

Author and Disclosure Information

Jennie B. Jarrett is with the University of Chicago. David Moss is with Nellis Air Force Base Family Medicine Residency in Las Vegas.

Article PDF
Article PDF
Related Articles

 

A 34-year-old woman presents to your office for unbearable sweating on her hands, face, and axillary regions. It occurs nearly daily, causing social embarrassment. She has tried multiple antiperspirants to no avail. What can she can take to reduce the sweating?

Hyperhidrosis is a common, self-limiting problem that affects 2% to 3% of the United States population.2 Patients may complain of localized sweating of the hands, feet, face, or underarms, or more systemic, generalized sweating in multiple locations. Either way, patients note a significant impact on their quality of life.

Treatment of hyperhidrosis has traditionally focused on topical therapies to the affected areas. Research by both subjective report and objective measurements has shown that antiperspirants containing aluminum salt are effective at reducing sweating, particularly in the axilla, hands, and feet.3,4 Additionally, a systematic review of observational and experimental studies found topical glycopyrrolate to be efficacious for craniofacial hyperhidrosis, with minimal adverse effects.5 The availability of low-cost prescription and OTC aluminum-based antiperspirant agents makes topicals the firstline choice.

More invasive treatments are available for hyperhidrosis refractory to topicals. In a double-blind RCT, researchers injected either botulinum toxin type A (BTX-A) 50 U or placebo in patients with bilateral primary axillary hyperhidrosis.6 Of the 207 patients who received treatment injections, 96.1% had at least a 50% reduction of axillary sweating four weeks after initial injection, as measured by gravimetric assessment. The BTX-A injections also produced a prolonged effect; mean duration between injections was 30.6 weeks.

Other invasive treatments include iontophoresis, surgery, and laser therapy; however, these methods are not suitable for body-wide application and are thus not appropriate for patients with generalized hyperhidrosis.

Oxybutynin is the first oral agent to emerge as a treatment option for hyperhidrosis. This cholinergic antagonist has historically been used to treat overactive bladder. But oxybutynin not only reduces urinary frequency, it also decreases secretions in various locations and can therefore reduce perspiration and cause dry mouth.

In one prospective placebo-controlled trial, 50 patients with generalized hyperhidrosis were randomly assigned to either oxybutynin (titrated from 2.5 mg orally once daily to 5 mg orally twice daily) or placebo for six weeks.7 Seventeen patients (73.9%) receiving oxybutynin for palmar or axillary hyperhidrosis reported moderate to “great” resolution of their symptoms, compared with six patients (27.3%) in the placebo group. Dry mouth was reported in 34.8% of patients receiving oxybutynin versus 9.1% of those who received placebo; however, no patients dropped out of the study due to this adverse effect.7

STUDY SUMMARY

This multicenter RCT compared oxybutynin to placebo in 62 adults with localized or generalized hyperhidrosis from 12 outpatient dermatology practices in France. It is the first study to include patients with both localized and generalized forms of the condition.

Patients were included if they were older than 18, enrolled in the National Health Insurance system in France, and reported a Hyperhidrosis Disease Severity Scale (HDSS) score ≥ 2. The HDSS is a validated, one-question tool (“How would you rate the severity of your sweating?”). Patients provide a score of 1 (no perceptible sweating and no interference with everyday life) to 4 (intolerable sweating with constant interference with everyday life).8 Patients were excluded if they had any contraindications to the use of an anticholinergic medication.

Patients randomly assigned to oxybutynin took 2.5 mg/d by mouth initially and increased gradually over eight days until reaching an effective dose that was no more than 7.5 mg/d. They then continued at that dose for six weeks.

The primary outcome was improvement on the HDSS by one or more points, measured at the beginning of the trial and again at six weeks. Secondary outcomes included change in quality of life, as measured by the Dermatology Life Quality Index (DLQI) and reported adverse effects. The DLQI is a dermatology-specific quality-of-life measure consisting of 10 questions. Scores range from 0 (where disease has no impact on quality of life) to 30 (maximum impact of disease on quality of life).9

Improved HDSS and DLQI scores. Most patients (83%) in the study had generalized hyperhidrosis. Patients were in their mid-30s. Sixty percent of patients in the oxybutynin group had an improvement of one point or more on the 4-point HDSS, compared to 27% in the placebo group. DLQI scores improved by 6.9 points in the oxybutynin group and 2.3 points in the placebo group.

The most common adverse effect was dry mouth, which occurred in 13 patients (43%) in the oxybutynin group and in three patients (11%) in the placebo group; it did not cause any patients to drop out of the study. The second most common adverse effect was blurred vision, which only occurred in the oxybutynin group (four patients; 13%).

 

 

 

WHAT’S NEW

This is the first RCT to demonstrate the efficacy of an oral agent for generalized primary hyperhidrosis. This trial used a relatively low dose of oxybutynin, which produced significant benefit while minimizing anticholinergic adverse effects.

CAVEATS

There are many situations for which anticholinergic medications are inappropriate, including use by geriatric patients and those with gastrointestinal disorders, urinary retention, or glaucoma.

CHALLENGES TO IMPLEMENTATION

Few, if any, challenges exist to the utilization of oxybutynin; inexpensive generic versions are widely available.

ACKNOWLEDGEMENT

The PURLs Surveillance System was supported in part by Grant Number UL1RR024999 from the National Center for Research Resources, a Clinical Translational Science Award to the University of Chicago. The content is solely the responsibility of the authors and does not necessarily represent the official views of the National Center for Research Resources or the National Institutes of Health.

Copyright © 2017. The Family Physicians Inquiries Network. All rights reserved.

Reprinted with permission from the Family Physicians Inquires Network and The Journal of Family Practice (2017;66[6]:392-394).

 

A 34-year-old woman presents to your office for unbearable sweating on her hands, face, and axillary regions. It occurs nearly daily, causing social embarrassment. She has tried multiple antiperspirants to no avail. What can she can take to reduce the sweating?

Hyperhidrosis is a common, self-limiting problem that affects 2% to 3% of the United States population.2 Patients may complain of localized sweating of the hands, feet, face, or underarms, or more systemic, generalized sweating in multiple locations. Either way, patients note a significant impact on their quality of life.

Treatment of hyperhidrosis has traditionally focused on topical therapies to the affected areas. Research by both subjective report and objective measurements has shown that antiperspirants containing aluminum salt are effective at reducing sweating, particularly in the axilla, hands, and feet.3,4 Additionally, a systematic review of observational and experimental studies found topical glycopyrrolate to be efficacious for craniofacial hyperhidrosis, with minimal adverse effects.5 The availability of low-cost prescription and OTC aluminum-based antiperspirant agents makes topicals the firstline choice.

More invasive treatments are available for hyperhidrosis refractory to topicals. In a double-blind RCT, researchers injected either botulinum toxin type A (BTX-A) 50 U or placebo in patients with bilateral primary axillary hyperhidrosis.6 Of the 207 patients who received treatment injections, 96.1% had at least a 50% reduction of axillary sweating four weeks after initial injection, as measured by gravimetric assessment. The BTX-A injections also produced a prolonged effect; mean duration between injections was 30.6 weeks.

Other invasive treatments include iontophoresis, surgery, and laser therapy; however, these methods are not suitable for body-wide application and are thus not appropriate for patients with generalized hyperhidrosis.

Oxybutynin is the first oral agent to emerge as a treatment option for hyperhidrosis. This cholinergic antagonist has historically been used to treat overactive bladder. But oxybutynin not only reduces urinary frequency, it also decreases secretions in various locations and can therefore reduce perspiration and cause dry mouth.

In one prospective placebo-controlled trial, 50 patients with generalized hyperhidrosis were randomly assigned to either oxybutynin (titrated from 2.5 mg orally once daily to 5 mg orally twice daily) or placebo for six weeks.7 Seventeen patients (73.9%) receiving oxybutynin for palmar or axillary hyperhidrosis reported moderate to “great” resolution of their symptoms, compared with six patients (27.3%) in the placebo group. Dry mouth was reported in 34.8% of patients receiving oxybutynin versus 9.1% of those who received placebo; however, no patients dropped out of the study due to this adverse effect.7

STUDY SUMMARY

This multicenter RCT compared oxybutynin to placebo in 62 adults with localized or generalized hyperhidrosis from 12 outpatient dermatology practices in France. It is the first study to include patients with both localized and generalized forms of the condition.

Patients were included if they were older than 18, enrolled in the National Health Insurance system in France, and reported a Hyperhidrosis Disease Severity Scale (HDSS) score ≥ 2. The HDSS is a validated, one-question tool (“How would you rate the severity of your sweating?”). Patients provide a score of 1 (no perceptible sweating and no interference with everyday life) to 4 (intolerable sweating with constant interference with everyday life).8 Patients were excluded if they had any contraindications to the use of an anticholinergic medication.

Patients randomly assigned to oxybutynin took 2.5 mg/d by mouth initially and increased gradually over eight days until reaching an effective dose that was no more than 7.5 mg/d. They then continued at that dose for six weeks.

The primary outcome was improvement on the HDSS by one or more points, measured at the beginning of the trial and again at six weeks. Secondary outcomes included change in quality of life, as measured by the Dermatology Life Quality Index (DLQI) and reported adverse effects. The DLQI is a dermatology-specific quality-of-life measure consisting of 10 questions. Scores range from 0 (where disease has no impact on quality of life) to 30 (maximum impact of disease on quality of life).9

Improved HDSS and DLQI scores. Most patients (83%) in the study had generalized hyperhidrosis. Patients were in their mid-30s. Sixty percent of patients in the oxybutynin group had an improvement of one point or more on the 4-point HDSS, compared to 27% in the placebo group. DLQI scores improved by 6.9 points in the oxybutynin group and 2.3 points in the placebo group.

The most common adverse effect was dry mouth, which occurred in 13 patients (43%) in the oxybutynin group and in three patients (11%) in the placebo group; it did not cause any patients to drop out of the study. The second most common adverse effect was blurred vision, which only occurred in the oxybutynin group (four patients; 13%).

 

 

 

WHAT’S NEW

This is the first RCT to demonstrate the efficacy of an oral agent for generalized primary hyperhidrosis. This trial used a relatively low dose of oxybutynin, which produced significant benefit while minimizing anticholinergic adverse effects.

CAVEATS

There are many situations for which anticholinergic medications are inappropriate, including use by geriatric patients and those with gastrointestinal disorders, urinary retention, or glaucoma.

CHALLENGES TO IMPLEMENTATION

Few, if any, challenges exist to the utilization of oxybutynin; inexpensive generic versions are widely available.

ACKNOWLEDGEMENT

The PURLs Surveillance System was supported in part by Grant Number UL1RR024999 from the National Center for Research Resources, a Clinical Translational Science Award to the University of Chicago. The content is solely the responsibility of the authors and does not necessarily represent the official views of the National Center for Research Resources or the National Institutes of Health.

Copyright © 2017. The Family Physicians Inquiries Network. All rights reserved.

Reprinted with permission from the Family Physicians Inquires Network and The Journal of Family Practice (2017;66[6]:392-394).

References

1. Schollhammer M, Brenaut E, Menard-Andivot N, et al. Oxybutynin as a treatment for generalized hyperhidrosis: a randomized, placebo-controlled trial. Br J Dermatol. 2015; 173:1163-1168.
2. Grabell DA, Hebert AA. Current and emerging medical therapies for primary hyperhidrosis. Dermatol Ther (Heidelb). 2017;7:25-36.
3. Innocenzi D, Lupi F, Bruni F, et al. Efficacy of a new aluminium salt thermophobic foam in the treatment of axillary and palmar primary hyperhidrosis: a pilot exploratory trial. Curr Med Res Opin. 2005;21:1949-1953.
4. Goh CL. Aluminum chloride hexahydrate versus palmar hyperhidrosis. Evaporimeter assessment. Int J Dermatol. 1990;29:368-370.
5. Nicholas R, Quddus A, Baker DM. Treatment of primary craniofacial hyperhidrosis: a systematic review. Am J Clin Dermatol. 2015;16:361-370.
6. Naumann M, Lowe NJ, Kumar CR, et al. Botulinum toxin type A is a safe and effective treatment for axillary hyperhidrosis over 16 months: a prospective study. Arch Dermatol. 2003; 139:731-736.
7. Wolosker N, de Campos JR, Kauffman P, et al. A randomized placebo-controlled trial of oxybutynin for the initial treatment of palmar and axillary hyperhidrosis. J Vasc Surg. 2012; 55:1696-1700.
8. Varella AY, Fukuda JM, Teivelis MP, et al. Translation and validation of Hyperhidrosis Disease Severity Scale. Rev Assoc Med Bras. 2016;62:843-847.
9. Finlay AY, Khan GK. Dermatology Life Quality Index (DLQI)—a simple practical measure for routine clinical use. Clin Exp Dermatol. 1994;19:210-216.

References

1. Schollhammer M, Brenaut E, Menard-Andivot N, et al. Oxybutynin as a treatment for generalized hyperhidrosis: a randomized, placebo-controlled trial. Br J Dermatol. 2015; 173:1163-1168.
2. Grabell DA, Hebert AA. Current and emerging medical therapies for primary hyperhidrosis. Dermatol Ther (Heidelb). 2017;7:25-36.
3. Innocenzi D, Lupi F, Bruni F, et al. Efficacy of a new aluminium salt thermophobic foam in the treatment of axillary and palmar primary hyperhidrosis: a pilot exploratory trial. Curr Med Res Opin. 2005;21:1949-1953.
4. Goh CL. Aluminum chloride hexahydrate versus palmar hyperhidrosis. Evaporimeter assessment. Int J Dermatol. 1990;29:368-370.
5. Nicholas R, Quddus A, Baker DM. Treatment of primary craniofacial hyperhidrosis: a systematic review. Am J Clin Dermatol. 2015;16:361-370.
6. Naumann M, Lowe NJ, Kumar CR, et al. Botulinum toxin type A is a safe and effective treatment for axillary hyperhidrosis over 16 months: a prospective study. Arch Dermatol. 2003; 139:731-736.
7. Wolosker N, de Campos JR, Kauffman P, et al. A randomized placebo-controlled trial of oxybutynin for the initial treatment of palmar and axillary hyperhidrosis. J Vasc Surg. 2012; 55:1696-1700.
8. Varella AY, Fukuda JM, Teivelis MP, et al. Translation and validation of Hyperhidrosis Disease Severity Scale. Rev Assoc Med Bras. 2016;62:843-847.
9. Finlay AY, Khan GK. Dermatology Life Quality Index (DLQI)—a simple practical measure for routine clinical use. Clin Exp Dermatol. 1994;19:210-216.

Issue
Clinician Reviews - 27(7)
Issue
Clinician Reviews - 27(7)
Page Number
24,30-31
Page Number
24,30-31
Publications
Publications
Topics
Article Type
Display Headline
Oral Agent Offers Relief From Generalized Hyperhidrosis
Display Headline
Oral Agent Offers Relief From Generalized Hyperhidrosis
Sections
Disallow All Ads
Content Gating
No Gating (article Unlocked/Free)
Alternative CME
Article PDF Media

Oral agent offers relief from generalized hyperhidrosis

Article Type
Changed
Tue, 06/23/2020 - 12:37
Display Headline
Oral agent offers relief from generalized hyperhidrosis
 

ILLUSTRATIVE CASE

A 34-year-old woman presents to your office for unbearable sweating. She notes that the sweating occurs nearly daily on her hands, face, and in her axillary regions, causing social embarrassment. She has tried multiple antiperspirants to no avail. Is there anything she can take to reduce the sweating?

Hyperhidrosis is a common, self-limiting problem affecting 2% to 3% of the population in the United States.2 Patients may complain of localized sweating of the hands, feet, face, or underarms or more systemic, generalized sweating in multiple locations. Either way, patients always note a significant impact on their quality of life.

Treatment of hyperhidrosis has traditionally focused on topical therapies to the affected areas. Research has shown that localized treatment with antiperspirants containing aluminum salt is effective by both subjective report and objective measurements at reducing sweating—particularly in the axilla, hands, and feet.3,4 Additionally, a systematic review of observational and experimental studies found topical glycopyrrolate to be efficacious for craniofacial hyperhidrosis with minimal adverse effects.5 The availability of low-cost prescription and over-the-counter aluminum-based antiperspirant agents makes topicals the first-line choice.

More invasive treatments are available for hyperhidrosis that is refractory to topicals. In a double-blind, randomized controlled trial, researchers injected either botulinum toxin type A (BTX-A) 50 U or placebo in patients with bilateral primary axillary hyperhidrosis.6 Of the 207 patients who received treatment injections, 96.1% had at least a 50% reduction of axillary sweating at 4 weeks after one injection, as measured by gravimetric assessment. The BTX-A injections also produced a prolonged effect; mean duration between injections was 30.6 weeks.

Other invasive treatments include iontophoresis, surgery, and laser therapy; however, these methods are not suitable for body-wide application and are, thus, not appropriate for patients with generalized hyperhidrosis.

Oxybutynin is the first oral agent to emerge as a treatment option for hyperhidrosis. This cholinergic antagonist had historically been used to treat overactive bladder. As a cholinergic antagonist, oxybutynin not only reduces urinary frequency, but also decreases secretions in various locations and, thus, can cause dry mouth and reduce perspiration.

In one prospective placebo-controlled trial, 50 patients with generalized hyperhidrosis were randomized to receive either oxybutynin titrated from 2.5 mg orally once daily to 5 mg orally twice daily or placebo for 6 weeks.7 Seventeen (73.9%) patients receiving oxybutynin for palmar or axillary hyperhidrosis reported moderate to “great” resolution of their symptoms compared with 6 (27.3%) patients in the placebo group. Dry mouth was reported in 34.8% of patients receiving oxybutynin vs 9.1% of those who received placebo (P=.038); however, no patients dropped out of the study due to this adverse effect.7

STUDY SUMMARY

This multicenter, randomized controlled trial compared oxybutynin to placebo in 62 adults with localized or generalized hyperhidrosis from 12 outpatient dermatology practices in France. It is the first study to include patients with a localized, as well as a generalized form of the condition.

Patients were included if they were >18 years of age, enrolled in the National Health Insurance system in France, and reported a Hyperhidrosis Disease Severity Scale (HDSS) score ≥2. The HDSS is a validated, one-question tool (“How would you rate the severity of your sweating?”). Patients provide a score of 1 (no perceptible sweating and no interference with everyday life) to 4 (intolerable sweating with constant interference with everyday life).8 Patients were excluded if they had any contraindications to the use of an anticholinergic medication.

This trial used a relatively low dose of oxybutynin, which produced significant benefit while minimizing anticholinergic adverse effects.

Patients randomized to oxybutynin took 2.5 mg/d orally initially and increased gradually over 8 days until reaching an effective dose that was not more than 7.5 mg/d. They then continued at that dose for 6 weeks. The primary outcome was improvement on the HDSS by one or more points measured at the beginning of the trial and at 6 weeks. Secondary outcomes included change in quality of life, as measured by the Dermatology Life Quality Index (DLQI) and reported adverse effects. The DLQI is a dermatology-specific quality-of-life measure consisting of 10 questions. Scores range from 0 (where their disease has no impact on their quality of life) to 30 (maximum impact of their disease on their quality of life).9

Improved HDSS and DLQI scores. Most patients (83%) in the study had generalized hyperhidrosis. Patients were in their mid-thirties. Sixty percent of the patients in the oxybutynin group had an improvement of one point or more on the 4-point HDSS compared to 27% in the placebo group (P<.01). DLQI scores improved by 6.9 points in the oxybutynin group and 2.3 points in the placebo group (P<.01).

The most common adverse effect was dry mouth, which occurred in 13 patients (43%) in the oxybutynin group and in 3 patients (11%) in the placebo group (P<.01); it did not cause any patients to drop out of the study. The second most common adverse effect was blurred vision, which only occurred in the oxybutynin group (4 patients; 13%).

 

 

 

WHAT'S NEW

This is the first randomized controlled trial to demonstrate the efficacy of an oral agent for generalized primary hyperhidrosis. This trial used a relatively low dose of oxybutynin, which produced significant benefit while minimizing anticholinergic adverse effects.

CAVEATS

There are many situations for which anticholinergic medications are inappropriate, including use by geriatric patients and those with gastrointestinal disorders, urinary retention, or glaucoma.

CHALLENGES TO IMPLEMENTATION

Few if any challenges exist to the utilization of oxybutynin; inexpensive generic versions are widely available.

ACKNOWLEDGEMENT

The PURLs Surveillance System was supported in part by Grant Number UL1RR024999 from the National Center For Research Resources, a Clinical Translational Science Award to the University of Chicago. The content is solely the responsibility of the authors and does not necessarily represent the official views of the National Center For Research Resources or the National Institutes of Health.

Files
References

1. Schollhammer M, Brenaut E, Menard-Andivot N, et al. Oxybutynin as a treatment for generalized hyperhidrosis: a randomized, placebo-controlled trial. Br J Dermatol. 2015;173:1163-1168.

2. Grabell DA, Hebert AA. Current and emerging medical therapies for primary hyperhidrosis. Dermatol Ther (Heidelb). 2017;7:25-36.

3. Innocenzi D, Lupi F, Bruni F, et al. Efficacy of a new aluminium salt thermophobic foam in the treatment of axillary and palmar primary hyperhidrosis: a pilot exploratory trial. Curr Med Res Opin. 2005;21:1949-1953.

4. Goh CL. Aluminum chloride hexahydrate versus palmar hyperhidrosis. Evaporimeter assessment. Int J Dermatol. 1990;29:368-370.

5. Nicholas R, Quddus A, Baker DM. Treatment of primary craniofacial hyperhidrosis: a systematic review. Am J Clin Dermatol. 2015;16:361-370.

6. Naumann M, Lowe NJ, Kumar CR, et al. Botulinum toxin type a is a safe and effective treatment for axillary hyperhidrosis over 16 months: a prospective study. Arch Dermatol. 2003;139:731-736.

7. Wolosker N, de Campos JR, Kauffman P, et al. A randomized placebo-controlled trial of oxybutynin for the initial treatment of palmar and axillary hyperhidrosis. J Vasc Surg. 2012;55:1696-1700.

8. Varella AY, Fukuda JM, Teivelis MP, et al. Translation and validation of Hyperhidrosis Disease Severity Scale. Rev Assoc Med Bras. 2016;62:843-847.

9. Finlay AY, Khan GK. Dermatology Life Quality Index (DLQI)—a simple practical measure for routine clinical use. Clin Exp Dermatol. 1994;19:210-216.

Article PDF
Author and Disclosure Information

University of Illinois at Chicago (Dr. Jarrett); Nellis Air Force Base Family Medicine Residency, Las Vegas, Nev (Dr. Moss)

DEPUTY EDITOR
Anne Mounsey, MD

Department of Family Medicine, University of North Carolina, Chapel Hill

Issue
The Journal of Family Practice - 66(6)
Publications
Topics
Page Number
392-394
Sections
Files
Files
Author and Disclosure Information

University of Illinois at Chicago (Dr. Jarrett); Nellis Air Force Base Family Medicine Residency, Las Vegas, Nev (Dr. Moss)

DEPUTY EDITOR
Anne Mounsey, MD

Department of Family Medicine, University of North Carolina, Chapel Hill

Author and Disclosure Information

University of Illinois at Chicago (Dr. Jarrett); Nellis Air Force Base Family Medicine Residency, Las Vegas, Nev (Dr. Moss)

DEPUTY EDITOR
Anne Mounsey, MD

Department of Family Medicine, University of North Carolina, Chapel Hill

Article PDF
Article PDF
 

ILLUSTRATIVE CASE

A 34-year-old woman presents to your office for unbearable sweating. She notes that the sweating occurs nearly daily on her hands, face, and in her axillary regions, causing social embarrassment. She has tried multiple antiperspirants to no avail. Is there anything she can take to reduce the sweating?

Hyperhidrosis is a common, self-limiting problem affecting 2% to 3% of the population in the United States.2 Patients may complain of localized sweating of the hands, feet, face, or underarms or more systemic, generalized sweating in multiple locations. Either way, patients always note a significant impact on their quality of life.

Treatment of hyperhidrosis has traditionally focused on topical therapies to the affected areas. Research has shown that localized treatment with antiperspirants containing aluminum salt is effective by both subjective report and objective measurements at reducing sweating—particularly in the axilla, hands, and feet.3,4 Additionally, a systematic review of observational and experimental studies found topical glycopyrrolate to be efficacious for craniofacial hyperhidrosis with minimal adverse effects.5 The availability of low-cost prescription and over-the-counter aluminum-based antiperspirant agents makes topicals the first-line choice.

More invasive treatments are available for hyperhidrosis that is refractory to topicals. In a double-blind, randomized controlled trial, researchers injected either botulinum toxin type A (BTX-A) 50 U or placebo in patients with bilateral primary axillary hyperhidrosis.6 Of the 207 patients who received treatment injections, 96.1% had at least a 50% reduction of axillary sweating at 4 weeks after one injection, as measured by gravimetric assessment. The BTX-A injections also produced a prolonged effect; mean duration between injections was 30.6 weeks.

Other invasive treatments include iontophoresis, surgery, and laser therapy; however, these methods are not suitable for body-wide application and are, thus, not appropriate for patients with generalized hyperhidrosis.

Oxybutynin is the first oral agent to emerge as a treatment option for hyperhidrosis. This cholinergic antagonist had historically been used to treat overactive bladder. As a cholinergic antagonist, oxybutynin not only reduces urinary frequency, but also decreases secretions in various locations and, thus, can cause dry mouth and reduce perspiration.

In one prospective placebo-controlled trial, 50 patients with generalized hyperhidrosis were randomized to receive either oxybutynin titrated from 2.5 mg orally once daily to 5 mg orally twice daily or placebo for 6 weeks.7 Seventeen (73.9%) patients receiving oxybutynin for palmar or axillary hyperhidrosis reported moderate to “great” resolution of their symptoms compared with 6 (27.3%) patients in the placebo group. Dry mouth was reported in 34.8% of patients receiving oxybutynin vs 9.1% of those who received placebo (P=.038); however, no patients dropped out of the study due to this adverse effect.7

STUDY SUMMARY

This multicenter, randomized controlled trial compared oxybutynin to placebo in 62 adults with localized or generalized hyperhidrosis from 12 outpatient dermatology practices in France. It is the first study to include patients with a localized, as well as a generalized form of the condition.

Patients were included if they were >18 years of age, enrolled in the National Health Insurance system in France, and reported a Hyperhidrosis Disease Severity Scale (HDSS) score ≥2. The HDSS is a validated, one-question tool (“How would you rate the severity of your sweating?”). Patients provide a score of 1 (no perceptible sweating and no interference with everyday life) to 4 (intolerable sweating with constant interference with everyday life).8 Patients were excluded if they had any contraindications to the use of an anticholinergic medication.

This trial used a relatively low dose of oxybutynin, which produced significant benefit while minimizing anticholinergic adverse effects.

Patients randomized to oxybutynin took 2.5 mg/d orally initially and increased gradually over 8 days until reaching an effective dose that was not more than 7.5 mg/d. They then continued at that dose for 6 weeks. The primary outcome was improvement on the HDSS by one or more points measured at the beginning of the trial and at 6 weeks. Secondary outcomes included change in quality of life, as measured by the Dermatology Life Quality Index (DLQI) and reported adverse effects. The DLQI is a dermatology-specific quality-of-life measure consisting of 10 questions. Scores range from 0 (where their disease has no impact on their quality of life) to 30 (maximum impact of their disease on their quality of life).9

Improved HDSS and DLQI scores. Most patients (83%) in the study had generalized hyperhidrosis. Patients were in their mid-thirties. Sixty percent of the patients in the oxybutynin group had an improvement of one point or more on the 4-point HDSS compared to 27% in the placebo group (P<.01). DLQI scores improved by 6.9 points in the oxybutynin group and 2.3 points in the placebo group (P<.01).

The most common adverse effect was dry mouth, which occurred in 13 patients (43%) in the oxybutynin group and in 3 patients (11%) in the placebo group (P<.01); it did not cause any patients to drop out of the study. The second most common adverse effect was blurred vision, which only occurred in the oxybutynin group (4 patients; 13%).

 

 

 

WHAT'S NEW

This is the first randomized controlled trial to demonstrate the efficacy of an oral agent for generalized primary hyperhidrosis. This trial used a relatively low dose of oxybutynin, which produced significant benefit while minimizing anticholinergic adverse effects.

CAVEATS

There are many situations for which anticholinergic medications are inappropriate, including use by geriatric patients and those with gastrointestinal disorders, urinary retention, or glaucoma.

CHALLENGES TO IMPLEMENTATION

Few if any challenges exist to the utilization of oxybutynin; inexpensive generic versions are widely available.

ACKNOWLEDGEMENT

The PURLs Surveillance System was supported in part by Grant Number UL1RR024999 from the National Center For Research Resources, a Clinical Translational Science Award to the University of Chicago. The content is solely the responsibility of the authors and does not necessarily represent the official views of the National Center For Research Resources or the National Institutes of Health.

 

ILLUSTRATIVE CASE

A 34-year-old woman presents to your office for unbearable sweating. She notes that the sweating occurs nearly daily on her hands, face, and in her axillary regions, causing social embarrassment. She has tried multiple antiperspirants to no avail. Is there anything she can take to reduce the sweating?

Hyperhidrosis is a common, self-limiting problem affecting 2% to 3% of the population in the United States.2 Patients may complain of localized sweating of the hands, feet, face, or underarms or more systemic, generalized sweating in multiple locations. Either way, patients always note a significant impact on their quality of life.

Treatment of hyperhidrosis has traditionally focused on topical therapies to the affected areas. Research has shown that localized treatment with antiperspirants containing aluminum salt is effective by both subjective report and objective measurements at reducing sweating—particularly in the axilla, hands, and feet.3,4 Additionally, a systematic review of observational and experimental studies found topical glycopyrrolate to be efficacious for craniofacial hyperhidrosis with minimal adverse effects.5 The availability of low-cost prescription and over-the-counter aluminum-based antiperspirant agents makes topicals the first-line choice.

More invasive treatments are available for hyperhidrosis that is refractory to topicals. In a double-blind, randomized controlled trial, researchers injected either botulinum toxin type A (BTX-A) 50 U or placebo in patients with bilateral primary axillary hyperhidrosis.6 Of the 207 patients who received treatment injections, 96.1% had at least a 50% reduction of axillary sweating at 4 weeks after one injection, as measured by gravimetric assessment. The BTX-A injections also produced a prolonged effect; mean duration between injections was 30.6 weeks.

Other invasive treatments include iontophoresis, surgery, and laser therapy; however, these methods are not suitable for body-wide application and are, thus, not appropriate for patients with generalized hyperhidrosis.

Oxybutynin is the first oral agent to emerge as a treatment option for hyperhidrosis. This cholinergic antagonist had historically been used to treat overactive bladder. As a cholinergic antagonist, oxybutynin not only reduces urinary frequency, but also decreases secretions in various locations and, thus, can cause dry mouth and reduce perspiration.

In one prospective placebo-controlled trial, 50 patients with generalized hyperhidrosis were randomized to receive either oxybutynin titrated from 2.5 mg orally once daily to 5 mg orally twice daily or placebo for 6 weeks.7 Seventeen (73.9%) patients receiving oxybutynin for palmar or axillary hyperhidrosis reported moderate to “great” resolution of their symptoms compared with 6 (27.3%) patients in the placebo group. Dry mouth was reported in 34.8% of patients receiving oxybutynin vs 9.1% of those who received placebo (P=.038); however, no patients dropped out of the study due to this adverse effect.7

STUDY SUMMARY

This multicenter, randomized controlled trial compared oxybutynin to placebo in 62 adults with localized or generalized hyperhidrosis from 12 outpatient dermatology practices in France. It is the first study to include patients with a localized, as well as a generalized form of the condition.

Patients were included if they were >18 years of age, enrolled in the National Health Insurance system in France, and reported a Hyperhidrosis Disease Severity Scale (HDSS) score ≥2. The HDSS is a validated, one-question tool (“How would you rate the severity of your sweating?”). Patients provide a score of 1 (no perceptible sweating and no interference with everyday life) to 4 (intolerable sweating with constant interference with everyday life).8 Patients were excluded if they had any contraindications to the use of an anticholinergic medication.

This trial used a relatively low dose of oxybutynin, which produced significant benefit while minimizing anticholinergic adverse effects.

Patients randomized to oxybutynin took 2.5 mg/d orally initially and increased gradually over 8 days until reaching an effective dose that was not more than 7.5 mg/d. They then continued at that dose for 6 weeks. The primary outcome was improvement on the HDSS by one or more points measured at the beginning of the trial and at 6 weeks. Secondary outcomes included change in quality of life, as measured by the Dermatology Life Quality Index (DLQI) and reported adverse effects. The DLQI is a dermatology-specific quality-of-life measure consisting of 10 questions. Scores range from 0 (where their disease has no impact on their quality of life) to 30 (maximum impact of their disease on their quality of life).9

Improved HDSS and DLQI scores. Most patients (83%) in the study had generalized hyperhidrosis. Patients were in their mid-thirties. Sixty percent of the patients in the oxybutynin group had an improvement of one point or more on the 4-point HDSS compared to 27% in the placebo group (P<.01). DLQI scores improved by 6.9 points in the oxybutynin group and 2.3 points in the placebo group (P<.01).

The most common adverse effect was dry mouth, which occurred in 13 patients (43%) in the oxybutynin group and in 3 patients (11%) in the placebo group (P<.01); it did not cause any patients to drop out of the study. The second most common adverse effect was blurred vision, which only occurred in the oxybutynin group (4 patients; 13%).

 

 

 

WHAT'S NEW

This is the first randomized controlled trial to demonstrate the efficacy of an oral agent for generalized primary hyperhidrosis. This trial used a relatively low dose of oxybutynin, which produced significant benefit while minimizing anticholinergic adverse effects.

CAVEATS

There are many situations for which anticholinergic medications are inappropriate, including use by geriatric patients and those with gastrointestinal disorders, urinary retention, or glaucoma.

CHALLENGES TO IMPLEMENTATION

Few if any challenges exist to the utilization of oxybutynin; inexpensive generic versions are widely available.

ACKNOWLEDGEMENT

The PURLs Surveillance System was supported in part by Grant Number UL1RR024999 from the National Center For Research Resources, a Clinical Translational Science Award to the University of Chicago. The content is solely the responsibility of the authors and does not necessarily represent the official views of the National Center For Research Resources or the National Institutes of Health.

References

1. Schollhammer M, Brenaut E, Menard-Andivot N, et al. Oxybutynin as a treatment for generalized hyperhidrosis: a randomized, placebo-controlled trial. Br J Dermatol. 2015;173:1163-1168.

2. Grabell DA, Hebert AA. Current and emerging medical therapies for primary hyperhidrosis. Dermatol Ther (Heidelb). 2017;7:25-36.

3. Innocenzi D, Lupi F, Bruni F, et al. Efficacy of a new aluminium salt thermophobic foam in the treatment of axillary and palmar primary hyperhidrosis: a pilot exploratory trial. Curr Med Res Opin. 2005;21:1949-1953.

4. Goh CL. Aluminum chloride hexahydrate versus palmar hyperhidrosis. Evaporimeter assessment. Int J Dermatol. 1990;29:368-370.

5. Nicholas R, Quddus A, Baker DM. Treatment of primary craniofacial hyperhidrosis: a systematic review. Am J Clin Dermatol. 2015;16:361-370.

6. Naumann M, Lowe NJ, Kumar CR, et al. Botulinum toxin type a is a safe and effective treatment for axillary hyperhidrosis over 16 months: a prospective study. Arch Dermatol. 2003;139:731-736.

7. Wolosker N, de Campos JR, Kauffman P, et al. A randomized placebo-controlled trial of oxybutynin for the initial treatment of palmar and axillary hyperhidrosis. J Vasc Surg. 2012;55:1696-1700.

8. Varella AY, Fukuda JM, Teivelis MP, et al. Translation and validation of Hyperhidrosis Disease Severity Scale. Rev Assoc Med Bras. 2016;62:843-847.

9. Finlay AY, Khan GK. Dermatology Life Quality Index (DLQI)—a simple practical measure for routine clinical use. Clin Exp Dermatol. 1994;19:210-216.

References

1. Schollhammer M, Brenaut E, Menard-Andivot N, et al. Oxybutynin as a treatment for generalized hyperhidrosis: a randomized, placebo-controlled trial. Br J Dermatol. 2015;173:1163-1168.

2. Grabell DA, Hebert AA. Current and emerging medical therapies for primary hyperhidrosis. Dermatol Ther (Heidelb). 2017;7:25-36.

3. Innocenzi D, Lupi F, Bruni F, et al. Efficacy of a new aluminium salt thermophobic foam in the treatment of axillary and palmar primary hyperhidrosis: a pilot exploratory trial. Curr Med Res Opin. 2005;21:1949-1953.

4. Goh CL. Aluminum chloride hexahydrate versus palmar hyperhidrosis. Evaporimeter assessment. Int J Dermatol. 1990;29:368-370.

5. Nicholas R, Quddus A, Baker DM. Treatment of primary craniofacial hyperhidrosis: a systematic review. Am J Clin Dermatol. 2015;16:361-370.

6. Naumann M, Lowe NJ, Kumar CR, et al. Botulinum toxin type a is a safe and effective treatment for axillary hyperhidrosis over 16 months: a prospective study. Arch Dermatol. 2003;139:731-736.

7. Wolosker N, de Campos JR, Kauffman P, et al. A randomized placebo-controlled trial of oxybutynin for the initial treatment of palmar and axillary hyperhidrosis. J Vasc Surg. 2012;55:1696-1700.

8. Varella AY, Fukuda JM, Teivelis MP, et al. Translation and validation of Hyperhidrosis Disease Severity Scale. Rev Assoc Med Bras. 2016;62:843-847.

9. Finlay AY, Khan GK. Dermatology Life Quality Index (DLQI)—a simple practical measure for routine clinical use. Clin Exp Dermatol. 1994;19:210-216.

Issue
The Journal of Family Practice - 66(6)
Issue
The Journal of Family Practice - 66(6)
Page Number
392-394
Page Number
392-394
Publications
Publications
Topics
Article Type
Display Headline
Oral agent offers relief from generalized hyperhidrosis
Display Headline
Oral agent offers relief from generalized hyperhidrosis
Sections
PURLs Copyright

Copyright © 2017. The Family Physicians Inquiries Network. All rights reserved.

Inside the Article

PRACTICE CHANGER

Use low-dose oxybutynin as a first-line treatment option for patients with primary hyperhidrosis to improve symptoms and quality of life.1

STRENGTH OF RECOMMENDATION

B: Based on a single, good quality, randomized controlled trial.

Schollhammer M, Brenaut E, Menard-Andivot N, et al. Oxybutynin as a treatment for generalized hyperhidrosis: a randomized, placebo-controlled trial. Br J Dermatol. 2015;173:1163-1168.

Disallow All Ads
Content Gating
No Gating (article Unlocked/Free)
Alternative CME
PubMed ID
28574524
Use ProPublica
Hide sidebar & use full width
render the right sidebar.
Conference Recap Checkbox
Not Conference Recap
Clinical Edge
Article PDF Media
Media Files

When Can Exercise Supplant Surgery for Degenerative Meniscal Tears?

Article Type
Changed
Fri, 05/05/2017 - 01:24
Display Headline
When Can Exercise Supplant Surgery for Degenerative Meniscal Tears?

 

A 48-year-old man presents to your office for follow-up of right knee pain that has been bothering him for the past 12 months. He denies any trauma or inciting incident for the pain. On physical exam, he does not have crepitus but does have medial joint line tenderness of his right knee. An MRI shows a partial medial meniscal tear. Do you refer him to physical therapy (PT) or to orthopedics for arthroscopy and repair?

The meniscus—cartilage in the knee joint that provides support, stability, and lubrication to the joint during activity—can tear during a traumatic event or as a result of degeneration over time. Traumatic meniscal tears typically occur in those younger than 30 during sports (eg, basketball, soccer), whereas degenerative meniscal tears generally occur in patients ages 40 to 60.2,3 The annual incidence of all meniscal tears is 79 per 100,000.4 While some clinicians can diagnose traumatic meniscal tears based on history and physical examination, degenerative meniscal tears are more challenging and typically warrant an MRI for confirmation.3

Meniscal tears can be treated either conservatively, with supportive care and exercise, or surgically. Unfortunately, there are no national orthopedic guidelines available to help direct care. In one observational study, 95 of 117 patients (81.2%) were generally satisfied with surgical treatment at four-year follow-up; satisfaction was higher among those with a traumatic meniscal tear than in those with a degenerative tear.5

Two systematic reviews of surgery versus nonoperative management or sham therapies found no additional benefit of surgery for meniscal tears in a variety of patients with and without osteoarthritis.6,7 However, both studies were of only moderate quality, because of the number of patients in the nonoperative groups who ultimately underwent surgery. Neither of the studies directly compared surgery to nonoperative management.6,7Another investigation—a multicenter, randomized, double-blind, sham-controlled study conducted in Finland involving 146 patients—compared sham surgery to arthroscopic partial meniscectomy. Both groups received instruction on performing post-procedure exercises, and both groups had similar and marked improvement in pain and function.8

Clinical practice recommendations devised from a vast systematic review of the literature recommend that the decision for surgery be based on patient-specific factors, such as symptoms, age, mechanism of tear, extent of damage, and occupational/social/activity needs.9

STUDY SUMMARY

Exercise is as good as surgery

The current superiority RCT compared exercise therapy to arthroscopic partial meniscectomy. Subjects (ages 35 to 60) presented to the orthopedic department of two hospitals in Norway with unilateral knee pain of more than two months’ duration and an MRI-delineated medial meniscal tear. They were included in the study only if they had radiographic evidence of minimal osteoarthritis (Kellgren-Lawrence classification grade ≤ 2). Exclusion criteria included acute trauma, locked knee, ligament injury, and knee surgery in the same knee within the previous two years.

The primary outcomes were change in patient-reported knee function (as determined by overall Knee injury and Osteoarthritis Outcome Score [KOOS] after two years) and thigh muscle strength at three months (as measured by physiotherapists). The researchers used four of the five KOOS subscales for this analysis: pain, other symptoms (swelling, grinding/noise from the joint, ability to straighten and bend), function in sports/recreation, and knee-related quality of life (QOL). The average score of each subscale was used.

Secondary outcomes included the five individual KOOS subscales (the four previously mentioned, plus activities of daily living [ADLs]), as well as thigh muscle strength and lower-extremity performance test results.

Methods. Testing personnel were blinded to group allocation; participants wore pants or neoprene sleeves to cover surgical scars. A total of 140 patients were randomized to either 12 weeks (24-36 sessions) of exercise therapy alone or a standardized arthroscopic partial meniscectomy; upon discharge, those in the latter group received written and oral encouragement to perform simple exercises at home, two to four times daily, to regain range of motion and reduce swelling.

Results. At two years, the overall mean improvement in KOOS4 score from baseline was similar between the exercise group and the meniscectomy group (25.3 pts vs 24.4 pts, respectively; mean difference [MD], 0.9). Additionally, muscle strength (measured as peak torque flexion and extension and total work flexion and extension) at both three and 12 months showed significant objective improvements favoring exercise therapy.

In the secondary analysis of the KOOS subscale scores, change from baseline was nonsignificant for four of the five (pain, ADL, sports/recreation, and QOL). Only the symptoms subscale had a significant difference favoring exercise therapy (MD, 5.3 pts); this was likely clinically insignificant on a grading scale of 0 to 100.

Of the patients allocated to exercise therapy alone, 19% crossed over and underwent surgery during the two-year study period.

 

 

 

WHAT’S NEW

Head-to-head comparison adds evidence

This is the first trial to directly compare exercise therapy to surgery in patients with meniscal tears. Interestingly, exercise therapy was as effective after a two-year follow-up period and was superior in the short term for thigh muscle strength.1

The results of this study build on those from the aforementioned smaller study conducted in Finland.8 In that study, both groups received instruction for the same graduated exercise plan. The researchers found that exercise was comparable to surgery for meniscal tears in patients with no osteoarthritis.

CAVEATS

What about more severe osteoarthritis?

This trial included patients with no to mild osteoarthritis in addition to their meniscal tear.1 It is unclear if the results would be maintained in those with more advanced disease. Additionally, 19% of patients crossed over from the exercise group to the surgery group, even though muscle strength improved. Therefore, education about the risks of surgery and the potential lack of benefit is important.

CHALLENGES TO IMPLEMENTATION

Cost and effort of PT

The cost of PT can be a barrier for patients who have adequate insurance coverage for surgery but inadequate coverage for PT. Additionally, exercise therapy requires significant and ongoing time and effort, which may deter those with busy lifestyles. Patients and clinicians may view surgery as an “easier” fix.

ACKNOWLEDGEMENT

The PURLs Surveillance System was supported in part by Grant Number UL1RR024999 from the National Center For Research Resources, a Clinical Translational Science Award to the University of Chicago. The content is solely the responsibility of the authors and does not necessarily represent the official views of the National Center For Research Resources or the National Institutes of Health.

Copyright © 2017. The Family Physicians Inquiries Network. All rights reserved.

Reprinted with permission from the Family Physicians Inquiries Network and The Journal of Family Practice (2017;66[4]:250-252).

References

1. Kise NJ, Risberg MA, Stensrud S, et al. Exercise therapy versus arthroscopic partial meniscectomy for degenerative meniscal tear in middle aged patients: randomised controlled trial with two year follow-up. BMJ. 2016;354:i3740.
2. Beals CT, Magnussen RA, Graham WC, et al. The prevalence of meniscal pathology in asymptomatic athletes. Sports Med. 2016;46:1517-1524.
3. Maffulli N, Longo UG, Campi S, et al. Meniscal tears. Open Access J Sports Med. 2010;1:45-54.
4. Peat G, Bergknut C, Frobell R, et al. Population-wide incidence estimates for soft tissue knee injuries presenting to healthcare in southern Sweden: data from the Skåne Healthcare Register. Arthritis Res Ther. 2014;16:R162.
5. Ghislain NA, Wei JN, Li YG. Study of the clinical outcome between traumatic and degenerative (non-traumatic) meniscal tears after arthroscopic surgery: a 4-years follow-up study. J Clin Diagn Res. 2016;10:RC01-RC04.
6. Khan M, Evaniew N, Bedi A, et al. Arthroscopic surgery for degenerative tears of the meniscus: a systematic review and meta-analysis. CMAJ. 2014;186:1057-1064.
7. Monk P, Garfjeld Roberts P, Palmer AJ, et al. The urgent need for evidence in arthroscopic meniscal surgery: a systematic review of the evidence for operative management of meniscal tears. Am J Sports Med. 2017;45:965-973.
8. Sihvonen R, Paavola M, Malmivaara A, et al; Finnish Degenerative Meniscal Lesion Study (FIDELITY) Group. Arthroscopic partial meniscectomy versus sham surgery for a degenerative meniscal tear. N Engl J Med. 2013;369:2515-2524.
9. Beaufils P, Hulet C, Dhénain M, et al. Clinical practice guidelines for the management of meniscal lesions and isolated lesions of the anterior cruciate ligament of the knee in adults. Orthop Traumatol Surg Res. 2009;95:437-442.

Article PDF
Author and Disclosure Information

Jennie B. Jarrett is with the University of Illinois at Chicago. Sandra Sauereisen is with the St. Margaret Family Medicine Residency Program at the University of Pittsburgh Medical Center.

Issue
Clinician Reviews - 27(5)
Publications
Topics
Page Number
14-16
Sections
Author and Disclosure Information

Jennie B. Jarrett is with the University of Illinois at Chicago. Sandra Sauereisen is with the St. Margaret Family Medicine Residency Program at the University of Pittsburgh Medical Center.

Author and Disclosure Information

Jennie B. Jarrett is with the University of Illinois at Chicago. Sandra Sauereisen is with the St. Margaret Family Medicine Residency Program at the University of Pittsburgh Medical Center.

Article PDF
Article PDF
Related Articles

 

A 48-year-old man presents to your office for follow-up of right knee pain that has been bothering him for the past 12 months. He denies any trauma or inciting incident for the pain. On physical exam, he does not have crepitus but does have medial joint line tenderness of his right knee. An MRI shows a partial medial meniscal tear. Do you refer him to physical therapy (PT) or to orthopedics for arthroscopy and repair?

The meniscus—cartilage in the knee joint that provides support, stability, and lubrication to the joint during activity—can tear during a traumatic event or as a result of degeneration over time. Traumatic meniscal tears typically occur in those younger than 30 during sports (eg, basketball, soccer), whereas degenerative meniscal tears generally occur in patients ages 40 to 60.2,3 The annual incidence of all meniscal tears is 79 per 100,000.4 While some clinicians can diagnose traumatic meniscal tears based on history and physical examination, degenerative meniscal tears are more challenging and typically warrant an MRI for confirmation.3

Meniscal tears can be treated either conservatively, with supportive care and exercise, or surgically. Unfortunately, there are no national orthopedic guidelines available to help direct care. In one observational study, 95 of 117 patients (81.2%) were generally satisfied with surgical treatment at four-year follow-up; satisfaction was higher among those with a traumatic meniscal tear than in those with a degenerative tear.5

Two systematic reviews of surgery versus nonoperative management or sham therapies found no additional benefit of surgery for meniscal tears in a variety of patients with and without osteoarthritis.6,7 However, both studies were of only moderate quality, because of the number of patients in the nonoperative groups who ultimately underwent surgery. Neither of the studies directly compared surgery to nonoperative management.6,7Another investigation—a multicenter, randomized, double-blind, sham-controlled study conducted in Finland involving 146 patients—compared sham surgery to arthroscopic partial meniscectomy. Both groups received instruction on performing post-procedure exercises, and both groups had similar and marked improvement in pain and function.8

Clinical practice recommendations devised from a vast systematic review of the literature recommend that the decision for surgery be based on patient-specific factors, such as symptoms, age, mechanism of tear, extent of damage, and occupational/social/activity needs.9

STUDY SUMMARY

Exercise is as good as surgery

The current superiority RCT compared exercise therapy to arthroscopic partial meniscectomy. Subjects (ages 35 to 60) presented to the orthopedic department of two hospitals in Norway with unilateral knee pain of more than two months’ duration and an MRI-delineated medial meniscal tear. They were included in the study only if they had radiographic evidence of minimal osteoarthritis (Kellgren-Lawrence classification grade ≤ 2). Exclusion criteria included acute trauma, locked knee, ligament injury, and knee surgery in the same knee within the previous two years.

The primary outcomes were change in patient-reported knee function (as determined by overall Knee injury and Osteoarthritis Outcome Score [KOOS] after two years) and thigh muscle strength at three months (as measured by physiotherapists). The researchers used four of the five KOOS subscales for this analysis: pain, other symptoms (swelling, grinding/noise from the joint, ability to straighten and bend), function in sports/recreation, and knee-related quality of life (QOL). The average score of each subscale was used.

Secondary outcomes included the five individual KOOS subscales (the four previously mentioned, plus activities of daily living [ADLs]), as well as thigh muscle strength and lower-extremity performance test results.

Methods. Testing personnel were blinded to group allocation; participants wore pants or neoprene sleeves to cover surgical scars. A total of 140 patients were randomized to either 12 weeks (24-36 sessions) of exercise therapy alone or a standardized arthroscopic partial meniscectomy; upon discharge, those in the latter group received written and oral encouragement to perform simple exercises at home, two to four times daily, to regain range of motion and reduce swelling.

Results. At two years, the overall mean improvement in KOOS4 score from baseline was similar between the exercise group and the meniscectomy group (25.3 pts vs 24.4 pts, respectively; mean difference [MD], 0.9). Additionally, muscle strength (measured as peak torque flexion and extension and total work flexion and extension) at both three and 12 months showed significant objective improvements favoring exercise therapy.

In the secondary analysis of the KOOS subscale scores, change from baseline was nonsignificant for four of the five (pain, ADL, sports/recreation, and QOL). Only the symptoms subscale had a significant difference favoring exercise therapy (MD, 5.3 pts); this was likely clinically insignificant on a grading scale of 0 to 100.

Of the patients allocated to exercise therapy alone, 19% crossed over and underwent surgery during the two-year study period.

 

 

 

WHAT’S NEW

Head-to-head comparison adds evidence

This is the first trial to directly compare exercise therapy to surgery in patients with meniscal tears. Interestingly, exercise therapy was as effective after a two-year follow-up period and was superior in the short term for thigh muscle strength.1

The results of this study build on those from the aforementioned smaller study conducted in Finland.8 In that study, both groups received instruction for the same graduated exercise plan. The researchers found that exercise was comparable to surgery for meniscal tears in patients with no osteoarthritis.

CAVEATS

What about more severe osteoarthritis?

This trial included patients with no to mild osteoarthritis in addition to their meniscal tear.1 It is unclear if the results would be maintained in those with more advanced disease. Additionally, 19% of patients crossed over from the exercise group to the surgery group, even though muscle strength improved. Therefore, education about the risks of surgery and the potential lack of benefit is important.

CHALLENGES TO IMPLEMENTATION

Cost and effort of PT

The cost of PT can be a barrier for patients who have adequate insurance coverage for surgery but inadequate coverage for PT. Additionally, exercise therapy requires significant and ongoing time and effort, which may deter those with busy lifestyles. Patients and clinicians may view surgery as an “easier” fix.

ACKNOWLEDGEMENT

The PURLs Surveillance System was supported in part by Grant Number UL1RR024999 from the National Center For Research Resources, a Clinical Translational Science Award to the University of Chicago. The content is solely the responsibility of the authors and does not necessarily represent the official views of the National Center For Research Resources or the National Institutes of Health.

Copyright © 2017. The Family Physicians Inquiries Network. All rights reserved.

Reprinted with permission from the Family Physicians Inquiries Network and The Journal of Family Practice (2017;66[4]:250-252).

 

A 48-year-old man presents to your office for follow-up of right knee pain that has been bothering him for the past 12 months. He denies any trauma or inciting incident for the pain. On physical exam, he does not have crepitus but does have medial joint line tenderness of his right knee. An MRI shows a partial medial meniscal tear. Do you refer him to physical therapy (PT) or to orthopedics for arthroscopy and repair?

The meniscus—cartilage in the knee joint that provides support, stability, and lubrication to the joint during activity—can tear during a traumatic event or as a result of degeneration over time. Traumatic meniscal tears typically occur in those younger than 30 during sports (eg, basketball, soccer), whereas degenerative meniscal tears generally occur in patients ages 40 to 60.2,3 The annual incidence of all meniscal tears is 79 per 100,000.4 While some clinicians can diagnose traumatic meniscal tears based on history and physical examination, degenerative meniscal tears are more challenging and typically warrant an MRI for confirmation.3

Meniscal tears can be treated either conservatively, with supportive care and exercise, or surgically. Unfortunately, there are no national orthopedic guidelines available to help direct care. In one observational study, 95 of 117 patients (81.2%) were generally satisfied with surgical treatment at four-year follow-up; satisfaction was higher among those with a traumatic meniscal tear than in those with a degenerative tear.5

Two systematic reviews of surgery versus nonoperative management or sham therapies found no additional benefit of surgery for meniscal tears in a variety of patients with and without osteoarthritis.6,7 However, both studies were of only moderate quality, because of the number of patients in the nonoperative groups who ultimately underwent surgery. Neither of the studies directly compared surgery to nonoperative management.6,7Another investigation—a multicenter, randomized, double-blind, sham-controlled study conducted in Finland involving 146 patients—compared sham surgery to arthroscopic partial meniscectomy. Both groups received instruction on performing post-procedure exercises, and both groups had similar and marked improvement in pain and function.8

Clinical practice recommendations devised from a vast systematic review of the literature recommend that the decision for surgery be based on patient-specific factors, such as symptoms, age, mechanism of tear, extent of damage, and occupational/social/activity needs.9

STUDY SUMMARY

Exercise is as good as surgery

The current superiority RCT compared exercise therapy to arthroscopic partial meniscectomy. Subjects (ages 35 to 60) presented to the orthopedic department of two hospitals in Norway with unilateral knee pain of more than two months’ duration and an MRI-delineated medial meniscal tear. They were included in the study only if they had radiographic evidence of minimal osteoarthritis (Kellgren-Lawrence classification grade ≤ 2). Exclusion criteria included acute trauma, locked knee, ligament injury, and knee surgery in the same knee within the previous two years.

The primary outcomes were change in patient-reported knee function (as determined by overall Knee injury and Osteoarthritis Outcome Score [KOOS] after two years) and thigh muscle strength at three months (as measured by physiotherapists). The researchers used four of the five KOOS subscales for this analysis: pain, other symptoms (swelling, grinding/noise from the joint, ability to straighten and bend), function in sports/recreation, and knee-related quality of life (QOL). The average score of each subscale was used.

Secondary outcomes included the five individual KOOS subscales (the four previously mentioned, plus activities of daily living [ADLs]), as well as thigh muscle strength and lower-extremity performance test results.

Methods. Testing personnel were blinded to group allocation; participants wore pants or neoprene sleeves to cover surgical scars. A total of 140 patients were randomized to either 12 weeks (24-36 sessions) of exercise therapy alone or a standardized arthroscopic partial meniscectomy; upon discharge, those in the latter group received written and oral encouragement to perform simple exercises at home, two to four times daily, to regain range of motion and reduce swelling.

Results. At two years, the overall mean improvement in KOOS4 score from baseline was similar between the exercise group and the meniscectomy group (25.3 pts vs 24.4 pts, respectively; mean difference [MD], 0.9). Additionally, muscle strength (measured as peak torque flexion and extension and total work flexion and extension) at both three and 12 months showed significant objective improvements favoring exercise therapy.

In the secondary analysis of the KOOS subscale scores, change from baseline was nonsignificant for four of the five (pain, ADL, sports/recreation, and QOL). Only the symptoms subscale had a significant difference favoring exercise therapy (MD, 5.3 pts); this was likely clinically insignificant on a grading scale of 0 to 100.

Of the patients allocated to exercise therapy alone, 19% crossed over and underwent surgery during the two-year study period.

 

 

 

WHAT’S NEW

Head-to-head comparison adds evidence

This is the first trial to directly compare exercise therapy to surgery in patients with meniscal tears. Interestingly, exercise therapy was as effective after a two-year follow-up period and was superior in the short term for thigh muscle strength.1

The results of this study build on those from the aforementioned smaller study conducted in Finland.8 In that study, both groups received instruction for the same graduated exercise plan. The researchers found that exercise was comparable to surgery for meniscal tears in patients with no osteoarthritis.

CAVEATS

What about more severe osteoarthritis?

This trial included patients with no to mild osteoarthritis in addition to their meniscal tear.1 It is unclear if the results would be maintained in those with more advanced disease. Additionally, 19% of patients crossed over from the exercise group to the surgery group, even though muscle strength improved. Therefore, education about the risks of surgery and the potential lack of benefit is important.

CHALLENGES TO IMPLEMENTATION

Cost and effort of PT

The cost of PT can be a barrier for patients who have adequate insurance coverage for surgery but inadequate coverage for PT. Additionally, exercise therapy requires significant and ongoing time and effort, which may deter those with busy lifestyles. Patients and clinicians may view surgery as an “easier” fix.

ACKNOWLEDGEMENT

The PURLs Surveillance System was supported in part by Grant Number UL1RR024999 from the National Center For Research Resources, a Clinical Translational Science Award to the University of Chicago. The content is solely the responsibility of the authors and does not necessarily represent the official views of the National Center For Research Resources or the National Institutes of Health.

Copyright © 2017. The Family Physicians Inquiries Network. All rights reserved.

Reprinted with permission from the Family Physicians Inquiries Network and The Journal of Family Practice (2017;66[4]:250-252).

References

1. Kise NJ, Risberg MA, Stensrud S, et al. Exercise therapy versus arthroscopic partial meniscectomy for degenerative meniscal tear in middle aged patients: randomised controlled trial with two year follow-up. BMJ. 2016;354:i3740.
2. Beals CT, Magnussen RA, Graham WC, et al. The prevalence of meniscal pathology in asymptomatic athletes. Sports Med. 2016;46:1517-1524.
3. Maffulli N, Longo UG, Campi S, et al. Meniscal tears. Open Access J Sports Med. 2010;1:45-54.
4. Peat G, Bergknut C, Frobell R, et al. Population-wide incidence estimates for soft tissue knee injuries presenting to healthcare in southern Sweden: data from the Skåne Healthcare Register. Arthritis Res Ther. 2014;16:R162.
5. Ghislain NA, Wei JN, Li YG. Study of the clinical outcome between traumatic and degenerative (non-traumatic) meniscal tears after arthroscopic surgery: a 4-years follow-up study. J Clin Diagn Res. 2016;10:RC01-RC04.
6. Khan M, Evaniew N, Bedi A, et al. Arthroscopic surgery for degenerative tears of the meniscus: a systematic review and meta-analysis. CMAJ. 2014;186:1057-1064.
7. Monk P, Garfjeld Roberts P, Palmer AJ, et al. The urgent need for evidence in arthroscopic meniscal surgery: a systematic review of the evidence for operative management of meniscal tears. Am J Sports Med. 2017;45:965-973.
8. Sihvonen R, Paavola M, Malmivaara A, et al; Finnish Degenerative Meniscal Lesion Study (FIDELITY) Group. Arthroscopic partial meniscectomy versus sham surgery for a degenerative meniscal tear. N Engl J Med. 2013;369:2515-2524.
9. Beaufils P, Hulet C, Dhénain M, et al. Clinical practice guidelines for the management of meniscal lesions and isolated lesions of the anterior cruciate ligament of the knee in adults. Orthop Traumatol Surg Res. 2009;95:437-442.

References

1. Kise NJ, Risberg MA, Stensrud S, et al. Exercise therapy versus arthroscopic partial meniscectomy for degenerative meniscal tear in middle aged patients: randomised controlled trial with two year follow-up. BMJ. 2016;354:i3740.
2. Beals CT, Magnussen RA, Graham WC, et al. The prevalence of meniscal pathology in asymptomatic athletes. Sports Med. 2016;46:1517-1524.
3. Maffulli N, Longo UG, Campi S, et al. Meniscal tears. Open Access J Sports Med. 2010;1:45-54.
4. Peat G, Bergknut C, Frobell R, et al. Population-wide incidence estimates for soft tissue knee injuries presenting to healthcare in southern Sweden: data from the Skåne Healthcare Register. Arthritis Res Ther. 2014;16:R162.
5. Ghislain NA, Wei JN, Li YG. Study of the clinical outcome between traumatic and degenerative (non-traumatic) meniscal tears after arthroscopic surgery: a 4-years follow-up study. J Clin Diagn Res. 2016;10:RC01-RC04.
6. Khan M, Evaniew N, Bedi A, et al. Arthroscopic surgery for degenerative tears of the meniscus: a systematic review and meta-analysis. CMAJ. 2014;186:1057-1064.
7. Monk P, Garfjeld Roberts P, Palmer AJ, et al. The urgent need for evidence in arthroscopic meniscal surgery: a systematic review of the evidence for operative management of meniscal tears. Am J Sports Med. 2017;45:965-973.
8. Sihvonen R, Paavola M, Malmivaara A, et al; Finnish Degenerative Meniscal Lesion Study (FIDELITY) Group. Arthroscopic partial meniscectomy versus sham surgery for a degenerative meniscal tear. N Engl J Med. 2013;369:2515-2524.
9. Beaufils P, Hulet C, Dhénain M, et al. Clinical practice guidelines for the management of meniscal lesions and isolated lesions of the anterior cruciate ligament of the knee in adults. Orthop Traumatol Surg Res. 2009;95:437-442.

Issue
Clinician Reviews - 27(5)
Issue
Clinician Reviews - 27(5)
Page Number
14-16
Page Number
14-16
Publications
Publications
Topics
Article Type
Display Headline
When Can Exercise Supplant Surgery for Degenerative Meniscal Tears?
Display Headline
When Can Exercise Supplant Surgery for Degenerative Meniscal Tears?
Sections
Disallow All Ads
Content Gating
No Gating (article Unlocked/Free)
Alternative CME
Article PDF Media

When can exercise supplant surgery for degenerative meniscal tears?

Article Type
Changed
Tue, 06/23/2020 - 13:37
Display Headline
When can exercise supplant surgery for degenerative meniscal tears?
 

ILLUSTRATIVE CASE

A 48-year-old man presents to your office for follow-up of right knee pain that has been bothering him for the last 12 months. He denies any trauma or inciting incident for the pain. On physical exam, he does not have crepitus, but has medial joint line tenderness of his right knee. A magnetic resonance image (MRI) shows a partial, medial meniscal tear. Do you refer him to Physical Therapy (PT) or Orthopedics for arthroscopy and repair?

The meniscus—cartilage in the knee joint that provides support, stability, and lubrication to the joint during activity—can tear during a traumatic event or because of degeneration over time. Traumatic meniscal tears typically happen to younger adults and teens (<30 years of age) during sports, such as basketball and soccer,whereas degenerative meniscal tears generally present in patients ages 40 to 60 years.2,3 The annual incidence of all meniscal tears is 79 per 100,000.4 While some physicians can diagnose traumatic meniscal tears based on history and physical examination, degenerative meniscal tears are generally more challenging, and typically warrant an MRI for confirmation.3

Meniscal tears can be treated either conservatively, with supportive care and exercise, or with surgery. Unfortunately, there are no national orthopedic guidelines available to help direct care. In one observational study of surgery as treatment for both traumatic and degenerative meniscal tears, 95 out of 117 patients (81.2%) were generally satisfied with this treatment at the 4-year follow-up, with higher satisfaction in the traumatic meniscal tear group than in the degenerative tear group.5

Two systematic reviews of surgery vs nonoperative management or sham therapies found no additional benefit of surgery for meniscal tears in a variety of patients with and without osteoarthritis.6,7 However, both studies were of only moderate quality because of the number of patients in the nonoperative groups who ultimately obtained surgery. And neither of the studies directly compared surgery to nonoperative management.6,7

Yet another investigation, a multicenter, randomized, double-blind, sham-controlled study conducted in Finland involving 146 patients, compared sham surgery to arthroscopic partial meniscectomy. Both groups received instruction on performing post-procedure exercises, and both groups had similar and marked improvement in pain and function.8

Clinical practice recommendations devised from a systematic and vast review of the literature recommend that the decision for surgery be based on patient-specific factors such as symptoms, age, mechanism of tear, extent of damage, and occupational/­social/activity needs.9

STUDY SUMMARY

Exercise is as good as—and in one way, better than—surgery

The current randomized controlled superiority trial compared exercise therapy to arthroscopic partial meniscectomy in patients ages 35 to 60 years presenting to the orthopedic departments of 2 hospitals in Norway with unilateral knee pain for more than 2 months and an MRI-delineated medial meniscal tear. Patients were included only if they had radiographic evidence of minimal osteoarthritis (Kellgren-Lawrence classification grade ≤2). Exclusion criteria were acute trauma, locked knee, ligament injury, and knee surgery in the same knee within the previous 2 years.

The primary outcomes were change in patient-reported knee function as determined by overall knee injury and osteoarthritis outcome score (KOOS4) after 2 years and thigh muscle strength at 3 months as measured by physiotherapists. The KOOS4 consists of 4 out of the 5 KOOS subscales: pain, other symptoms (swelling, grinding/noise from the joint, ability to straighten and bend), function in sports/recreation, and knee-related quality of life (QOL). This study utilized the average score of each subscale.

Secondary outcomes were the 5 individual KOOS subscales (the 4 previously mentioned plus activities of daily living [ADLs]), as well as thigh muscle strength and lower extremity performance test results.

Methods. Testing personnel were blinded to group allocation; participants wore pants or neoprene sleeves to cover surgical scars. A total of 140 patients were randomized to either 12 weeks (24-36 sessions) of exercise therapy alone or a standardized arthroscopic partial meniscectomy with written and oral encouragement upon discharge to perform simple exercises at home 2 to4 times daily (to regain range of motion and reduce swelling).

Results. The overall mean improvement in KOOS4 score from baseline at 2 years was similar between the exercise group and the meniscectomy group (25.3 points vs 24.4 points, respectively; mean difference [MD], 0.9; 95% confidence interval [CI], -4.3 to 6.1; P=.72). Additionally, muscle strength (measured as peak torque flexion and extension and total work flexion and extension) at both 3 and 12 months showed significant objective improvements favoring exercise therapy.

Exercise therapy was as effective as surgery after a 2-year follow-up period and was superior in the short term for thigh muscle strength.

Secondary outcomes comparing the change from baseline of KOOS subscale scores showed 4 of the 5 having non-significant differences (pain, ADL, sports/­recreation, and QOL). Only the symptoms subscale had a significant difference favoring exercise therapy (MD, 5.3 points; 95% CI, 0.5 to 10.2; P=.03), which was likely clinically insignificant when using a grading scale of 0 to 100.

Of those patients allocated to exercise therapy alone, 19% crossed over and underwent surgery during the 2 years of the study.

 

 

 

WHAT'S NEW

Head-to-head comparison adds evidence to previous findings

This is the first trial to directly compare exercise therapy to surgery in patients with meniscal tears. Interestingly, exercise therapy was as effective after a 2-year follow-up period and was superior in the short term for thigh muscle strength.1 The results of this study build on those from the smaller study conducted in Finland mentioned earlier.8 In that study, both groups received instruction for the same graduated exercise plan. The researchers found that exercise was comparable to surgery for meniscal tears in patients with no osteoarthritis.

CAVEATS

Results may not translate to those with more severe osteoarthritis

This trial included patients with only mild to no osteoarthritis in addition to their meniscal tear.1 It is unclear if the results would be maintained in patients with more advanced disease. Additionally, 19% of patients crossed over from the exercise group to the surgery group, even though muscle strength improved. Therefore, education about the risks of surgery and the potential lack of benefit is important.

CHALLENGES TO IMPLEMENTATION

The cost and effort of physical therapy may be a deterrent

The cost of PT can be a barrier for some patients who have adequate insurance coverage for surgery, but inadequate coverage for PT. Additionally, exercise therapy requires significant and ongoing amounts of time and effort, which may be a deterrent for patients with busy lifestyles. Patients and physicians may view surgery as an “easier” fix.

ACKNOWLEDGEMENT

The PURLs Surveillance System was supported in part by Grant Number UL1RR024999 from the National Center For Research Resources, a Clinical Translational Science Award to the University of Chicago. The content is solely the responsibility of the authors and does not necessarily represent the official views of the National Center For Research Resources or the National Institutes of Health.

Files
References

1. Kise NJ, Risberg MA, Stensrud S, et al. Exercise therapy versus arthroscopic partial meniscectomy for degenerative meniscal tear in middle aged patients: randomised controlled trial with two year follow-up. BMJ. 2016;354:i3740.

2. Beals CT, Magnussen RA, Graham WC, et al. The prevalence of meniscal pathology in asymptomatic athletes. Sports Med. 2016;46:1517-1524.

3. Maffulli N, Longo UG, Campi S, et al. Meniscal tears. Open Access J Sports Med. 2010;1:45-54.

4. Peat G, Bergknut C, Frobell R, et al. Population-wide incidence estimates for soft tissue knee injuries presenting to healthcare in southern Sweden: data from the Skåne Healthcare Register. Arthritis Res Ther. 2014;16:R162.

5. Ghislain NA, Wei JN, Li YG. Study of the clinical outcome between traumatic and degenerative (non-traumatic) meniscal tears after arthroscopic surgery: a 4-years follow-up study. J Clin Diagn Res. 2016;10:RC01-RC04.

6. Khan M, Evaniew N, Bedi A, et al. Arthroscopic surgery for degenerative tears of the meniscus: a systematic review and meta-analysis. CMAJ. 2014;186:1057-1064.

7. Monk P, Garfjeld Roberts P, Palmer AJR, et al. The urgent need for evidence in arthroscopic meniscal surgery: a systematic review of the evidence for operative management of meniscal tears. Am J Sports Med. 2016;pii: 0363546516650180. [Epub ahead of print]

8. Sihvonen R, Paavola M, Malmivaara A, et al; Finnish Degenerative Meniscal Lesion Study (FIDELITY) Group. Arthroscopic partial meniscectomy versus sham surgery for a degenerative meniscal tear. N Engl J Med. 2013;369:2515-2524.

9. Beaufils P, Hulet C, Dhénain M, et al. Clinical practice guidelines for the management of meniscal lesions and isolated lesions of the anterior cruciate ligament of the knee in adults. Orthop Traumatol Surg Res. 2009;95:437-442.

Article PDF
Author and Disclosure Information

University of Illinois at Chicago (Dr. Jarrett); University of Pittsburgh Medical Center St. Margaret Family Medicine Residency Program, Pa (Dr. Sauereisen)

DEPUTY EDITOR
Corey Lyon, DO

University of Colorado Family Medicine Residency, Denver

Issue
The Journal of Family Practice - 66(4)
Publications
Topics
Page Number
250-252
Sections
Files
Files
Author and Disclosure Information

University of Illinois at Chicago (Dr. Jarrett); University of Pittsburgh Medical Center St. Margaret Family Medicine Residency Program, Pa (Dr. Sauereisen)

DEPUTY EDITOR
Corey Lyon, DO

University of Colorado Family Medicine Residency, Denver

Author and Disclosure Information

University of Illinois at Chicago (Dr. Jarrett); University of Pittsburgh Medical Center St. Margaret Family Medicine Residency Program, Pa (Dr. Sauereisen)

DEPUTY EDITOR
Corey Lyon, DO

University of Colorado Family Medicine Residency, Denver

Article PDF
Article PDF
 

ILLUSTRATIVE CASE

A 48-year-old man presents to your office for follow-up of right knee pain that has been bothering him for the last 12 months. He denies any trauma or inciting incident for the pain. On physical exam, he does not have crepitus, but has medial joint line tenderness of his right knee. A magnetic resonance image (MRI) shows a partial, medial meniscal tear. Do you refer him to Physical Therapy (PT) or Orthopedics for arthroscopy and repair?

The meniscus—cartilage in the knee joint that provides support, stability, and lubrication to the joint during activity—can tear during a traumatic event or because of degeneration over time. Traumatic meniscal tears typically happen to younger adults and teens (<30 years of age) during sports, such as basketball and soccer,whereas degenerative meniscal tears generally present in patients ages 40 to 60 years.2,3 The annual incidence of all meniscal tears is 79 per 100,000.4 While some physicians can diagnose traumatic meniscal tears based on history and physical examination, degenerative meniscal tears are generally more challenging, and typically warrant an MRI for confirmation.3

Meniscal tears can be treated either conservatively, with supportive care and exercise, or with surgery. Unfortunately, there are no national orthopedic guidelines available to help direct care. In one observational study of surgery as treatment for both traumatic and degenerative meniscal tears, 95 out of 117 patients (81.2%) were generally satisfied with this treatment at the 4-year follow-up, with higher satisfaction in the traumatic meniscal tear group than in the degenerative tear group.5

Two systematic reviews of surgery vs nonoperative management or sham therapies found no additional benefit of surgery for meniscal tears in a variety of patients with and without osteoarthritis.6,7 However, both studies were of only moderate quality because of the number of patients in the nonoperative groups who ultimately obtained surgery. And neither of the studies directly compared surgery to nonoperative management.6,7

Yet another investigation, a multicenter, randomized, double-blind, sham-controlled study conducted in Finland involving 146 patients, compared sham surgery to arthroscopic partial meniscectomy. Both groups received instruction on performing post-procedure exercises, and both groups had similar and marked improvement in pain and function.8

Clinical practice recommendations devised from a systematic and vast review of the literature recommend that the decision for surgery be based on patient-specific factors such as symptoms, age, mechanism of tear, extent of damage, and occupational/­social/activity needs.9

STUDY SUMMARY

Exercise is as good as—and in one way, better than—surgery

The current randomized controlled superiority trial compared exercise therapy to arthroscopic partial meniscectomy in patients ages 35 to 60 years presenting to the orthopedic departments of 2 hospitals in Norway with unilateral knee pain for more than 2 months and an MRI-delineated medial meniscal tear. Patients were included only if they had radiographic evidence of minimal osteoarthritis (Kellgren-Lawrence classification grade ≤2). Exclusion criteria were acute trauma, locked knee, ligament injury, and knee surgery in the same knee within the previous 2 years.

The primary outcomes were change in patient-reported knee function as determined by overall knee injury and osteoarthritis outcome score (KOOS4) after 2 years and thigh muscle strength at 3 months as measured by physiotherapists. The KOOS4 consists of 4 out of the 5 KOOS subscales: pain, other symptoms (swelling, grinding/noise from the joint, ability to straighten and bend), function in sports/recreation, and knee-related quality of life (QOL). This study utilized the average score of each subscale.

Secondary outcomes were the 5 individual KOOS subscales (the 4 previously mentioned plus activities of daily living [ADLs]), as well as thigh muscle strength and lower extremity performance test results.

Methods. Testing personnel were blinded to group allocation; participants wore pants or neoprene sleeves to cover surgical scars. A total of 140 patients were randomized to either 12 weeks (24-36 sessions) of exercise therapy alone or a standardized arthroscopic partial meniscectomy with written and oral encouragement upon discharge to perform simple exercises at home 2 to4 times daily (to regain range of motion and reduce swelling).

Results. The overall mean improvement in KOOS4 score from baseline at 2 years was similar between the exercise group and the meniscectomy group (25.3 points vs 24.4 points, respectively; mean difference [MD], 0.9; 95% confidence interval [CI], -4.3 to 6.1; P=.72). Additionally, muscle strength (measured as peak torque flexion and extension and total work flexion and extension) at both 3 and 12 months showed significant objective improvements favoring exercise therapy.

Exercise therapy was as effective as surgery after a 2-year follow-up period and was superior in the short term for thigh muscle strength.

Secondary outcomes comparing the change from baseline of KOOS subscale scores showed 4 of the 5 having non-significant differences (pain, ADL, sports/­recreation, and QOL). Only the symptoms subscale had a significant difference favoring exercise therapy (MD, 5.3 points; 95% CI, 0.5 to 10.2; P=.03), which was likely clinically insignificant when using a grading scale of 0 to 100.

Of those patients allocated to exercise therapy alone, 19% crossed over and underwent surgery during the 2 years of the study.

 

 

 

WHAT'S NEW

Head-to-head comparison adds evidence to previous findings

This is the first trial to directly compare exercise therapy to surgery in patients with meniscal tears. Interestingly, exercise therapy was as effective after a 2-year follow-up period and was superior in the short term for thigh muscle strength.1 The results of this study build on those from the smaller study conducted in Finland mentioned earlier.8 In that study, both groups received instruction for the same graduated exercise plan. The researchers found that exercise was comparable to surgery for meniscal tears in patients with no osteoarthritis.

CAVEATS

Results may not translate to those with more severe osteoarthritis

This trial included patients with only mild to no osteoarthritis in addition to their meniscal tear.1 It is unclear if the results would be maintained in patients with more advanced disease. Additionally, 19% of patients crossed over from the exercise group to the surgery group, even though muscle strength improved. Therefore, education about the risks of surgery and the potential lack of benefit is important.

CHALLENGES TO IMPLEMENTATION

The cost and effort of physical therapy may be a deterrent

The cost of PT can be a barrier for some patients who have adequate insurance coverage for surgery, but inadequate coverage for PT. Additionally, exercise therapy requires significant and ongoing amounts of time and effort, which may be a deterrent for patients with busy lifestyles. Patients and physicians may view surgery as an “easier” fix.

ACKNOWLEDGEMENT

The PURLs Surveillance System was supported in part by Grant Number UL1RR024999 from the National Center For Research Resources, a Clinical Translational Science Award to the University of Chicago. The content is solely the responsibility of the authors and does not necessarily represent the official views of the National Center For Research Resources or the National Institutes of Health.

 

ILLUSTRATIVE CASE

A 48-year-old man presents to your office for follow-up of right knee pain that has been bothering him for the last 12 months. He denies any trauma or inciting incident for the pain. On physical exam, he does not have crepitus, but has medial joint line tenderness of his right knee. A magnetic resonance image (MRI) shows a partial, medial meniscal tear. Do you refer him to Physical Therapy (PT) or Orthopedics for arthroscopy and repair?

The meniscus—cartilage in the knee joint that provides support, stability, and lubrication to the joint during activity—can tear during a traumatic event or because of degeneration over time. Traumatic meniscal tears typically happen to younger adults and teens (<30 years of age) during sports, such as basketball and soccer,whereas degenerative meniscal tears generally present in patients ages 40 to 60 years.2,3 The annual incidence of all meniscal tears is 79 per 100,000.4 While some physicians can diagnose traumatic meniscal tears based on history and physical examination, degenerative meniscal tears are generally more challenging, and typically warrant an MRI for confirmation.3

Meniscal tears can be treated either conservatively, with supportive care and exercise, or with surgery. Unfortunately, there are no national orthopedic guidelines available to help direct care. In one observational study of surgery as treatment for both traumatic and degenerative meniscal tears, 95 out of 117 patients (81.2%) were generally satisfied with this treatment at the 4-year follow-up, with higher satisfaction in the traumatic meniscal tear group than in the degenerative tear group.5

Two systematic reviews of surgery vs nonoperative management or sham therapies found no additional benefit of surgery for meniscal tears in a variety of patients with and without osteoarthritis.6,7 However, both studies were of only moderate quality because of the number of patients in the nonoperative groups who ultimately obtained surgery. And neither of the studies directly compared surgery to nonoperative management.6,7

Yet another investigation, a multicenter, randomized, double-blind, sham-controlled study conducted in Finland involving 146 patients, compared sham surgery to arthroscopic partial meniscectomy. Both groups received instruction on performing post-procedure exercises, and both groups had similar and marked improvement in pain and function.8

Clinical practice recommendations devised from a systematic and vast review of the literature recommend that the decision for surgery be based on patient-specific factors such as symptoms, age, mechanism of tear, extent of damage, and occupational/­social/activity needs.9

STUDY SUMMARY

Exercise is as good as—and in one way, better than—surgery

The current randomized controlled superiority trial compared exercise therapy to arthroscopic partial meniscectomy in patients ages 35 to 60 years presenting to the orthopedic departments of 2 hospitals in Norway with unilateral knee pain for more than 2 months and an MRI-delineated medial meniscal tear. Patients were included only if they had radiographic evidence of minimal osteoarthritis (Kellgren-Lawrence classification grade ≤2). Exclusion criteria were acute trauma, locked knee, ligament injury, and knee surgery in the same knee within the previous 2 years.

The primary outcomes were change in patient-reported knee function as determined by overall knee injury and osteoarthritis outcome score (KOOS4) after 2 years and thigh muscle strength at 3 months as measured by physiotherapists. The KOOS4 consists of 4 out of the 5 KOOS subscales: pain, other symptoms (swelling, grinding/noise from the joint, ability to straighten and bend), function in sports/recreation, and knee-related quality of life (QOL). This study utilized the average score of each subscale.

Secondary outcomes were the 5 individual KOOS subscales (the 4 previously mentioned plus activities of daily living [ADLs]), as well as thigh muscle strength and lower extremity performance test results.

Methods. Testing personnel were blinded to group allocation; participants wore pants or neoprene sleeves to cover surgical scars. A total of 140 patients were randomized to either 12 weeks (24-36 sessions) of exercise therapy alone or a standardized arthroscopic partial meniscectomy with written and oral encouragement upon discharge to perform simple exercises at home 2 to4 times daily (to regain range of motion and reduce swelling).

Results. The overall mean improvement in KOOS4 score from baseline at 2 years was similar between the exercise group and the meniscectomy group (25.3 points vs 24.4 points, respectively; mean difference [MD], 0.9; 95% confidence interval [CI], -4.3 to 6.1; P=.72). Additionally, muscle strength (measured as peak torque flexion and extension and total work flexion and extension) at both 3 and 12 months showed significant objective improvements favoring exercise therapy.

Exercise therapy was as effective as surgery after a 2-year follow-up period and was superior in the short term for thigh muscle strength.

Secondary outcomes comparing the change from baseline of KOOS subscale scores showed 4 of the 5 having non-significant differences (pain, ADL, sports/­recreation, and QOL). Only the symptoms subscale had a significant difference favoring exercise therapy (MD, 5.3 points; 95% CI, 0.5 to 10.2; P=.03), which was likely clinically insignificant when using a grading scale of 0 to 100.

Of those patients allocated to exercise therapy alone, 19% crossed over and underwent surgery during the 2 years of the study.

 

 

 

WHAT'S NEW

Head-to-head comparison adds evidence to previous findings

This is the first trial to directly compare exercise therapy to surgery in patients with meniscal tears. Interestingly, exercise therapy was as effective after a 2-year follow-up period and was superior in the short term for thigh muscle strength.1 The results of this study build on those from the smaller study conducted in Finland mentioned earlier.8 In that study, both groups received instruction for the same graduated exercise plan. The researchers found that exercise was comparable to surgery for meniscal tears in patients with no osteoarthritis.

CAVEATS

Results may not translate to those with more severe osteoarthritis

This trial included patients with only mild to no osteoarthritis in addition to their meniscal tear.1 It is unclear if the results would be maintained in patients with more advanced disease. Additionally, 19% of patients crossed over from the exercise group to the surgery group, even though muscle strength improved. Therefore, education about the risks of surgery and the potential lack of benefit is important.

CHALLENGES TO IMPLEMENTATION

The cost and effort of physical therapy may be a deterrent

The cost of PT can be a barrier for some patients who have adequate insurance coverage for surgery, but inadequate coverage for PT. Additionally, exercise therapy requires significant and ongoing amounts of time and effort, which may be a deterrent for patients with busy lifestyles. Patients and physicians may view surgery as an “easier” fix.

ACKNOWLEDGEMENT

The PURLs Surveillance System was supported in part by Grant Number UL1RR024999 from the National Center For Research Resources, a Clinical Translational Science Award to the University of Chicago. The content is solely the responsibility of the authors and does not necessarily represent the official views of the National Center For Research Resources or the National Institutes of Health.

References

1. Kise NJ, Risberg MA, Stensrud S, et al. Exercise therapy versus arthroscopic partial meniscectomy for degenerative meniscal tear in middle aged patients: randomised controlled trial with two year follow-up. BMJ. 2016;354:i3740.

2. Beals CT, Magnussen RA, Graham WC, et al. The prevalence of meniscal pathology in asymptomatic athletes. Sports Med. 2016;46:1517-1524.

3. Maffulli N, Longo UG, Campi S, et al. Meniscal tears. Open Access J Sports Med. 2010;1:45-54.

4. Peat G, Bergknut C, Frobell R, et al. Population-wide incidence estimates for soft tissue knee injuries presenting to healthcare in southern Sweden: data from the Skåne Healthcare Register. Arthritis Res Ther. 2014;16:R162.

5. Ghislain NA, Wei JN, Li YG. Study of the clinical outcome between traumatic and degenerative (non-traumatic) meniscal tears after arthroscopic surgery: a 4-years follow-up study. J Clin Diagn Res. 2016;10:RC01-RC04.

6. Khan M, Evaniew N, Bedi A, et al. Arthroscopic surgery for degenerative tears of the meniscus: a systematic review and meta-analysis. CMAJ. 2014;186:1057-1064.

7. Monk P, Garfjeld Roberts P, Palmer AJR, et al. The urgent need for evidence in arthroscopic meniscal surgery: a systematic review of the evidence for operative management of meniscal tears. Am J Sports Med. 2016;pii: 0363546516650180. [Epub ahead of print]

8. Sihvonen R, Paavola M, Malmivaara A, et al; Finnish Degenerative Meniscal Lesion Study (FIDELITY) Group. Arthroscopic partial meniscectomy versus sham surgery for a degenerative meniscal tear. N Engl J Med. 2013;369:2515-2524.

9. Beaufils P, Hulet C, Dhénain M, et al. Clinical practice guidelines for the management of meniscal lesions and isolated lesions of the anterior cruciate ligament of the knee in adults. Orthop Traumatol Surg Res. 2009;95:437-442.

References

1. Kise NJ, Risberg MA, Stensrud S, et al. Exercise therapy versus arthroscopic partial meniscectomy for degenerative meniscal tear in middle aged patients: randomised controlled trial with two year follow-up. BMJ. 2016;354:i3740.

2. Beals CT, Magnussen RA, Graham WC, et al. The prevalence of meniscal pathology in asymptomatic athletes. Sports Med. 2016;46:1517-1524.

3. Maffulli N, Longo UG, Campi S, et al. Meniscal tears. Open Access J Sports Med. 2010;1:45-54.

4. Peat G, Bergknut C, Frobell R, et al. Population-wide incidence estimates for soft tissue knee injuries presenting to healthcare in southern Sweden: data from the Skåne Healthcare Register. Arthritis Res Ther. 2014;16:R162.

5. Ghislain NA, Wei JN, Li YG. Study of the clinical outcome between traumatic and degenerative (non-traumatic) meniscal tears after arthroscopic surgery: a 4-years follow-up study. J Clin Diagn Res. 2016;10:RC01-RC04.

6. Khan M, Evaniew N, Bedi A, et al. Arthroscopic surgery for degenerative tears of the meniscus: a systematic review and meta-analysis. CMAJ. 2014;186:1057-1064.

7. Monk P, Garfjeld Roberts P, Palmer AJR, et al. The urgent need for evidence in arthroscopic meniscal surgery: a systematic review of the evidence for operative management of meniscal tears. Am J Sports Med. 2016;pii: 0363546516650180. [Epub ahead of print]

8. Sihvonen R, Paavola M, Malmivaara A, et al; Finnish Degenerative Meniscal Lesion Study (FIDELITY) Group. Arthroscopic partial meniscectomy versus sham surgery for a degenerative meniscal tear. N Engl J Med. 2013;369:2515-2524.

9. Beaufils P, Hulet C, Dhénain M, et al. Clinical practice guidelines for the management of meniscal lesions and isolated lesions of the anterior cruciate ligament of the knee in adults. Orthop Traumatol Surg Res. 2009;95:437-442.

Issue
The Journal of Family Practice - 66(4)
Issue
The Journal of Family Practice - 66(4)
Page Number
250-252
Page Number
250-252
Publications
Publications
Topics
Article Type
Display Headline
When can exercise supplant surgery for degenerative meniscal tears?
Display Headline
When can exercise supplant surgery for degenerative meniscal tears?
Sections
PURLs Copyright

Copyright © 2017. The Family Physicians Inquiries Network. All rights reserved.

Inside the Article

PRACTICE CHANGER

Recommend supervised exercise therapy to your patients with a medial, degenerative meniscal tear and a minimal history of osteoarthritis because it is as effective as partial meniscectomy, entails little risk, and has the added benefit of increasing muscle strength.1

STRENGTH OF RECOMMENDATION

B: Based on a single, good quality, randomized controlled trial.

Kise NJ, Risberg MA, Stensrud S, et al. Exercise therapy versus arthroscopic partial meniscectomy for degenerative meniscal tear in middle aged patients: randomised controlled trial with two year follow-up. BMJ. 2016;354:i3740.

Disallow All Ads
Content Gating
No Gating (article Unlocked/Free)
Alternative CME
PubMed ID
28375398
Use ProPublica
Hide sidebar & use full width
render the right sidebar.
Conference Recap Checkbox
Not Conference Recap
Clinical Edge
Article PDF Media
Media Files